Quick viewing(Text Mode)

Passing the General Surgery Oral Board Exam FM.Qxd 10/28/05 02:30 AM Page I

FM.qxd 10/28/05 02:30 AM Page i

Passing the General Oral Board Exam FM.qxd 10/28/05 02:30 AM Page i

Passing the General Surgery Oral Board Exam

Marc Neff, M.D.

Associates in General Surgery, P.A. Cherry Hill, New Jersey FM.qxd 10/28/05 02:30 AM Page iv

Marc Neff, M.D. 2201 Chapel Ave. West Cherry Hill, New Jersey 08002 [email protected]

Library of Congress Control Number: 2005926770

ISBN-10: 0-387-26077-3 ISBN-13: 978-0387-26077-8

Printed on acid-free paper.

©2006 Springer Science+Business Media, Inc. All rights reserved. This work may not be translated or copied in whole or in part without the written per- mission of the publisher (Springer Science+Business Media, Inc., 233 Spring Street, New York, NY 10013, USA), except for brief excerpts in connection with reviews or scholarly analysis. Use in connection with any form of information storage and retrieval, electronic adaptation, computer software, or by similar or dis- similar methodology now known or hereafter developed is forbidden. The use in this publication of trade names, trademarks, service marks, and similar terms, even if they are not identified as such, is not to be taken as an expression of opinion as to whether or not they are subject to proprietary rights.

Printed in the United States of America. (SPI/BIN)

987654321

springeronline.com FM.qxd 10/28/05 02:30 AM Page v

For Lauren, Jamie, and Andrew who have taught me: “Don’t give up, just go on.” FM.qxd 10/28/05 02:30 AM Page vii

Acknowledgements

This book would not have been possible without the Thanks to the attendings, students, residents, and remarkable support of countless individuals. I know I will patients that I’ve had the pleasure of knowing during my leave some out, and I hope those I do will accept my medical career. They have provided me on a daily basis apologies. with the constant intellectual stimulation to challenge The first “thanks” goes to my immediate family. They what I know and do. They make my job the best job in the have provided the support that helps me get up again world. They also remind me to never be satisfied in my each and every morning and remind me that there is life level of knowledge and inspire me to read through the outside medicine. Of all, my deceased father and brother piles of journals in my office and at home. deserve the most thanks. It was they who gave me the Finally, thanks to the many healthcare professionals that inspiration to pursue a career in medicine and continue have guided my day-to-day experiences in medicine. to inspire my pursuit of excellence. They both were well Without the secretaries, nurses, nurse managers, operators, respected surgeons in their field and died long before coders, billers, administrators, phlebotomists, respiratory their time. therapists, dictationists, librarians, and other unsung heroes, Thanks to my friends. I have been fortunate to have so the great machinery of the hospital would grind to a halt many friends in my life, each of whom has enriched my and we doctors would not be able to function. I remember experiences in innumerable ways. From elementary school learning volumes from several experienced nurses who through residency, I have truly been blessed with friends I helped to keep me out of trouble during my residency years. could trust and depend on. They have contributed greatly I would not be where I am today if we were not all on to the richness of my life. Sadly, as our paths diverged, the same team. Even if it means sometimes fighting a losing some friends were left behind and I miss them all. battle, together we are all trying to make sick patients well.

vii FM.qxd 10/28/05 02:30 AM Page ix

Contents

Introduction xi Organizational Theme xiii Zenker’s Diverticulum 56 Achalasia 58 Malignancy, General 1 Esophageal Cancer 60 Esophageal Perforation 62 Breast Esophageal Varices 64 Nipple Discharge 3 GERD 66 DCIS (Ductal Carcinoma In Situ) 5 Hiatal 68 Inflammatory Breast Cancer 7 Invasive Ductal Carcinoma 9 Genitourinary Paget’s disease 11 Renal Mass 70 Scrotal Mass 71 Colon and Small Bowel Acute Bowel Ischemia 12 Hepatobiliary Colon Cancer 14 Gallstone Ileus 72 Enterocutaneous Fistula 17 Abscess 74 Hemorrhoids 19 Liver Mass 76 Incarcerated Hernia 21 Post- Cholangitis 78 Intestinal Angina 23 Post-cholecystectomy Jaundice 80 Large Bowel Obstruction 25 Lower GI Bleeding (LGIB) 27 Perirectal Abscess 29 Acute Pancreatitis 82 Rectal Cancer 31 Chronic Pancreatitis 84 RLQ Pain 33 Pancreatic Cancer 86 Ulcerative Colitis 35 Pancreatic Pseudocyst 88

Endocrine Pediatric Surgery Carcinoid 37 Neonatal Bowel Obstruction 90 Cushing’s Syndrome 39 Pyloric Stenosis 92 Hyperthyroidism 41 Tracheoesophageal Fistula 94 Insulinoma 43 Pheochromocytoma 45 Perioperative Care Primary Aldosteronism 47 Hypotension in the Recovery Room 96 Neck Mass 49 Postoperative Fever 98 Hyperparathyroidism 52 Recent MI 100 Thyroid Nodule 54 Renal Failure 101

ix FM.qxd 10/28/05 02:30 AM Page x

x Contents

Skin and Soft Tissue Extremity Compartment Syndrome 128 Melanoma 103 Duodenal Trauma 130 Sarcoma 105 GU Trauma 132 Skin Cancer (Other than Melanoma) 107 Liver Trauma 134 Pelvic Fracture 136 and Duodenum Penetrating Neck Trauma 138 Duodenal Ulcer 109 Pulmonary Embolism 140 Gastric Cancer 112 Splenic Trauma 142 Gastric Ulcer 114 Thoracic Trauma 144 Mallory–Weiss Tear 116 Upper GI Bleeding 118 Vascular Abdominal Aortic Aneurysm 146 Acute Lower Extremity Ischemia 148 Thoracic Carotid Stenosis 150 Emphysema 120 Chronic Lowe Extremity Ischemia 152 Lung Cancer 122 Venous Stasis Ulcer 154

Trauma and Critical Care Conclusion and Common Curveballs 156 Abdominal Compartment Syndrome (ACS) 124 Colon and Rectal Trauma 126 Index 159 FM.qxd 10/28/05 02:30 AM Page xi

Introduction

We regret to inform you that you were not successful in the or that I would mismanage or kill my next hundred Certifying Examination given in Cleveland, OH, in October patients. For me, failing on my first try meant that I was 2002. It was the consensus of your examiners that your per- going to “kick it” to those examiners during the next exam. formance during the examination was not of the level I will share with you a philosophy from my upbringing in required for certification. Philadelphia, home to many underdogs over the years: Who That’s the way the letter reads if you do not pass the knows how to climb a ladder better—the person who climbed General Surgery Oral Exam. Three more paragraphs fol- it once and never missed a step, or the person who climbs it, lowed in that awful letter I read to myself on a cold fall falls, and climbs it again, paying close attention to every rung evening in November 2002, less than four days after I had because he knows what it feels like to fall and is determined to taken the exam. I wondered why I wasn’t more upset. Was succeed? it because a good friend had informed me earlier in the day I can offer a couple of general suggestions to those of that he too had failed? Was it that my gut instinct since I you preparing for this exam: left the Oral Exam had been telling me to prepare myself (1) Read a general surgery textbook cover to cover (it for bad news? Who knows. After reading that letter, how- really doesn’t matter which text you choose), ever, I did know one thing: on my next try, I would know (2) Read lots of previous questions, which you can get everything there was to know about the field of General from any course or your colleagues, Surgery so there would be no possible way for me to fail a (3) Remember that self-induced anxiety is your biggest second time. enemy! Out of that sentiment came the thoughts for this book. I couldn’t sleep well the night after I opened that letter. As I If you passed the written exam, you know the material. thought about what I had done to prepare for the exam— You just have to keep from freezing or getting tongue-tied two review courses, flashcards, a variety of review texts— when you are asked to sum up verbally two or more weeks I realized that my biggest help had been a book entitled Safe of outpatient work-up/inpatient care in about seven min- Answers for the Board. It was an excellent resource that utes per question. The Oral Surgery Certification Exam is helped to clarify and crystallize a lot of what I learnt in res- overly subjective, so regardless of what you are asked idency and I recommend it to all potential examinees. remember that this is a test of your thinking ability and However, after a search on the Internet I discovered there is confidence more than a pure test of your knowledge. no book that tells you what the wrong answers are, or reveals Consider each question as a real-life situation. You are the common curveballs examiners are likely to throw your not an unsafe surgeon, and you should treat the questions way. My goal then became to put together a study guide that the way you would a patient: never make up answers or not only included much of the material necessary to pass operations, and don’t waste time on history and physical the Oral Exam, but that would also prepare an examinee for examination if they tell you “that is all you need.” The what actually happens during the exam. examiners are looking to see whether you can process I like to think that the underdog always proves to be the information and come up with a rational plan of action. fiercest competitor. I knew that my failure didn’t mean I You do this everyday. was less of a surgeon than those who passed. I knew it Above all, remember that the exam starts right after they didn’t mean that I wouldn’t become a successful surgeon, shake your hand. With all this in mind, let’s begin . . .

xi FM.qxd 10/28/05 02:30 AM Page xiii

Organizational Theme

Each chapter/topic will be organized as follows: Common Curveballs

Possible ways the Examiners may challenge you. This Concept includes the “change in scenario,” in which an examiner satisfied with your initial answer wants to challenge you Brief pathophysiologic discussion on the general surgery further by changing results of your diagnostic tests or topic. your interventions. Remember, these are possible real-life situations and approach them as you would any patient in the hospital. Don’t expect any question to end without Way Question May be Asked? at least one curveball on the Oral Exam. Space following Common Curveballs has been left blank intentionally so Common scenario presentations and some variations on you can fill in references to your favorite text or other the theme. review material.

How to Answer? Strikeouts

Possible way to answer the question based on the author’s Answers if you miss, say, or fail to say, you’re likely com- summary of several references, review courses, and multiple ing back next year to try the oral exam again. examinee experiences (those of the author and the author’s colleagues).

xiii Part 1.qxd 10/19/05 2:51 AM Page 1

Malignancy, General

Concept (c) CT scan abd/pelvis, angio, tumor markers, ERCP in pancreatic CA Make sure you understand the common nature of spread (3) Make sure to determine if lesion is resectable or if of the cancer (hematogenous—papillary/thyroid; peri- pt needs pre-op chemo/XRT toneal—ovarian; lymphatic—breast). All cancers need to (a) rectal CA Stage II or above gets pre-op XRT be addressed with regard to: (b) inflammatory breast CA gets pre-op chemo In the OR, if you don’t have a dx yet (as in case of pan- Staging, creatic mass), must do frozen section Surgical treatment, Frozen section also appropriate after resection to check Neoadjuvant/adjuvant therapies. margins in gastric, esophageal, lung CA Examination of lymph node basins when appropriate Frozen section of SLNs (controversial—careful here in Way Question May be Asked? any CA but melanoma and breast! Not necessarily a right answer, but know your answer and stick to it!) About half the time will be clearly cancer from the outset. Be able to describe common lymph node dissections The other half, you will get the diagnosis after a long, Don’t forget to ask for pathology report—size, margins, exhaustive work-up. lymph nodes, tumor type, nuclear grade (receptor status for breast cancer) How to Answer? Don’t forget to discuss post-op chemo/XRT management As with everything else, be methodical Don’t leave out of history: important systemic symptoms like change in bowel habits, dysphagia, weight loss, Common Curveballs anorexia, jaundice, last mammograms, family history of malignancies. Cancer diagnosis unable to determine pre-op (common with pancreatic/cholangioCA) Don’t leave out of physical exam: important informa- Margins positive in gastric/breast cancer tion like abdominal masses, examination of important Mediastinoscopy positive lymphatic basins, complete skin exam in someone with Post-op 1 yr with local recurrence or rising tumor melanoma. markers in first year of post-op follow-up Make sure you do everything you can to work-up pt Post-op 1 yr with metastatic lesion (resect in sarcoma if pre-op both in: primary site controlled and in melanoma if single (1) Determining a diagnosis—FNA, U/S, mammo- organ met) gram in breast CA Post-op discussion of chemo/XRT regimen (2) Appropriate staging—(don’t go overboard with “Scenario switch” where you were working up one diag- ordering tests!) nosis and find a malignancy (bloody nipple d/c after (a) LFTs, CXR for breast cancer resection, path reveals small focus ductal carcinoma) (b) PFTs, CT scan chest to adrenals in lung cancer, Pt will have synchronous tumor in colon CA +/− mediastinoscopy Pt will have post-op leak after GI resection

1 Part 1.qxd 10/19/05 2:51 AM Page 2

2 Malignancy, General

Pt will want to preserve their breast with advanced Failure to know chemo/XRT regimen after resection for breast CA breast CA Pt will have positive SLN on permanent/frozen section Failure to check margins after GI resection Pt will have non-diagnostic FNA or percutaneous Aggressive resection of metastatic lesions in breast CA biopsy Failure to do FNA on palpable thyroid nodule/breast Asked to describe your technique for performing SLN lesion biopsies Failure to get pre-op lymphoscintigraphy if performing SLN for trunk melanoma (can go to at least four dif- ferent lymph node basins) Strikeouts Failing to use pre-op XRT in rectal CA stage II or above Failure to check old CXR if suspect lung CA Failing to evaluate adrenal glands in evaluation of lung Failure to check old mammogram/or order mammo- CA gram in breast CA Failing to determine resectability pre-op in pt with pan- Failure to complete lymph node dissection for positive creatic neoplasm SLN (don’t get into discussion about most recent Failing to examine lymph node basins on pre-op H&P NSABP trials randomizing pts to not have complete Failing to ask about prior hx malignancy in pre-op ALND for + SLN in breast CA) H&P Part 1.qxd 10/19/05 2:51 AM Page 3

Breast—Nipple Discharge

Concept Diagnostic Tests Either from benign or malignant cause. Benign typically are Must get mammogram non-spontaneous, bilateral, clear or milky, and from multiple U/S (subareolar area images poorly on mammogram) ducts. Bloody discharge typically from an intraductal papil- Hemoccult test loma (45%), duct ectasia (35%) or infection (~5%). However, Cytology (rarely helpful, and negative result doesn’t it may be a cancer (~5%); this is a common curveball. exclude malignancy Ductogram (painful, and rarely helpful) MRI (rarely helpful for papilloma, but may detect other Way Question May be Asked? lesions)

“A 45 y/o female presents to your office with the complaint Then, if you have bloody discharge, you are in one of of unilateral bloody nipple discharge for the past one several situations: month.” May be given just nipple discharge and have to (1) Negative mammogram/negative PE for mass/negative work through type, spontaneity, laterality, and recent med- responsible quadrant ications that have been started. May also be given nipple Have pt follow-up in several weeks and check for discharge in young female. responsible quadrant on breast self-exam. Then, on follow-up: How to Answer? (a) Negative mammogram/PE for mass/negative responsible quadrant Full History → total subareolar ductal system resection (b) Negative mammogram/PE for mass/positive Risk factors for malignancy responsible quadrant Trauma → subareolar wedge resection ductal system for Fluid characteristics (clear, milky, serous, bloody) that quadrant Bilateral or unilateral (c) Positive mammogram/PE for mass/positive When discharge occurs stimulated or spontaneous responsible quadrant (spontaneous worrisome) → excisional biopsy of mass and subareolar Trauma wedge resection Thyroid disorder (2) Negative mammogram/PE for mass/positive responsi- New medications ble quadrant → subareolar wedge resection of the ductal system Full Physical Exam draining that quadrant (3) Positive mammogram/PE for mass/positive responsi- Examination of both breasts in upright and supine ble quadrant positions → excisional biopsy or core-needle biopsy of mass on Examination of lymph node basins mammogram and subareolar wedge resection Try to determine a responsible quadrant/responsible ducts

3 Part 1.qxd 10/19/05 2:51 AM Page 4

4 Breast—Nipple Discharge

Surgical Treatment Not bloody discharge but persistent atypical cells on slide cytology (now what do you do?) Circumareolar incision (some make incision at nip- Pt will be pregnant ple/areola border) Pt will be teenager Elevate areola Dissect ducts leading to areola Identify abnormal duct by dilatation, stent, dye or mass Strikeouts (if can identify single duct otherwise subareolar wedge resection of the ductal system draining that Performing surgery for non-spontaneous, bilateral, quadrant) clear/milky discharge Tie off distal duct or will still drain out of nipple post- Failing to check the same breast for palpable masses or op (your seroma!) examine the other breast Failing to establish risk factors for malignancy Failing to check nodal status if pathology returns Common Curveballs malignancy Failing to order a mammogram/U/S The pathology won’t be a benign intraductal papilloma Discussing ductoscopy but a type of breast cancer (may range from LCIS Performing mastectomy for bloody nipple discharge and DCIS to invasive cancer→don’t forget about Not being able to shift into discussion of malignancy if checking lymph nodes and adjuvant therapy!) pathology doesn’t reveal expected papilloma, but The nipple discharge will persist after a subareolar rather an invasive carcinoma wedge resection Trusting slide cytology/hemoccult tests and not taking There won’t be a responsible quadrant pt to surgery with suspicious nipple discharge There will be a mass in the same breast, different quad- Wasting time working up a prolactinoma rant, or in the opposite breast Part 1.qxd 10/19/05 2:51 AM Page 5

Breast—DCIS (Ductal Carcinoma In Situ)

Concept Ultrasound useful in palpable masses to determine if cystic or solid Premalignant lesion with various subtypes. Pt has about a MRI not used for screening purposes 1 in 3 chance of developing invasive ductal carcinoma in Any suspicious microcalcifications (clustered, branch- her lifetime. Several key features from pathologic stand- ing, heterogeneous) need to be biopsied (stereotactic point: size of tumor, was it unifocal or multifocal, comedo core needle or needle localization/excisional biopsy) necrosis, how well differentiated was the tumor? After biopsy has identified lesion as DCIS, the pt still needs that area excised with adequate (> 2 mm) free Way Question May be Asked? margins. If you don’t get this after your needle-loc, you’ll need to re-excise until you begin to distort the breast, or you get free margins “51 y/o female presents to your office with an abnormal If DCIS is diffuse: multifocal (scattered in one quad- mammogram. A cluster of 5 microcalcifications were seen rant) or other quadrants (multicentric), consider in the UOQ of the left breast. She underwent a core-nee- total mastectomy dle biopsy that revealed DCIS. What would you do?” May If the tumor is high grade, has comedo necrosis, or is be given DCIS in a number of different ways from mam- large, a total mastectomy is appropriate (no ALND mogram showing asymmetric density, nodule, speculated necessary here unless final path reveals invasive car- lesion, but most commonly from cluster or branching het- cinoma)—be sure to offer immediate reconstruction erogenous microcalcifications. as an option Pt will need post-op XRT (unless had mastectomy or How to Answer? has low grade, small tumor with > 1 cm margin) to breast and be placed on 5 years Tamoxifen (unless History contraindication like endometrial CA or h/o DVTs) Establish risk factors for breast cancer (menarche, breast- feeding, family history of breast/ovarian/ prostate can- cer, number of children, previous breast cancer,...) Common Curveballs

Physical Exam There will be a palpable mass (separate from mammo- graphic finding) Symmetry, dimpling, erythema There will be more than one mammographically Try to palpate for any masses detected lesion Check both breasts! There will be lesion in opposite breast Examine for adenopathy Pt will have recurrence after mastectomy to chest wall or incision site (changing scenario) How to Answer? Pt will have invasive carcinoma (changing scenario) On pathology, resection margin will be positive or less Need to order bilateral mammograms and compare to than 1 mm previous Pt will be pregnant

5 Part 1.qxd 10/19/05 2:51 AM Page 6

6 Breast—Ductal Carcinoma In Situ

Stereotactic core can’t be performed (too superficial or Forgetting post-op chemo/XRT treatment when appro- too deep or pt can’t lay prone on stereotactic table) priate Lobular carcinoma in situ on final pathology (maybe Forgetting ALND if invasive cancer identified even at margins) Performing ALND for DCIS Talking about SLN Bx for comedo DCIS (only in research protocols currently) Strikeouts Talking about use of chemotherapy/Arimidex/or the new med you read about in the journal last week as Forgetting to examine both breasts an experimental trial for your pt with DCIS Forgetting to order bilateral mammograms Not performing re-excision for margin < 1 mm. Part 1.qxd 10/19/05 2:51 AM Page 7

Breast—Inflammatory Breast Cancer

Concept Diagnostic Tests (in all Breast Questions!) Poor prognosis regardless of therapy offered. Do want to Mammogram (bilateral) try to provide local control. Need to look for tumor cells U/S (if mass) in subdermal lymphatics and treat aggressively. MRI (usually for palpable lesion not seen on mammo Differential diagnosis includes mastitis, abscess, or U/S) Mondor’s disease, and inflammatory breast CA. DDx Way Question May be Asked? Mastitis Breast abscess “A 58 y/o female presents to your office complaining of a Superficial thrombophlebitis (palpable cord) breast infection. Examination reveals an erythematous, ede- Inflammatory breast cancer matous right breast. What do you want to do?” May also be given a failed course of antibiotics, a history of trauma, or recent breast-feeding/nursing to try to lead you astray. Surgical Treatment (1) Okay to try short course antibiotics (1 week) How to Answer? (2) If fails to resolve or strong suspicion, get incisional biopsy (including skin) through reddened area and History include adjacent normal skin (some recommend Risk factors: FNA because clinical grounds confirm stage of dis- Family history ease and you just want a dx of cancer to start Prior breast surgery chemotherapy, but you get more info from core nee- History of malignancy dle or incisional biopsy—ER/PR receptor status) Age menarche, menopause, 1st pregnancy (3) If pathology confirms inflammatory breast Ca Estrogen use (OCPs) (tumor in subdermal lymphatics), proceed with Important questions: metastatic work-up History of trauma (a) CXR Nursing (b) CT scan head/abd/pelvis (look for metastases) Time course (c) Bone scan Breast self exams (palpable masses before (d) +/− PET scan inflammation?) (4) Three cycles of chemotherapy (usually multi-agent) (5) Algorithm (a) if pt has complete response→ MRM to aug- Physical Exam ment local control, followed by eight cycles of Examine both breasts (peau d’orange) chemo, chest wall radiation, and tamoxifen if Examine lymph node basins (cervical/axillary) ER/PR positive Palpable cord (Mondor’s disease) (b) no response→ chest wall radiation, MRM

7 Part 1.qxd 10/19/05 2:51 AM Page 8

8 Breast—Inflammatory Breast Cancer

(6) If already eroding through skin, can give XRT up Strikeouts front to shrink tumor (also works if grossly eroding through skin and infected) Not performing FNA instead of incisional biopsy (need receptor status) Not recognizing inflammatory breast cancer as a T4 Common Curveballs lesion Not performing biopsy at all but proceeding straight to Will erode through skin during treatment chemotherapy Pt will not have response to chemo Not performing mastectomy at end of neoadjuvant Pt will be pregnant therapy (even if complete clinical resolution) Pt will somewhat respond during antibiotic treatment Not treating first with chemotherapy but proceeding Pt will have mass/abnL mammogram for opposite straight with mastectomy breast Talking about MRI (PET scan only be appropriate here Pt will develop DVT during chemo (switch scenario) for complete staging purposes) Pt will push towards saving her breast or immediate Trying breast conservation/breast reconstruction reconstruction (NO!) Trying to perform SLN Bx’s FNA positive but can’t get any receptor information (need to do core or incisional biopsy) Pt will develop lymphedema post ALND Part 1.qxd 10/19/05 2:51 AM Page 9

Breast—Invasive Ductal Carcinoma

Concept How to Answer Malignancy that needs complete staging work-up and then Need to order bilateral mammograms and compare to adjuvant treatment. Most women are candidates for breast any previous conservation therapy (BCT). Need to know the absolute Ultrasound useful in palpable masses to determine if and relative contraindications to BCT. cystic or solid (especially in premenopausal breasts and may show characteristics of malignancy) FNA can be done in office setting of any palpable Way Question May be Asked? lesion Core-needle bx can be done in office or under stereo- “45 y/o female presents to your office with a palpable mass tactic/U/S guidance in the UOQ of the right breast. What would you do?” Will Excisional biopsy should be performed on: likely be presented with pt with either a palpable abnor- mality, a locally advanced lesion, or a suspicious mammo- Solid mass graphic abnormality. Just be systematic and do what you Cyst with bloody content would normally do in your practice. Cyst that recurs more than twice If FNA reveals malignancy, then can plan full cancer staging in one trip to the OR. How to Answer? Contraindications for breast conservation therapy: History Tumor ≥ 5 cm Establish risk factors for breast cancer (menarche, Large tumor to breast ratio (cosmetic outcome) breast-feeding, family history of breast/ovarian/ Two or more primary tumors in separate quadrants prostate cancer, number of children, age first preg- (multifocal) nancy previous breast cancer, previous breast prob- Previous breast irradiation (from prior BCT) lems, etc.) Collagen vascular disease (scleroderma or lupus— Sx’s: bone pain, wt loss, change in breast appearance can’t receive XRT) Diffuse suspicious or indeterminate calcifications Subareolar tumor Physical Exam Surgical Treatment Symmetry, dimpling, erythema, edema Try to palpate any mass (hard/soft, well circumscribed?, (1) Lumpectomy (with clear margins), ALND, and mobile/fixed, tender) post-op XRT Check both breasts! (2) MRM (combines total mastectomy and ALND) Examine for cervical/axillary adenopathy ALND includes level 1 and 2 (lateral to and behind the Examine liver pectoralis minor muscle) and should be done in all pts

9 Part 1.qxd 10/19/05 2:51 AM Page 10

10 Breast—Invasive Ductal Carcinoma

SLN Bx is now accepted technique, but if positive by Sentinel node biopsy won’t work or will be only positive frozen section or final pathology, would proceed to lymph node complete ALND until results of latest NSABP trial Pt will be pregnant (no XRT, SLN bx, or antimetabolite are scrutinized (only mention if you know how to do based chemoTx) SLN Bx and use lymphazurin blue dye and tech- Can give AC after late 1st trimester (only antimetabolite netium-99 sulfur colloid) methotrexate nsafe during pregnancy) No XRT until pt delivers (needs 24 weeks chemo so ok Adjuvant chemotherapy treatment (Adriamycin/Cyto- unless < 14 weeks pregnant) xan = AC) No Tamoxifen or bone scan (1) All premenopausal women with invasive breast Pt will have contraindication to BCT cancer > 1 cm in size Pt will have contraindication to adriamycin (poor EF) (2) All postmenopausal women with positive lymph Pt will initially present with nipple discharge nodes Pt will have clinically positive axillary nodes (3) Postmenopausal women with T2 or greater Pt will have very strong family history (discussion of lesions (> 2 cm in size) BRCA1,2) Adjuvant hormonal treatment (Tamoxifen) Cancer will present in a cyst that had bloody fluid on (1) All premenopausal women with invasive breast FNA cancer > 1 cm in size Retroarealor cancer—will you perform mastectomy? (2) All postmenopausal women (unless contraindi- T1 lesion, and lymph nodes negative in postmenopausal cation) but receptors are unfavorable→ will you give chemo? T2 lesion in postmenopausal and receptors are Adjuvant XRT favorable→ will you give chemo and/or hormonal (1) 5000 rad in divided doses to chest wall in all pts therapy? who underwent BCT (can’t give during preg- T1a lesion (<5mm) in premenopausal woman, will you nancy, but can usually delay until after preg- offer chemo/XRT nancy as need 6 months chemoTx→no therapeutic abortions!!!) (2) When > 4 LNs involved with tumor, XRT to axilla reduces local recurrence Strikeouts Pathology results Need to know tumor characteristics: nuclear grade, Forgetting to examine both breasts vascular invasion, tumor size, ER/PR receptors, Forgetting to order bilateral mammograms S-phase fraction, Her-2 Neu Not asking about receptors on pathology Only go into Sentinel Lymph Node Biopsy if you do Forgetting post-op chemo/XRT treatment when appro- this in your practice and are prepared to perform priate complete ALND in any pt with metastatic cancer Forgetting ALND if invasive cancer identified seen on the sentinel node (whether intra-op frozen Going into lengthy discussion about sentinel lymph section or post-op final histology) node biopsy when you don’t do these routinely in your practice Performing therapeutic abortion for breast Ca in the Common Curveballs pregnant pt Not knowing contraindications to BCT There will be a separate mammographic finding Not knowing who gets adjuvant treatment and with There will be palpable lesion not seen on mammogram what chemo/hormonal agents There will be lesion in opposite breast Not recognizing Stage IIIB (signs of inoperability → Pt will have recurrence after your surgical treatment neoadjuvant chemo→mastectomy →XRT): Don’t do pulmonary/liver metastatectomy Chest wall invasion Margins will be positive for cancer or DCIS (or less Inflammatory breast cancer than 1mm) Ulceration Part 1.qxd 10/19/05 2:51 AM Page 11

Breast—Paget’s Disease

Concept pathology, if Paget’s cells identified, proceed to simple mastectomy (if CA in mastectomy → Malignant cells that have migrated from underlying DCIS specimen don’t forget ALND!) → or invasive cancer. Paget’s cells are identified in the epider- (2) Palpable mass or lesion on mammogram wedge mis. May regress with topical steroids, so don’t prescribe resection of NAC and excisional biopsy of mass, if them. Bilateral eczematous changes to the nipple areolar Paget’s cells identified and mass is invasive cancer, complex (NAC) are likely benign. then MRM (3) Palpable mass or lesion on mammogram→ wedge resection of NAC and excisional biopsy of mass, if Way Question May be Asked? Paget’s cells identified and mass is DCIS, then sim- ple mastectomy “43 y/o female presents to your office with a 4 week history Don’t forget radiation/chemotherapy/hormonal ther- of itching to her left nipple. Examination reveals a red- apy when appropriate for DCIS or underlying inva- dened eczematous L NAC and a 1.5 cm mass in the upper sive cancer outer quadrant approximately 4 cm from the NAC margin. What would you do?” May or may not be associated mass, but always do PE/mammogram/U/S. Common Curveballs

There will be a palpable mass How to Answer? There will be a mammographically detected lesion There will be lesion in opposite breast History Pt will have recurrence after mastectomy to chest wall Establish risk factors for breast cancer (menarche, or incision site (changing scenario) breast-feeding, family history, number of children, previous breast cancer, etc.) Strikeouts

Physical Exam Forgetting to order mammograms Try to palpate a mass Forgetting post-op chemo/XRT treatment when Check both breasts! appropriate Examine for cervical/axillary adenopathy Forgetting ALND if invasive cancer identified Forgetting to obtain usual history/physical exam (estab- lish risk factors, checking masses in both breasts) How to Answer? Forgetting to examine both breasts/axillae Need to order bilateral mammograms Treating nipple with steroids (Paget’s can remit on Now, a couple of situations possible: steroids) (1) No palpable mass, no lesions on mammography→wedge resection of NAC, check

11 Part 1.qxd 10/19/05 2:51 AM Page 12

Colon and Small Bowel—Acute Bowel Ischemia

Concept Classic: abrupt onset abdominal pain, diarrhea, hema- tochezia Pathogenesis includes multiple etiologies but can generally Recent surgery (AAA, bypass) be broken down into occlusive and nonocclusive types. Recent MI (embolus) DDx for a patient with suspected colonic ischemia should Classic triad of: fever, abdominal pain, and heme + stools also include other colonic disorders such as ulcerative coli- Abrupt onset of: pain, diarrhea, hematochezia tis, infectious colitis, and pseudomembranous colitis. A breakdown of important types of occlusive and nonocclu- Physical Exam sive ischemia follows: “Toxic” appearance, shock, acidosis, leukocytosis Occlusive Nonocclusive Keep in mind: “Pain out of proportion to physical Embolism Hypovolemia exam” Thrombosis Cardiac failure/cardiogenic Peritonitis Vascular compression shock Heme + Stool AAA repair Hypotension Gross blood (usually late finding) Watershed areas vulnerable to low flow states in the Irregular heart rate (a. fib) colon: Griffith’s point (splenic flexure) and Sudeck’s point (rectosigmoid) for nonocclusive ischemia. Diagnostic Tests Full labs (amylase and lactate also helpful)—acidosis a late finding Way Question May be Asked? EKG (r/o a. fib) Abd x-ray: free air, pneumatosis intestinalis, portal vein air “Called to see a 54 y/o male POD#2 after an uncompli- CT scan: “thumb printing”, bowel wall thickening, cated AAA repair with a massive bloody bowel move- pneumatosis, portal vein air ment. What do you want to do?” Situation could also be : mucosal edema, submucosal hemor- after recent open heart surgery, recent MI, or a more rhage, mucosal ulceration, bluish-black discol- chronic form with post-prandial pain for several months oration, areas of black nonviable mucosa, may be with associated weight loss. Remember “pain out of pro- skip areas (keep air insufflation to a minimum,may portion to physical exam” is classic for acute bowel prep with gentle tap water enema) ischemia. Try to separate generalized intestinal ischemia Arteriography (to evaluate small bowel): from colonic ischemia and occlusive from low flow Could see SMA embolus states. Could see SMA thrombosis Could see normal proximal vessels but then distal spasm How to Answer? History Surgical Treatment Risk factors: valvular disease, CAD, hypercoagulable (1) Decide if this is acute small bowel mesenteric occlu- state, cardiac arrhythmias sion vs. colonic ischemia

12 Part 1.qxd 10/19/05 2:51 AM Page 13

Strikeouts 13

(2) Initially Facts on Aortic Surgery: Volume support (may need SGC), ICU, O2,bowel rest, NGT, Foley Ischemia complicates of 1–2% elective cases Serial labs/exams 50% mortality rate Abx when remarkable endoscopic findings or evi- Early colonoscopy dence of toxemia reimplantation IMA when: severe SMA dx, enlarged IMA, loss of Doppler (3) If colonic ischemia improves in sigmoid mesentery, hx of prior colon resec- colonoscopy 6–8 weeks after for evaluate for resolu- tion, poor IMA back bleeding (stump pres- tion/sequelae (stricture) 5% pt with recurrent sure < 40 mmHg) episodes If need to take back to OR, perform end (4) If becomes toxic or peritoneal signs and Hartman pouch Resuscitate pt and prepare for OR In OR: Control contamination Common Curveballs Palpation of celiac, SMA, IMA pulses Pattern of ischemia may suggest etiology (com- Pt will have had prior surgery plete vs. patchy) Pt will have SMA embolus Hand-held Doppler, Fluorescein injection/ Pt will have SMA thrombosis Wood’s lamp, warm packs Will be SMV thrombosis + − Left colon involvement: resection w/colostomy + Pt will need resuscitation pre-op / SGC mucous fistula or Hartmann’s pouch Pt will have necrotic bowel at second look Right colon involvement: resection with Pt will have hypercoagulable syndrome and mucous fistula Pt will have had recent AAA repair with worry about Second look if any question of via- graft contamination bility Whole small bowel will initially appear necrotic Asked how to identify SMA (elevate transverse colon, (5) For small bowel ischemia, if pt toxic or positive follow middle colic to SMA, will need to make inci- angiogram (angiogram helpful in situations of sus- sion in of mesentery, artery is medial to pected small bowel ischemia, not for ischemic coli- SMV) tis), proceed to OR: Pt will return 6 weeks after colonic ischemia treated Prep access to greater saphenous vein in thigh non-operatively with stricture Expose SMA (a) SMA embolus (proximal braches of SMA are spared) Strikeouts Seen 3–8 cm from SMA origin (spares first por- tion of jejunum only) Performing intestinal anastomosis in setting of Embolectomy through transverse arteriotomy ischemia/contamination Heparin post-op Not performing second look if question viability at 1st “Second look procedure” within 24 h operation (b) SMA thrombosis Not knowing how to deal with SMA embolus/throm- Embolectomy to SMA bosis Assess flow Not resuscitating pt pre-op but taking straight to OR If poor, SVG between infrarenal aorta and Discussing urokinase infusion for pt with embolic SMA occlusion of mesenteric vessels or colonic ischemia (can use suprarenal aorta and pass graft behind Delaying on operation when pt is toxic pancreas to SMA) Not resecting necrotic bowel Heparin post-op Not understanding difference between acute mesenteric Resect nonviable segments + “second look pro- ischemia and colonic ischemia cedure” within 24 h Part 1.qxd 10/19/05 2:51 AM Page 14

Colon and Small Bowel—Colon Cancer

Concept Diagnostic Tests Third most common cancer in the US. Because of its fre- Usual labs (including CEA, LFTs) quency, be prepared for the unusual presentations of this CXR malignancy (familial polyposis, invading surrounding Colonoscopy (rule out synchronous lesions) structures, local recurrence, rising CEA levels, metastases +/− Air contrast Barium Enema to the liver). +/− CT scan to r/o metastases (most would do this) What if pt sent to you after polypectomy? (remember Haggit’s classifications) Way Question May be Asked? Needs resection if: Positive margin of resection “54 y/o male presents to the office with a history of iron Invading submucosa deficiency anemia and a colonoscopy performed by the Poorly differentiated referring GI doc reveals a large adenomatous polyp in the Venous/lymphatic invasion cecum.” May have a pt with change in bowel habits, blood Cancer in any sessile polyp in stool, weight loss, abdominal pain, strong family his- tory, presentation similar to perforated diverticulitis, or Surgical Treatment even erosion into genitourinary system (fecaluria or pneu- maturia). Don’t forget mechanical/antibiotic bowel prep (be pre- pared to discuss your preference) Consider pre-op ureteral stents for large/bulky/fixed How to Answer? tumor “No touch” technique never proven to be of any clini- Complete history and physical exam (may have been given cal benefit all this already) Resection of the involved segment of colon, its drain- ing lymphatics, and the segmental blood supply History Tumors of cecum and ascending colon→ right hemicolec- Risk factors (family history, IBD, previous polyps) tomy (ligation of ileocolic, right colic, right branch of Change in bowel habits middle colic, removal of 5–8 cm of ileum to proximal Blood in stool transverse colon) Weight loss Tumors of proximal transverse colon→ extended right Pain hemicolectomy (ligation of ileocolic vessels to middle colic artery, removal of terminal ileum to splenic flexure, anas- Physical Exam tomosis between ileum and descending colon) Tumors of splenic flexure and descending colon→left Abdominal masses hemicolectomy (ligation of left colic with removal of Lymphadenopathy descending colon and splenic flexure with anastomosis of Digital rectal exam transverse to upper sigmoid)

14 Part 1.qxd 10/19/05 2:51 AM Page 15

Strikeouts 15

Tumors of sigmoid and rectosigmoid→sigmoid colec- T4 lesion→XRT to tumor bed to decrease local tomy (ligation of IMA distal to takeoff of left colic, anas- recurrence tomosis between descending colon and upper ) Subtotal → appropriate for pts with synchro- nous cancers, pts with metachronous cancers found after Common Curveballs previous resection, pts with a proximal colon perforation due to an obstructing distal cancer Pt sent to you after polypectomy (what histology gets En bloc resection→ when tumor appears to invade adja- further surgery?) cent organs, this does not preclude resection for cure, mark Cancer will present as large bowel obstruction margins of resection with clips for post-op XRT (T4 but Cancer will present as perforated diverticulitis (careful still Stage II lesion if lymph nodes negative). Check to to check frozen section intra-op and do wide resec- make sure no other metastatic disease first (intra-op U/S of tion and if suspicion of cancer) liver). Cancer will be eroding into surrounding structures If trigone of bladder involved, need cystectomy and (bladder, kidney, duodenum) ileal conduit Cancer will recur locally If invading head of pancreas→ Whipple procedure Rising CEA in 1st year post-op If kidney, check IVP to ensure other kidney okay before There will be peripheral lesion in the liver nephrectomy Pt will have a AAA at time of initial exploration (treat life-threatening problem first...depends on size of → → AAA 8 cm AAA gets priority, near-obstructing Malignant Obstruction colon lesion and 5 cm AAA→ colon lesion gets (1) Can try to convert obstruction lesion +/− colonic priority) stent to near obstructing with NGT, IVF, bowel rest, Pt will have IBD then pre-op bowel and do appropriate resection Ureteral injury during dissection (2) Defunctioning stoma and then second stage resec- Bleeding from spleen after mobilizing the splenic tion and anastomosis flexure (3) Primary resection with anastomosis and on-table Duodenal/vena cava injury when mobilizing right colon lavage thru appendiceal stump and sterilized anes- Synchronous lesions thesia tubing +/− defunctioning stoma (not the con- Prior colon surgery servative answer the examiners like!) Recent MI (4) Subtotal colectomy Unstable pt intra-op Being asked about adjuvant treatment Post-op anastomotic bleed, anastomotic leak, Perforated Lesions→ enterocutaneous fistula, or wound infection (could (1) If perforated left colon CA→ hemicolectomy/ even be necrotizing—be wary for scenario change) Hartmann’s procedure Asked to describe the Duke’s stage of the cancer you (2) If perforated right colon CA→ right hemicolec- are presented tomy with primary anastomosis Asked if you do your own (answer is (3) If perforated cecum and obstructing left colon YES!) CA→ (a) subtotal colecotmy with primary ileorectal anastomosis if stable Strikeouts (b) ileostomy, cecectomy, mucous fistula if unstable and then will need second stage to remove tumor Forgetting colonoscopy to rule out synchronous lesions Not performing staging work-up pre-op Liver lesions→ (1) can resect if less than 4 mets and can Not performing the correct surgical resection leave pt with adequate functional liver (~20%), need to Not performing en bloc resection when cancer has excise with 1 cm margin spread to adjacent organs (2) Don’t perform if pt has extrahepatic disease (peri- Talking about toneal mets, positive lymph nodes, lung lesion) Forgetting bowel prep Talking about cryoablation or RF ablation of liver lesions (while acceptable today for pts who aren’t Adjuvant Treatment surgical candidates, best done in setting of clinical Positive lymph nodes (Stage III pts), invasion of other trials and certainly don’t mention unless you have organs, or distal metastases → 5-FU and levamisole experience with this modality) Part 1.qxd 10/19/05 2:51 AM Page 16

16 Colon and Small Bowel—Colon Cancer

Doing liver resection (or major en bloc resection) when prospective trials, unless you have fellowship training leaving extrahepatic (or other) disease behind and the 20 precepted cases currently recommended Talking about laparoscopic colon resection (while under your belt, you won’t likely be able to defend acceptable today given multiple randomized, this position) Part 1.qxd 10/19/05 2:51 AM Page 17

Colon and Small Bowel—Enterocutaneous Fistula

Pathophysiology Physical Exam Remember FRIEND mnemonic...foreign body, radia- Close examination of wound for erythema, fluid, SQ tion injury, ischemia/IBD, epithelialized tract, neoplasm, air, tenderness, culture any drainage, take out staples and distal obstruction as causes for fistula. Important con- early cepts surrounding enterocutaneous fistulae are controlling infection and drainage, maximizing nutrition/electrolytes, and ruling out distal obstruction/associated abscess. Diagnostic Tests Labs, CXR, AXR, U/A C+S, BCx, (one from any inva- sive catheters) U/S-abdominal/ extremity, CT scan Way Question May be Asked? Start antibiotics if evidence of sepsis Once you have proved to them you understand how “43 y/o female status post an with to work-up post-op fever, then the question will extensive enterolysis and adhesiolysis and repair of multi- likely focus on management of the enterocutaneous ple enterotomies for persistent PSBO develops a fever to fistula: 100.8 and on exam has erythema and tenderness about the Drain fistula with sump drain: vacsponge and pro- lower portion of her incision.” The question typically is tect surrounding skin open ended with first a discussion about how to manage a Measure output of fistula (gives you information post-op fever and then hones in on small bowel contents about likelihood of closure) coming out of the incision the next day (change scenario). Place CVP and restore lytes and intravascular volume NPO, TPN H2 blockers How to Answer? Somatostatin (+/− any real benefit) Abx if associated cellulitis/sepsis First, the basic approach to postoperative fever which must CT scan to r/o abscess and/or place percutaneous be systematic (remember 5Ws): drain Atelectasis Fistulogram after 7–10 days to r/o distal obstruction UTI IV sites Based on fistulogram, one of two possibilities: DVT/PE (1) No distal obstruction or intestinal discontinua- Wound infection tion—most low output enterocutaneous fistulae Anastomotic leak will close within 6 weeks Drug Fever (2) Distal obstruction or failure to close or high out- Rare entities (parotitis, in sinusitis, decubitus ulcer; put fistula need operative repair involving taking acalculous cholecystitis, C. diff, transfusion reaction, down the fistula, , and reanasto- thyroid storm, Addisonian crisis) mosis +/− G- or J-tube if suspected prolonged Symptoms of cough, abdominal pain, shortness of post-op ileus breath, pain at IV sites

17 Part 1.qxd 10/19/05 2:51 AM Page 18

18 Colon and Small Bowel—Enterocutaneous Fistula

Common Curveballs (25% glucose, 3–4% amino acids, 10% FFA want 50% of non-protein calories as glucose and 50% as Pt has SQ emphysema on examination of the wound FFA) and whole focus now shifts to managing a necrotizing soft-tissue infection Pt has associated intraabdominal abscess that can’t be Strikeouts percutaneously drained Complication after placing CVP line (make sure you Failure to do standard fever work-up check CXR) Operating too early for low-output fistula (give TPN CT scan will show multiple fluid collections but no dis- chance to allow fistula to close) crete abscess (always describe how you would order Failure to consider all possible causes (like distal CT scan – “with IV/PO contrast”) obstruction) prior to re-operation Trying to push you into earlier operation Starting long discussion about use of fibrin glue to Skin breakdown at fistula site close fistula tract Same pt, but now with a history of Crohn’s disease Failing to be methodical in care of fistula: resuscitation, Another fistula after you reoperate and take down the controlling fistula, expectant management and sup- first enterocutaneous fistula plemental nutrition/TPN Discussion about TPN (1 gm/kg/day protein, 25 Not being able to discuss basic TPN kcal/kg/day CHO) Part 1.qxd 10/19/05 2:51 AM Page 19

Colon and Small Bowel—Hemorrhoids

Concept May need to do exam under anesthesia Look for malignancy, fistula, other rectal pathology Usually a presentation of pain or bleeding. Need to recog- (careful for scenario switch) nize the difference between internal and external hemor- rhoids. Know the various stages. Treatment is always How to Answer initially conservative. Stages of Internal hemorrhoids (above dentate line and therefore usually painless) Way Question May be Asked? I Painless rectal bleeding II Prolapse with defecation, spontaneously reduce “25 y/o male presents to your office with the complaint of III Same as II, but reduction only manually a thrombosed hemorrhoid. It occurred about four days IV Unable to reduce ago and hurts whenever he sits down or has a bowel move- External hemorrhoids are below the dentate line and ment.” May also present as rectal bleeding, acute throm- hurt when become thrombosed bosis, or incontinence. Non-operative therapy (4–6 weeks) Bulk agents How to Answer? Increasing water intake (6–8 glasses/day) Topical agents (Tucks, Anusol HC, or Analpram) Sitz baths Again, don’t forget your basic H+P or it will turn out to be something other than hemorrhoids Surgical Treatment History For acute thrombosis, bleeding thrombosis, thrombosis Constipation with superficial necrosis→ elliptical surgical excision Pain under local anesthesia in your office Bleeding Stage I and II internal hemorrhoids→ rubber band lig- Topical therapy ation (make sure there is not significant external disease or History rectal complaints any other benign anorectal disease) Family history IBD Banding of only one or two quadrants Prolapse history No banding if pt has any prostheses (heart valve, breast Incontinence implant, pacemaker, joint replacement) Stage III and IV and recurrent symptomatic external Physical Exam hemorrhoids treated with Ferguson closed hemorrhoidec- tomy (elliptical incision over each hemorrhoid down to Examine abdomen sphincter and closure incorporating some of sphincter Rectal exam fibers to prevent prolapse) (with pt in left lateral decub position!) Can also include lateral internal sphincterotomy Rigid

19 Part 1.qxd 10/19/05 2:51 AM Page 20

20 Colon and Small Bowel—Hemorrhoids

During operation: Pt will have internal prostheses and desire hemorrhoidal Make sure to inject perineum, submucosa, and puden- banding dal nerves Pt will have inflammatory bowel disease (no hemor- Tape buttocks apart rhoidectomy!) Excise most symptomatic quadrant first Pt will be pregnant (manage nonoperatively) Excise minimal anoderm (remember risk of stenosis) Post-op urinary retention/bleeding/infection/pelvic sep- sis after closed hemorrhoidectomy Common Curveballs Strikeouts Anal carcinoma Rectal prolapse rather than prolapsing hemorrhoid Operating on thrombosed hemorrhoid after 48 hr Anal fissure Operating on Stage I or II internal hemorrhoids Pt will have postoperative incontinence or anal stenosis Not trying local measures first Pt will have history of portal HTN or be on blood thin- Discussing new PPH therapy ners Not being able to deal with complications of the proce- Pt will be over age 40 (then need to r/o proximal disease dure you choose with BE or colonoscopy before instituting therapy) Operating on pt with IBD Pt will develop postbanding bleeding or infection Not recognizing other anorectal pathology (cancer) Part 1.qxd 10/19/05 2:51 AM Page 21

Colon and Small Bowel—Incarcerated Hernia

Concept CT scan (IV/PO contrast) +/− SBFT in cases of PSBO that persist over 48 h Multiple etiologies, but always remember the most com- mon: and adhesions. Don’t forget the possibility of Surgical Treatment a malignancy, like an obstructing proximal colon cancer and an incompetent ileocecal valve. Important to decide in (1) NGT/NPO/IVF (always!) your own mind how long you will manage a small bowel (2) Serial labs/examinations obstruction non-operatively and what you will do intra-op (3) Volume resuscitation (remember significant third with any compromised or nonviable small bowel. space losses into GI tract!) (4) Peritoneal signs or suggestion of hernia strangulation→ OR Way Question May be Asked? (a) If OR for incarcerated hernia, can do through preperitoneal or traditional inguinal approach “63 y/o female evaluated in the ED for vomiting and (b) If can’t reduce incarcerated , abdominal distension. AXR reveal multiple air/fluid levels. divide inguinal ligament (if strangulated, con- What do you want to do?” Could be asked with a more trol strangulated contents and make lower mid- subtle picture of SBO, or may jump right into a discussion line incision) of management decisions. Don’t spend too much time on (c) Make sure to control sac and open under direct H+P if your examiner clearly doesn’t want you to. vision (d) If contents of hernia drop into abdominal cavity, explore either through preperitoneal incision by How to Answer? opening peritoneum, or using laparoscope (e) Don’t use mesh in situations where possible History ischemia (contamination) Pain (f) If hernia contents ischemic, try: Distension warm packs over area and come back after her- Previous abdominal surgery nia repaired Nausea/vomiting if still ischemic, resect with primary anastomosis

Physical Exam Common Curveballs Vital signs (dehydration, fever) Full exam especially checking for hernias Hernia will be incarcerated/strangulated Hyperactive bowel sounds Strangulated hernia will reduce with induction general Peritoneal signs anesthesia Will need to divide inguinal ligament to free femoral Diagnostic Tests hernia Will need to perform bowel resection Full labs (elevated WBC of peritonitis) PSBO will fail non-operative management Abdominal x-rays (3 views, look at gas pattern) Asked to describe inguinal anatomy 21 Part 1.qxd 10/19/05 2:51 AM Page 22

22 Colon and Small Bowel—Incarcerated Hernia

Won’t be able to use mesh (know at least one non-mesh Not dividing inguinal ligament to free incarcerated repair) femoral hernia Pt will have sliding hernia Not performing bowel resection for obviously ischemic SBFT/CT scan won’t identify point of obstruction bowel Pt will have malignancy Not knowing how to describe any non-mesh hernia Might need to open sac/incision ring to reduce hernia repairs Getting into discussion of using absorbable mesh (Surgissis or Alloderm) in contaminated field with Clean Kills necrotic bowel Not inspecting bowel that was incarcerated Trying to perform laparoscopically (only if you have Performing bowel resection through inguinal incision fellowship training in TAPPs and even then, this is a (some surgeons have successfully done this but not a risky answer!) safe board answer) Not placing NGT or volume resuscitating pt Part 1.qxd 10/19/05 2:51 AM Page 23

Colon and Small Bowel—Intestinal Angina

Concept Bifurcated graft between the supraceliac aorta (approached through the gastrohepatic omentum Chronic occlusion of two of the three main visceral arter- and both the celiac and the SMA) × ies (celiac, SMA, IMA). Postprandial pain related to insuf- Bifurcated 12 7 mm graft ficient blood flow 15–60 minutes after meals. Left limb anastomosed to celiac trunk in end to side fashion with heel on celiac trunk and toe as onlay path onto common hepatic Way Question May be Asked? End to end bypass to SMA done below the body of the pancreas “69 y/o male with history of peripheral vascular disease Aorta-SMA graft commonly placed behind the presents with weight loss and postprandial abdominal common hepatic artery pain.” Don’t expect a clear description of “food fear” or Must divide crus of right diaphragm to expose supaceliac weight loss. Many patients actually develop eating habits aorta to avoid the post-prandial pain including “small meal syn- Must divide Ligament of Trietz to expose SMA drome.” Patient may even present with UGI dysmotility or infrapancreatically ulcers. Good communication with anesthesiologist before supraceliac clamping to permit adequate volume loading and unclamping for expected decrease in BP How to Answer? A retrograde bypass is another option from a healthy Complete history including history of vascular diseases infrarenal aorta and the SMA distal to its occluded segment Complete physical exam including abdominal bruits Could also place a straight graft from infrarenal aorta to Be sure to ask about questions that relate to abdominal the SMA distal to its area of occlusion malignancy IMA reconstruction increases post-op morbidity Diagnostic Tests Follow pt with Duplex U/S prior to hospital discharge and closely post-op (every 6 months) Cardiac workup (on all vascular pts) Visceral duplex showing stenosis (high flow velocities) in celiac and SMA or reversal of flow in the hepatic Common Curveballs artery Gold standard = angiogram with AP and lateral Can’t get a mesenteric duplex at your hospital views Questions about surgical reconstruction of IMA Endovascular techniques an option for high risk pts Bowel injury and can’t use artificial graft Pt will have other pathology on initial ex lap Be able to describe operative technique: Pt has had prior abdominal surgery Transabdominal approach through midline incision Pt has graft thrombosis post-op Antegrade bypass from distal thoracic aorta Pt has acute MI

23 Part 1.qxd 10/19/05 2:51 AM Page 24

24 Colon and Small Bowel—Intestinal Angina

In work-up for chronic mesenteric ischemia, pt Not obtaining angiogram develops acute mesenteric ischemia with bowel Not performing appropriate pre-op workup/clearance necrosis in pt with obvious vascular disease Pt has post-op hepatic or renal failure secondary to supraceliac aortic cross clamp time (tolerance is typ- ically less than 1 hr)

Strikeouts

Not ruling out malignancy in pt with abdominal pain and wt. loss Not being able to describe operative technique Part 1.qxd 10/19/05 2:51 AM Page 25

Colon and Small Bowel—Large Bowel Obstruction

Concept Surgical Treatment Broad DDx but likely malignancy in older population. If no signs of peritonitis: History can be helpful here. You will probably be pushed Gastrografin enema into an operation on someone with obstruction second- CT scan abdomen ary to malignancy or diverticulitis related stricture or NGT/Foley/IVF/NPO/Serial exams perhaps a patient that has even perforated secondary to Try to convert to near-obstructing lesion and their obstruction. perform semi-electively after bowel prep If signs of peritonitis or complete obstruction: Way Question May be Asked? OR after initial evaluation and resuscitation (lines, IVF, Abx) In OR: “61 y/o male evaluated in emergency room with recent (1) Right hemicolectomy for obstructing lesions of constipation and change in bowel habits, complaining of right and proximal transverse colon (can do sudden onset of diffuse abdominal pain/distension and primary anastomosis here) has free air on AXR. What would you do?” Be prepared to (2) Left hemicolectomy/sigmoidectomy with see an x-ray here. colostomy and mucus fistula/Hartman’s pouch for lesions obstructing distal transverse How to Answer? colon/left colon/or sigmoid (3) Subtotal colectomy with primary anastomosis Have a DDx in Your Mind and Work for obstructing lesion in left/sigmoid with perfo- ration of cecum, useful in pts with metachron- Through cous lesions found after previous resection for Obstructing cancer pts with synchronous cancers. Not good option Diverticular/ischemic stricture in unstable pt given time involved Volvulus (4) Right hemicolectomy/ileostomy/mucus fistula Pseudo-obstruction (Ogilvie’s) for unstable pt with obstructing sigmoid/rectal Don’t forget about hx of prior operations lesion with perforation of cecum and gross con- tamination! Pt will then need work-up for malig- nancy and second operation to remove disease if Physical Exam survives (careful how close together you put Examine abdomen stomas unless you want a situation where the Rectal exam, heme occult test appliance will never seal properly!) Rigid sigmoidoscopy (unless true peritoneal signs, will (5) Defunctioning stoma (transverse loop colostomy) have therapeutic value if volvulus) and then a later operation to remove obstructing tumor/mass in descending colon/sigmoid

25 Part 1.qxd 10/19/05 2:51 AM Page 26

26 Colon and Small Bowel—Large Bowel Obstruction

Common Curveballs Strikeouts

Signs of peritonitis Doing anastomosis in face of frank contamination Perforation of right colon with mass on left Not knowing how to contruct ileostomy/mucus Rigid sigmoidoscopy won’t find cause for obstruction fistula/Hartman pouch Pt will be unstable intra-op Talking about on-table bowel lavage Pt will have AAA Performing long operation in elderly/unstable pt Pt will develop post-op abscess or abdominal Using Barium enema rather than water soluble contrast compartment syndrome when concerned about cause of obstruction and pos- Pt will develop ischemia at colostomy site sible perforation Pt will become coagulopathic during operation Discussing colonoscopically placed stents for malignant Inability to pass rectal tube for volvulus or keep in place obstruction (an option, but risk of perforation high to give pt bowel prep and don’t mention unless really familiar with this Cecum will get over-distended in follow-up of pseudo- modality) obstruction or will perforate Waiting for CT scan on pt with peritonitis Distal cancer will be fixed to pelvic structures Discussing cecostomy tubes or IV neostigmine Ureter/duodenal/liver injury while mobilizing right colon Splenic injury will mobilizing splenic flexure Hard peripheral liver lesion identified at time of emergency operation for peritonitis Pt will have had prior abdominal/colonic surgery Entering into a discussion of various stoma complica- tions (stenosis, hernia, retraction, prolapse) Part 1.qxd 10/19/05 2:51 AM Page 27

Colon and Small Bowel—Lower GI Bleeding (LGIB)

Concept Prior bleeding episodes Trauma Broad DDx but three common pathologies need to be Radiation (ischemia) ruled out: diverticulosis, angiodysplasia, and cancer. Pain with bleeding episode Likely to be self-limited in over 80% pts. AVM’s likely to Amount of bleeding, color rebleed, but less likely in diverticular disease unless young Dizziness or other evidence of shock age. ~10% pts will come to surgery. Physical Exam (brief and targeted in pt. with Way Question May be Asked? shock) Vital signs (r/o shock) “ 69 y/o female seen in the ED for dizziness after abruptly Signs of liver disease moving her bowels for a large amount of maroon colored Examine abdomen (prior scars) stools. She is tachycardic to 110s, but her BP is stable. What Rectal exam do you want to do?” May be given a pt that is stable or unsta- ble and make that determination in your mind early includ- ing if/when you plan to transfuse the pt. The scenario is likely Necessary Parts of Early Algorithm to be pretty basic as the examiners want to get at your man- Placing NGT and getting bilious aspirate agement algorithm and your indications for surgery. Anoscopy Rigid sigmoidoscopy (r/o rectal source) Assess stability of patient (never at fault for putting in How to Answer? ICU) Have a DDx Diverticulosis (painless bleeding) Diagnostic Tests Angiodysplasia (painless bleeding) Full laboratory panel including PT/PTT Cancer Bleeding time if pt on aspirin Ischemia Tagged RBC Scan (detects bleeding > 0.5 cc/min) IBD Angiography (detects bleeding > 2 cc/min and can be Infectious SMA injected first, then IMA followed by celiac Anorectal pathology trunk if first two negative therapeutic) Small bowel pathology (tumor, Meckel’s diverticulum) Colonoscopy (useful after prep in those pts who don’t Don’t forget about brisk bleeding from UGI source require urgent operation)

History Surgical Treatment Age (<30, consider IBD or Meckel’s otherwise divertic- (1) Know when to go to OR: ulosis dx/AVM most common) transfusion of > 4 U pRBC in 24 h Previous surgery (especially AAA) LGIB that causes hypotension Medications/known coagulopathy (ASA, coumadin use) LGIB refractory to maximal medical therapy 27 Part 1.qxd 10/19/05 2:51 AM Page 28

28 Colon and Small Bowel—Lower GI Bleeding

Continuous bleeding and can’t identify a source Colonoscopy (2) After tagged RBC scan And the advantages/disadvantages of each (a) If +, proceed to OR if unstable, angiogram if Pt will become unstable pre-op or intra-op stable Pt will need several transfusions (what is your limit for (b) If −, can still repeat within 24 hr and can prep going to OR?) for colonoscopy Won’t be able to identify a source pre-op or intra-op (3) After angiogram Ureter/duodenal/liver injury while mobilizing right colon (a) If +, vasopressin 0.2 U/min to control bleeding, Splenic injury will mobilizing splenic flexure no embolization of colonic pathology! Hard peripheral liver lesion identified at time of (b) If −, prep for colonoscopy emergency operation SMA injected first, then IMA followed by Pt will have had prior abdominal/colonic surgery celiac trunk if first two negative Pt will have had recent MI or severe cardiac disease (4) If can identify source Pt will have UGI source (change scenario) (a) On right side, resection + primary anastomosis Pt will take aspirin/Plavix/coumadin (b) On left side, resection with colostomy and Pt will have bleeding from stoma after subtotal colectomy mucous fistula/Hartmann’s pouch Pt will be Jehovah’s witness and won’t accept blood (5) If can’t identify source transfusions (a) Subtotal colectomy with primary ileorectal anastomosis or ileostomy depending on pts sta- bility Strikeouts (b) If see blood in mid-ileum or above, consider small bowel source prior to subtotal colectomy Embolizing colonic lesion identified by arteriogram Not placing NGT (failing to consider UGI source) Not performing rigid sigmoidoscopy Common Curveballs Not ruling out/correcting coagulopathy (coumadin, ASA, liver disease) Entire colon filled with blood on colonoscopy Performing long operation in elderly/unstable pt Will be blood in terminal ileum Not performing subtotal colectomy when can’t identify Colonic infarction after attempt at angiographic source embolization of diverticular bleed Sending unstable pt for bleeding scan Will have to operate on pt before RBC scan or Not considering angiogram or bleeding scan but pro- angiogram ceeding straight to surgery Being asked when it is appropriate to perform: Performing segmental ulectomics (very high rebleed Tagged RBC scan rate post-op) Angiography Part 1.qxd 10/19/05 2:51 AM Page 29

Colon and Small Bowel—Perirectal Abscess

Concept Data Constant pain in the rectal area. Arises from an infected anal Labs (elevated WBC) gland. After drainage, about half will develop an anal fistula. CT scan of pelvis (may predict level of the abscess)

Way Question May be Asked? Types of Abscesses I Perianal—abscess in SQ tissue adjacent to the anal “50 y/o male presents the ED with the complaint of hemor- verge rhoids. It started about 3 days ago, and the patient complains II Ischiorectal—the infection travels through the of severe pain, constant, and low grade fever and chills.” sphincters into the ischiorectal space, may be mini- mal external signs, usually fluctuance a few cm from anal verge How to Answer? III Intersphincteric—fluctuance/tenderness on rectal exam, abscess between internal and external Targetel full H+P, as always sphincters IV Horseshoe—bilateral ischiorectal spaces involved History and deep posterior anal space V Supralevator—from upward extension of Constipation intersphincteric or ischiorectal abscess, or from Pain just with defecation (fissure) or constant (abscess) downward extension of pelvic process Bleeding (diverticulitis, appendicitis, Crohn’s) Topical therapy History rectal complaints (incontinence, etc.) Surgical Treatment Family history IBD Fever/Chills Immediate incision and drainage Prior rectal surgery For I, II treatment is I + D under regional or general HIV status anesthesia Trauma For III, treatment is drainage into the Abd Pain For IV, drainage of deep postanal space by dividing overlying internal sphincter and lower portion of Physical Exam external sphincter and two counter incisions to drain ischiorectal extensions Examine abdomen For V, determine source first, if pelvic pathology, then Rectal exam (pt in left lateral decub position) external drainage of abscess, if rectal source, then Anoscopy/Sigmoidoscopy not necessary drainage with mushroom catheter May need to do exam under anesthesia Avoid fistulotomy at time of abscess drainage Look for malignancy, fistula, or other rectal pathology Low threshold to return to OR if no improvement (careful for scenario switch) (especially in supralevator abscesses)

29 Part 1.qxd 10/19/05 2:51 AM Page 30

30 Colon and Small Bowel—Perirectal Abscess

May need diverting colostomy for severe/recurrent Common Curveballs supralevator abscess or in pt with IBD If scenario continues to management of fistula, surgery is Pt will have anal/rectal cancer always the answer: Pt will develop post-op anal fistula Make sure to r/o associated GI diseases (IBD, HIV) Pt will have HIV or IBD Know pt’s baseline continence prior to going to OR Pt will have postoperative incontinence Evaluate entire colon with BE or colonoscopy Pt will have history of portal HTN or be on blood thinners Make need to inject H2O2 or methylene blue to identify fistula openings Pt will have inflammatory bowel disease Fistulotomy unless fistula involves > 30% sphincter Pt will be pregnant → fibers or is anterior fistula in a female Won’t be able to perform rectal exam in office/ED ( Must assess the level at which the fistula traverses the exam under anesthesia) sphincters Pt will have severe abscess that doesn’t improve despite Treat intestinal disease in IBD and this will often drainage (may need diverting colostomy) accompany resolution of perianal disease or use Seton as a drain Liberal use of the Seton (a nonabsorbable suture or Strikeouts rubber band placed through the tract that stimulates scar formation, gradually cuts through Not recognizing scenario and mistaking for throm- the sphincter mechanism as tightened over next bosed hemorrhoid several weeks, and minimizes post-op inconti- Performing fistulotomy during first treatment of anal nence) abscess Admitting pt, placing on IV abx, and “waiting” for Goodsall’s Rule abscess to mature/reach the surface when the external line lies anterior to the transverse Not recognizing associated GI diseases anal line, the track runs in a direct radial line to Not ruling out pelvic pathology with supralevator the internal opening in the anal canal abscess when the external opening is posterior to the Not making counter incisions with horseshoe abscess transverse anal line, the track curves backward to Not knowing Goodsall’s rule the posterior midline Not knowing how/when to use a Seton Part 1.qxd 10/19/05 2:51 AM Page 31

Colon and Small Bowel—Rectal Cancer

Concept Diagnostic Tests Prognosis worse than for colon cancer stage for stage. Full labs (especially LFTs, CEA) Preoperative therapy offered to most patients except those CXR (r/o mets) with disease limited to the mucosa. Examiners will likely Colonoscopy (to evaluate rest of colon!) want to know when you will perform LAR, APR, and Endorectal U/S (depth of invasion and lymph node transanal excision. status) +/− CT scan (most would!) +/− Air contrast Barium Enema Way Question May be Asked? +/− MRI

“62 y/o female with a history of painless rectal bleeding Surgical Treatment presents to your office for evaluation and on digital rectal exam, has an ulcerated mass beginning at 5 cm that Remember try to get 2 cm margins (most agree 0.5 cm appears fixed. What do you want to do?” May also be pre- at minimum) sented with patient that has obstruction, pain, or bright Always total mesorectal excision red blood per rectum. LAR easiest for lesions in upper third of rectum If trying for lesions > 5 cm from anal verge, use colonic J-pouch with coloanal anastomosis How to Answer? Use loop ileosotomy for any anastomosis constructed less than 5 cm from anal verge (reversed at 8–12 weeks History after gastrografin enema demonstrates no leak) Risk factors Involved organs (uterus, adnexa, posterior vaginal wall, Pain bladder) removed en bloc Change in bowel habits APR should be strongly considered for all invasive rec- Rectal mass tal CA < 5 cm from anal verge (closure of pelvic Previous colorectal surgery peritoneum not necessary, consider posterior Continence status (don’t want to do restorative vaginectomy in women with anterior tumors, have pt resection in pt with high likelihood of post-op incon- marked by stomal therapist pre-op) tinence) Poor risk pts can be treated by radiation, and re-evalu- ation +/− fulgeration Physical Exam Digital rectal exam (is it fixed, where is the lesion from Who gets Transrectal/Transanal Excision? anal verge, size/circumference involved) Lymphadenopathy (FNA any groin nodes if enlarged) (must be mobile, non-ulcerated lesion) Rigid Sigmoidoscopy (mobility of tumor and height < 3 cm in diameter from anal verge) T1 lesions (muscularis propria too deep!) within 8–10 cm of anal verge

31 Part 1.qxd 10/19/05 2:51 AM Page 32

32 Colon and Small Bowel—Rectal Cancer

Well to moderately differentiated tumor without ulcer- Being asked to describe surgical technique for an APR ation Local excision will come back with final pathology No lymphatic invasion on pathology Stage II depth of invasion Less than 1/3 rectal circumference Pt will have had prior colon surgery Performed in prone-jackknife position and excise Pt will have IBD tumor with full thickness of rectal wall into perirec- Pt will have local recurrence (after local transanal exci- tal fat with 1 cm circumferential margin (make sure sion → APR) to check path: r/o invasion of muscularis, lympho- Positive margins after local transanal excision (re- vascular invasion, or poorly differentiated tumor) excise!) Orient carefully for pathologist Injury to the ureter during LAR Post-op sexual dysfunction Presacral hemorrhage Who Gets Chemo/XRT? Stomal complications with APR (stenosis, retraction, (5-FU and Levamisole) hernia, prolapse) Invasion bladder/prostate/vagina Rectal CA beyond the mucosa (Stage B, C, D) Try to give XRT pre-op to spare small bowel (tattoo pre-op as tumor may “melt” away Strikeouts (If local excision comes back stage II depth of invasion, post-op XRT to primary site and pelvis) Performing a local resection when not indicated Treating anal cancer like rectal cancer Not doing staging work-up pre-op Common Curveballs Not evaluating rest of the colon pre-op Discussing transanal endoscopic microsurgery or endo- Testing your indications for local resection cavitary radiation Testing your knowledge of pre-op adjuvant therapy Attempting laparoscopically (current data support Post-op anastomotic leak colon CA approached laparoscopically, but not rec- Post-op pelvic abscess tal CA) Part 1.qxd 10/19/05 2:51 AM Page 33

Colon and Small Bowel—RLQ Pain

Concept Abdominal series EKG/CXR (depending on pt age) Wide variety of pathologies can contribute to RLQ pain. CT scan (in equivocal cases) Much can be gathered by H+P. Plan out a DDx in your U/S—transvaginal helpful in female to r/o gynecologic head and ask appropriate questions. In OR, be prepared process for what to do if the appendix is negative. It would be acceptable if unsure to admit pt overnight for observation (no antipyretics or antibiotics!) Way Question May be Asked? Surgical Treatment “21 y/o female evaluated in the ED for RLQ pain. Her (1) Appendicitis temperature is elevated and she has peritoneal signs. You Describe typical resection explore the pt through a RLQ transverse incision and find If base necrotic, partial cecectomy a normal appendix. What do you want to do?” Will likely If abscess, CT guided drain followed by interval appen- be placed in the position of taking patient to the OR and dectomy finding a normal appendix and asked what to do next. If comes back carcinoid, right hemicolectomy for→ carcinoid > 1.5 cm located at base of appendix How to Answer? serosal involvement History + lymph nodes Character of pain (2) Ectopic pregnancy GI/GU symptoms Unruptured→salpingotomy, evacuate contents, Previous surgery repair Appetite Ruptured→salpingectomy (preserve ovary) Menstrual history (if female) (3) TOA→ FHx IBD (so no confusion in future) Lavage, drain Physical Exam Salpingo-oopherectomy if necrotic Can treat with antibiotics (Ceftriaxone and Abdominal exam (tenderness, guarding, rebound, mass Doxycycline) if only PID (pulsatile?)) Rectal exam (4) Meckel’s Pelvic exam (if female)—don’t trust someone else’s exam! If negative appy, make sure to examine last 2 feet of Look for hernia terminal ileum Wedge resection of diverticulum, may need segmen- Data tal resection with primary anastomosis depending on inflammation Full lab panel (including amylase and pregnancy test) Always do appendectomy before closing! U/A 33 Part 1.qxd 10/19/05 2:51 AM Page 34

34 Colon and Small Bowel—RLQ Pain

If incidental finding, remove if pt < 18 years of age Other causes not listed above: or a narrow neck to diverticulum Giadiasis Renal stone (5) Terminal ileitis Diverticulitis (right or left sided +/− abscess) Do appendectomy if base of appendix is free of dis- Leaking AAA (take to OR immediately) ease Acute mesenteric ischemia Treat medically with Azulfidine, Prednisone, Flagyl Incarcerated hernia Surgery only for obstruction, bleeding, perfora- Testicular torsion/ovarian torsion tion, non-healing fistulas, failure of medical Ruptured ovarian cyst management Pt will be pregnant (appendix may not be in pelvic (6) Solid ovarian mass depending on trimester) (a) Postmenopausal—resect with full staging for Pt will be HIV + (CMV enteritis, TB, lymphoma) ovarian cancer (washings, biopsies, omentectomy, Mesenteric lymphadenitis para-aortic LN sampling, TAH/BSO) There will be no problem in RLQ except bile staining (b) Premenopausal—washings, biopsies, frozen and mass in RUQ (perf duodenal ulcer!—changing section after incisional biopsy, if malignant, scenario) unilateral salpingo-oophorectomy (7) Cystic ovarian mass Strikeouts (a) Postmenopausal—ovarian cancer staging procedure Describing complicated laparoscopic procedures (b) Premenopausal—treat as “6b” above if > 5 cm, Not looking for Meckel’s or into pelvis when appendix otherwise follow with U/S and refer to Gyn for normal follow-up Not knowing what to do for Carcinoid or Crohn’s disease Fumbling with the change in scenarios (can happen Common Curveballs anytime) Forgetting pregnancy test in females of child-bearing age Forgetting pelvic exam in females Any one of a variety of diagnoses, none of which are Not doing rectal exam appendicitis Getting CT scan showing appendicitis and discussing Be prepared for scenario to switch right after you admission, Abx, and interval appendectomy describe how to deal with one problem (after answer- Not having broad DDx ing for Meckel’s, expect an examiner to ask a ques- tion like “OK, what if the terminal ileum is inflamed?”) Changing scenarios is common here Inflammatory mass RLQ and can’t identify appendix Part 1.qxd 10/19/05 2:51 AM Page 35

Colon and Small Bowel—Ulcerative Colitis

Concept Abdominal series (colon dilatation, free air) +/− CT scan Inflammatory bowel disease of unknown etiology. Affects Stool for C.diff, O+P, enteric pathogens if unclear the mucosa of the rectum and colon. Doesn’t have skip etiology areas or full thickness involvement like Crohn’s. Surgery performed for intractable disease or toxic megacolon. Surgical Treatment Way Question May be Asked? (1) If suspect toxic megacolon (a) ICU/IVF/transfusion if necessary (b) Antibiotics “25 y/o male presents to the ED with abdominal pain and (c) NGT/NPO/bowel rest bloody diarrhea. Physical exam reveals a temperature of (d) TPN 101, moderate abdominal tenderness, and moderate dis- (e) Steroids tension. What do you want to do?” Presentation will usu- (f) Cyclosporine ally include some form of diarrhea, abdominal pain, and (g) Serial labs/x-rays/exams fever. Rarely arthritis, uveitis, and pyoderma. Make sure to (h) Failure to improve within 48 h or worsening differentiate the patient with ulcerative colitis flare from exam→OR for subtotal colectomy and Brooke the patient with toxic megacolon! ileostomy (can bring up mucous fistula to lower portion of wound and not open—will open in How to Answer? ~1/3 pts but less risky than rectal staple line leak) (i) In unstable pt, can perform Turnbull History procedure→ diverting ileostomy and blowhole Family history loop colostomy Systemic manifestations (arthritis, uveitis, and pyoderma) Medications (steroids?) (2) If responds to medical treatment, or less acute pres- Previous flares entation (a) Barium enema to evaluate extent of disease Physical Exam (b) Colonoscopy and multiple biopsies to evaluate for dysplasia Vital signs (fever, sepsis) (c) UGI with SBFT (r/o Crohn’s) if any doubt Abdominal exam (peritonitis?) (d) Medical treatment with: Rectal exam (will always be involved in UC) azulfidine prednisone Diagnostic Tests steroid enemas 6-MP Full labs lomotil Sigmoidoscopy low residue diet

35 Part 1.qxd 10/19/05 2:51 AM Page 36

36 Colon and Small Bowel—Ulcerative Colitis

(e) Surgery for: You’ll be asked your medical regimen for the chronic UC unresponsive to medical therapy (uncon- form of the disease trolled diarrhea, failure to thrive in children) Scenario will change from toxic megacolon to chronic Dysplasia or cancer on colonoscopic biopsy form of UC Severe extracolonic disease Asked differences between UC and Crohn’s (f) Surgical options (choice depends on severity of Asked to describe extracolonic manifestations rectal involvement) Staple line on Hartmann’s pouch will leak Total proctocolectomy with Brooke ileostomy Pt with perirectal abscess/fistula will have Crohn’s dis- (if severe) ease—how will you manage? Total colectomy, anorectal mucosectomy and ileorectal pull through anastomosis (use diverting ileostomy here) Strikeouts

Not making the diagnosis of UC Common Curveballs Not ruling out infectious diarrhea or C. diff and taking out the entire colon prematurely Will present as massive bleeding Not performing sigmoidoscopy Will not respond to medical treatment Not attempting to treat toxic megacolon with steroids, Pt will be unstable antibiotics, serial exams but going straight to OR There will be free perforation Not differentiating from Crohn’s Pt will have post-op intraabdominal abscess Not knowing difference between UC and Crohn’s Pt will have leak after ileoanal anastomosis Part 1.qxd 10/19/05 2:51 AM Page 37

Endocrine—Carcinoid

Concept Duodenum—treat like rectum with local excision unless tumor is > 2 cm, involves muscular coat, or cannot Malignant neuroendocrine tumor. Usually asymptomatic be adequately excised (Whipple resection then unless outside of GI tract: bronchus, rectum, mets to liver appropriate) so that hormones elaborated can bypass the portal system. Debulking surgery appropriate with liver mets and Symptoms of flushing and diarrhea from excess blood lymph node involvement serotonin level. Multiple wedge resections or lobectomy appropriate for liver mets Way Question May be Asked? Selective embolization also a treatment for liver mets For symptomatic carcinoid tumors, somatostatin is “31 y/o male undergoes a laparoscopic appendectomy for drug of choice. acute appendicitis and the pathology comes back with a 2.1 cm carcinoid at the base of the appendix. What do you do?” Chemo streptozotocin in advanced cases with little help Remember adequate pre-op work-up as right sided How to Answer? valvular fibrosis occurs in late disease Measurement 5’HIAA in 24 h urine Treatment depends on three factors: Octreotide scan to localize neuroendocrine tumors (1) Size (2 cm is the key—some text say 1.5 cm) Carcinoid crisis may occur shortly after inducing anes- (2) Site (appendix, rectum, duodenum) thesia with: (3) Pathology (depth of invasion) Cardiac arrhythmias Appendix—appendectomy okay unless Labile blood pressure (1) More than 2 cm Generalized flushing (2) Involves base of the appendix Treatment with octreotide (3) Involves appendiceal fat Multiple synchronous tumors in 1/3 pts so full ex lap (4) Involves lymph nodes Many pts with small intestinal carcinoid present as SBO Then right hemicolectomy is appropriate post-op treat- in pt without other risk factors ment. In carcinoid syndrome: Rectum—local excision okay unless Flushing may be brought on by emotional stress or (1) Greater than 2 cm meals (2) Invasion of muscular coat Diarrhea unrelated to flushing (3) Local recurrence (4) Fix to surrounding tissue Common Curveballs Then, APR is appropriate post-op treatment. Small bowel—wide local excision with mesenteric Carcinoid will be less than 2 cm but invade appendiceal lymph nodes fat

37 Part 1.qxd 10/19/05 2:51 AM Page 38

38 Endocrine—Carcinoid

Pt will ask for other options besides right hemicolec- Forgetting adequate pre-op w/u in pt with carcinoid tomy for 1 cm carcinoid at appendiceal base syndrome—pt will have tricuspid or pulmonic valvu- Pt will have carcinoid syndrome with carcinoid tumor lar disease you can’t locate Forgetting the characteristics that determine surgical Pt will present with episodic flushing and diarrhea treatment of carcinoid tumors Pt will have liver metastases and may require lobectomy Failing to perform appropriate cancer operation with to fully debulk resection of accompanying mesentery/lymph nodes Pt will have ampullary carcinoid requiring Whipple to Failing to perform full ex lap to r/o other carcinoid fully excise tumors in small bowel Pt will have carcinoid crisis intra-op Failing to recognize the carcinoid syndrome when Pt will present with SBO secondary to tumor, present path = small bowel carcinoid Asked about use of medication to treat carcinoid syndrome

Strikeouts

Forgetting full physical exam—pt will have rectal carcinoid Part 1.qxd 10/19/05 2:51 AM Page 39

Endocrine—Cushing’s Syndrome

Concept Diagnostic Tests The majority of Cushing’s syndrome cases are from an 24 h urine for cortisol (most cost-effective test if inci- ACTH secreting tumor of the pituitary gland. Other denteloma) possible etiologies include: ectopic ACTH producing Plasma cortisol level at 8 am and 8 pm (check for loss tumor, adrenal adenoma, adrenal carcinoma, and bilat- of diurnal variation) eral adrenal hyperplasia. The treatment of choice is sur- ACTH level, two possibilities: gical resection. (1) If elevated→pt has pituitary tumor or ectopic ACTH producing tumor (2) Low→ pt has adrenal pathology Way Question May be Asked? Dexamethasone suppression test, two possibilities (1) Suppress ACTH→ pituitary source “28 y/o female referred to your office for generalized (2) Doesn’t suppress ACTH→ ectopic cancer weakness, new onset diabetes and hypertension. On Then obtain CT scan: physical exam, she is obese and has a buffalo hump and Of head for pituitary source moon facies. What do you want to do?” Might be given Of chest/abdomen/pelvis for ectopic cancer source that the patient is referred to you with the diagnosis of Of abdomen for adrenal source Cushing’s syndrome. Examiners will not want to waste Should see contralateral gland to be atrophied time in the history and physical stage, for the most Should not see bilateral enlargement part—but still must know just in case. Most will want to If > 5 cm, suspect adrenocortical carcinoma get at your algorithm for managing the patient (this goes for most scenarios). Surgical Treatment Posterior unilateral adrenalectomy unless suspect How to Answer? malignancy For adrenocortical carcincoma, resection includes adre- History nal, kidney, and continuous structures (spleen, distal Steroid use pancreas, diaphragm) History of cancer (ACTH producing tumor of lung) If metastatic disease present, debulk Diabetes Can use mitotane if metastatic disease or pt not a Hypertension surgical candidate (adrenolytic agent destroys Zona Generalized weakness Fasiculata)

Physical Exam Common Curveballs Buffalo hump Truncal obesity Scenario will change with first presentation as pituitary Striae tumor, then presentation as adrenal tumor Moon facies Will be malignant tumor

39 Part 1.qxd 10/19/05 2:51 AM Page 40

40 Endocrine—Cushing’s Syndrome

Addisonian crisis post-op Strikeouts Being asked difference between Cushing’s syndrome and Cushing’s disease Not being able to diagnose location of tumor There will be an ACTH or CRF secreting tumor (typi- Not knowing treatment for pituitary tumor cally lung) Not knowing treatment for adrenal tumor Pt won’t be a surgical candidate Performing FNA on the adrenal tumor Will be given results of tests you order (24 h urine cor- Not recognizing the adrenal tumor for what it is and tisol, plasma cortisol levels, ACTH levels, dexam- directing therapy towards a pituitary lesion ethasone suppression test) Discussing bilateal laparoscopic adrenalectomy Being asked when to order the above tests Will present as an incidenteloma Describing surgical approach to adrenalectomy Part 1.qxd 10/19/05 2:51 AM Page 41

Endocrine—Hyperthyroidism

Concept Hyperfunctioning adenoma Malignancy Multiple etiologies, surgery only for very specific indica- Subacute thyroiditis tions. Important to now how to make diagnosis, the vari- Factitious thyrotoxicosis (exogenous T4) ous treatment options, when to treat with surgery, and how Ovarian (struma ovarii thyroid tissue in ovarian ter- to treat hyperthyroid crisis. atoma), testicular, pituitary tumors (rarest)

Way Question May be Asked? Laboratory tests TSH “27 y/o female referred to your office by a family practi- Free T4 tioner with the recent diagnosis of hyperthyroidism. What LATS level (Grave’s disease!) do you want to do?” May have symptoms of hyperthy- Thyroid antibody (thyroiditis) roidism and you need to make diagnosis first: tachycardia, Thyroid Scan (r/o “hot” nodule) heat intolerance, weight loss, fatigue, palpitations. U/S neck (r/o mass)

How to Answer? Management

Always a complete H+P (1) Medication (not for pts with toxic nodules) (a) PTU or Tapazole—problem is compliance History and complications of medications including agranulocytosis Anxiety (b) Can use PTU in pregnant pt Tremulousness (2) Radioactive iodine I 131 Weight loss (a) Good option in older pts Sweating (b) Single dose usually effective in Grave’s disease Heat intolerance and cause hypothyroidism in > 50% pts Palpitations (c) Repeat doses possible (3) Surgery Physical Exam (a) Lobectomy or subtotal thyroidectomy for toxic nodules Neck nodules (b) Subtotal thyroidectomy appropriate for: Exophthalmoses cosmesis pregnant pt in second trimester who fails PTU DDx failure of medical treatment after 1–2 years compressive symptoms Grave’s disease (most common) hyperthyroidism in children Toxic Multi-nodular goiter young women who want to become pregnant

41 Part 1.qxd 10/19/05 2:51 AM Page 42

42 Endocrine—Hyperthyroidism

thyrocardiac pts Common Curveballs pts with severe exophthalmos (c) Need to prepare pt for surgery: There will be a hot nodule PTU until surgery Will have post-op complication of: Beta blockers prn (if use, continue in post-op Laryngeal nerve injury period!) Hematoma Lugol’s solution (iodine) 2cc BID starting Hypothyroidism, hypoparathyroidism 10–14 days prior to surgery to decrease Injury to external branch of superior laryngeal nerve vascularity of thyroid gland Recurrent hyperthyroidism Continue beta blocker post-op for 8–10 days Pt will be pregnant (t 1/2 of hormone) Pt will fail medical therapy (d) Thyroxin for life post-op (can’t tell true thyroid Asked to describe subtotal thryoidectomy (leave 3–5 gm status until 1–2 years post-op tissue behind) Ask how to prepare pt prior to surgery Thyroid Storm—Life-Threatening Pt will have nodule that will be a malignancy (changing scenarios) on U/S, FNA, or final pathology Initiated by physiologic stresses (surgery, anesthesia, Pt will develop thyroid storm MI, infection, childbirth) Presentation of: fever, tachycardia, abdominal symp- toms, change mental status Strikeouts Mortality ~10% Treatment: Not making correct diagnosis IVF Not knowing how to treat hyperthyroidism Sedatives Not knowing indications for surgery O2 Not knowing how to treat/recognize thyroid storm Cooling blankets Not ruling out adenoma/malignancy PTU 250 mg q 4 h Not checking LATS/thyroid U/S, T4/TSH Hydrocortisone 100 q 6 h Not being comfortable with discussion of complica- Beta blockers (may need IV inderal for control tions of thyroidectomy cardiac arrhythmias) Not knowing when to do partial v. total thyroidectomy May need intubation Treat precipitating cause! Part 1.qxd 10/19/05 2:51 AM Page 43

Endocrine—Insulinoma

Concept Be sure to ask about family history for MEN 1 Early pregnancy could be mistaken for this syndrome Tumor in the pancreas that releases insulin. Association with MEN I syndrome (pituitary, pancreas, parathyroid) Laboratory Studies so be sure to ask about family history. Usually less than 2 cm in size, if > 3 cm, be suspicious of malignancy. Need to have a fasting glucose level (should be less than Management depends on tumor location, number of 60) tumors, malignancy, and part of MEN syndrome (hyper- Need to check fasting insulin level (should be greater parathyroidism, pituitary/pancreatic tumors). Need to r/o than 24) other causes for hypoglycemia like liver disorder (cirrhosis, Check insulin to glucose ratio (should be > 0.3) Gaucher’s dx), pregnancy, and exogenous administration. Check C-terminal peptide level to rule out exogenous insulin administration (will be elevated only with endogenous insulin) may see needle marks on Way Question May be Asked? arms/legs Could also check proinsulin level “37 y/o female presents with a history of repeated bouts of Will need to try to localize insulinoma (80% in pan- weakness and fatigue after meals, with a fasting glucose creas, may be multiple if familial variety, only 10% level of 40.” There are several ways the question can go, malignant) but, after ruling out liver disorder, and alcoholism Localization studies (don’t stop after CT scan!): (quickly), start focusing on insulinoma. Rarely will get (1) CT scan abd/pelvis with thin cuts through classic Whipple’s triad: pancreas symptoms with fasting (2) Arteriogram blood glucose < 60 at time of symptoms (3) Portosplenic vein sampling (be prepared for symptoms relieved with glucose administration results of this test!) (4) Endoscopic U/S (5) MRI How to Answer? Even if can’t localize (usually won’t be able to), you start Will need to perform complete H&P the pt on Diazoxide and prepare pt for surgery (be sure okay surgical candidate) History Surgical Treatment Syncope In OR, you need to fully examine pancreas by division gas- Blurred vision trohepatic ligament, Kocher maneuver, medial reflection Sweating → brought on by fasting or exercise of spleen, and divide the peritoneum on superior and infe- Palpitations rior borders of pancreas. Weakness Seizures If perform enucleation, dose of secretin intra-op to check Confusion for leak, place omental flap, and leave a drain. Don’t try to Wt. gain close defect created by “bovie down” to lesion. 43 Part 1.qxd 10/19/05 2:51 AM Page 44

44 Endocrine—Insulinoma

Then you will be in one of the following situations: Will do distal and pt still symptomatic. (1) You find tumor in the head of Examiner will ask you if you regret your decision pancreas→enucleation distal pancreas→distal (stick to your guns if you know you gave the right pancreatectomy answer—examiner most likely trying to get at how (2) You find tumor in pancreas with mets→ debulk- confident a surgeon you are) ing surgery, and use somatostatin, diazoxide, and streptozotocin post-op (3) You can’t find tumor→ intra-op U/S, rapid Strikeouts venous assays (a) If still can’t find, try to send to outside facil- Whipple procedure for tumor near surface (appropriate ity for rapid venous assays to detect drop in if deep in pancreatic head) insulin level Stopping after CT scan and proceeding straight to OR (b) If examiner won’t let you do→do distal pan- Failing to ask about family history suspicious for MEN createctomy and send for frozen section and (pituitary, pancreas, parathyroid problems) glucose/insulin measurement Failing to rule out exogenous insulin administration (c) Don’t do near total pancreatectomy unless Performing too radical a surgery before exhaustive an endocrine expert work-up including venous sampling and possible (4) MEN 1→ subtotal pancreatectomy because of referral to center specializing in the disease (always high incidence of islet cell hyperplasia better to refer pt to tertiary care center than doing blind near total pancreatectomy) Pt may have mild hyperglycemia for 2–3 days post-op. Not mentioning controlling symptoms pre-op with small, frequent meals or diazoxide pre-op Common Curveballs (suppresses insulin secretion with side effect of fluid retention and nausea) Failing to rule out liver disease Won’t be able to localize pre-op Failing to recognize insulin-producing tumor in pt with Won’t be able to localize intra-op (consider intra-op hypoglycemic symptoms reversible with sugar intake U/S) Not knowing Whipple’s triad Won’t have facilities to do rapid venous sampling Performing pancreatic resection when tumor near sur- Will find a mass and FNA will be a malignant face rather than enucleation adenocarcinoma Will be pancreatic duct leak after enucleation Will be multiple tumors It’s better to say “I don’t perform this procedure, but Will see pancreatic duct leak after enucleation the key steps are...” than to describe a procedure Will be exogenous administration if don’t check you don’t do and then get trapped answering C-terminal peptide/proinsulin level questions about technical steps in the procedure. Will be part of MEN syndrome Describing the exploration and enucleation to be done Will be malignant tumor or borderline tumor on final laparoscopically path Mistaking surgery for gastrinoma for insulinoma and Can’t enucleate because deep in head/tail of pancreas (if opening up duodenum and palpating for tumor you do, there will be damage to pancreatic duct Will get into massive hemorrhage during enucleation Part 1.qxd 10/19/05 2:51 AM Page 45

Endocrine—Pheochromocytoma

Concept Localization Studies Tumor of adrenal medulla producing excess catecholamines. (1) CT scan abd/pelvis 10% extraadrenal, pediatric, malignant, bilateral, familial Look at adrenals (MEN2a or 2b). Look for extraadrenal tumors Look for metastases (2) I-131 MIBG scan Way Question May be Asked? (3) Portosplenic vein sampling (be prepared for results of this test or to describe how it’s done!) “24 y/o female sent to you for evaluation of her frequent (4) MRI headaches, palpitations, and BP of 190/110. What do you want to do?” May be given a history including flushing, Surgical Treatment sweating, episodic attacks, or young patient with new onset hypertension. Need to adequately prepare pt preoperatively (1) Start alpha blocker two weeks prior to surgery, phe- noxybenzamine 20 mg bid and increase by 20 How to Answer? mg/day until BP and symptoms controlled (2) Add Beta blocker 3 days prior to surgery, inderal 10 History mg tid Frequent “attacks” (3) IVF hydration starting 2 days prior to surgery (typ- Anxiety ically pts are volume contracted) Sweating/flushing In OR, have rapid acting agents ready: Headaches Neosynephrine Palpitations Lidocaine Family history (MENIIa—hyperparathyroid, pheo, Propranolol medullary thyroid CA) Phentolamine (alpha-blocker) Other neuroectodermal diseases (von Hippel-Lindau, tuberous sclerosis, neurofibromatosis) Have CVP (or SGC) and A-line

Do not use MSO4/Demerol (stimulates catechol release), Physical Exam or atropine (increases tachycardia) Blood pressure Be prepared to describe right and left adrenalectomy Gentle abdominal exam (don’t want to compress organ and anatomy of adrenal vein on both sides! of Zuckerkandl) Palpate thyroid Surgery

Data Midline incision Full exploration of both adrenal glands, aortic bifurca- 24 h urine for VMA, metanephrine, normetanephrine tion, bladder, kidney hilum (make sure not on MAO inhibitor) Control venous drainage first! Ca++ and calcitonin levels (r/o MENIIa) Excise tumor with minimal manipulation 45 Part 1.qxd 10/19/05 2:51 AM Page 46

46 Endocrine—Pheochromocytoma

Debulk malignant tumors to help reduce symptoms Will be malignant tumor Bilateral adrenalectomy for bilateral disease, MENII Will have recurrent symptoms post-op Will be part of MENII syndrome (which operation is After tumor resected, give 1 mg glucagons to check performed first?) for occult tumor (tachycardia/inc. BP signs of resid- Pt will have fluctuations of BP and HR intra-op ual tumor).

Follow-Up Strikeouts Urinary studies every 3 months, then yearly Failing to ask about family history Screen all family members yearly for pheo, medullary Not knowing pre-op work-up or pre-op preparation thyroid cancer, hyperparathyroidism of pt Describing laparoscopic operation in pt with evidence Common Curveballs of metastases, prior operations, or large tumor (> 8 cm) Not ligating adrenal vein early in your description of Won’t be able to localize pre-op operation Won’t be able to localize intra-op (consider intra-op Not placing invasive hemodynamic devices intra-op U/S) Not screening relatives (for RET proto-oncogene) if Won’t have facilities to do rapid venous sampling suspect MENII syndrome Will be multiple tumors Part 1.qxd 10/19/05 2:51 AM Page 47

Endocrine—Primary Aldosteronism

Concept To differentiate bilateral hyperplasia for adenoma, use Captopril test Either unilateral adenoma (85%) or bilateral adrenal (1) Give dose of captopril and measure aldosterone hyperplasia (15%) resulting in elevated aldosterone levels, level before and after, if decreases, then bilateral hypertension, and hypokalemia. hyperplasia Always try to localize tumor (these are typically small): (1) CT scan abd/pelvis Way Question May be Asked? (2) Selective venous sampling (especially if CT negative) “32 y/o female with two year history of hypertension, (3) NP-59 Iodocholesterol scan (helps rule out unresponsive to medical treatment.” May also have hyperplasia too) fatigue, muscle cramps, polyuria, weight gain, peripheral edema (often rings on the fingers not fitting is first thing patient notices). Surgical Treatment If bilateral hyperplasia, treatment is medical with Spironolactone (effective in 90% cases) How to Answer? If single adenoma, unilateral adrenalectomy is indi- cated. This could be Have to think about surgically correctable forms of hyper- (1) A posterior approach through the 12th rib on tension in young patient with new onset HTN (coarcta- right or 11th on left tion, renal artery stenosis, Cushing’s, pheo) (2) Through midline laparotomy (3) Laparoscopic Complete History and Physical Exam Describe whatever approach you are comfortable with, but remember Questions to rule out pheo (1) On right, the adrenal vein enters posteriorly into Family history (if suspicious of pheo) the IVC and mobilization of the right lobe of the Make sure pt not taking any medications (especially liver is necessary if going transabdominal diuretics—will throw off lab values) (2) On left, mobilization of the colon, and pancreas +/− spleen may be required and adrenal vein Diagnostic Tests empties into renal vein (3) Only the venous drainage is consistent in adrenal Check for low potassium anatomy Check aldosterone/renin ratio (should be > 400 or pri- mary hyperaldosteronism) Careful of post-op hypotension→may need to use If renin is high, suspect other etiology (renal artery steroids stenosis)

47 Part 1.qxd 10/19/05 2:51 AM Page 48

48 Endocrine—Primary Aldosteronism

Common Curveballs Not ruling out other forms of surgically correctable HTN Not knowing medical treatment for b/l hyperplasia and Pt will have b/l hyperplasia if you don’t rule it out performing bilateral adrenalectomies Pt will have no tumor by CT scan so know other local- Not knowing how to treat post-op hypotension and izing studies doing extensive work-up for hemorrhagic/hypov- Pt will have renal artery stenosis or fibromuscular dys- olemic/cardiogenic shock plasia if you don’t check renin levels Misdiagnosing pt as a pheochromocytoma Pt will have post-op hypotension (from adrenal insuffi- Not knowing mechanism of action of aldosterone or ciency) renin-angiotensin-aldosterone axis Will injure spleen when mobilizing colon for left Not knowing that syndrome is called “Conn’s adrenalectomy Syndrome” Will injure IVC doing right adrenalectomy

Strikeouts

Not checking potassium, aldosterone, renin levels Not being able to describe surgical approach (no matter what question, this brings you back next year—and if you can’t describe, don’t make it up) Part 1.qxd 10/19/05 2:51 AM Page 49

Endocrine—Neck Mass

Concept Hoarseness Dysphagia Usually benign in practice, but a neck mass won’t be History (HIV+, prior malignancy) benign during the exam. Differential diagnosis should be Systemic symptoms (“B symptoms” with lymphoma?) complete and your H+P should guide you towards the Previous head/neck surgery (suspicious mole/melanoma underlying process. “Rule of 80s” after age 40: removed? Was it overlying parotid gland?) 80% non-thyroid neck masses in adults are neoplastic 80% of neoplastic masses are malignant Physical Exam 80% of malignant masses are metastatic Location 80% malignancies in adults are squamous cell carcino- Tenderness mas Fixed/Mobile 80% of metastatic are from primaries above level of Movement with swallowing clavicle Pulsatile (caratoid body tumor) Be wary of a neck mass in an infant, in the midline, or Sinus (cyst) in an HIV+ patient (lymphoma). Nasopharynx Oral Cavity Larynx Way Question May be Asked? Neck (thyroid) Other lymph node basins (axillae, groin) “43 y/o male presents to the office with a mass in his left Skin neck. It is non-tender and has been there for about 3 Breast months. He has a significant smoking history. What do you Abdomen (palpable liver/spleen) want to do?” You will likely have to ask some more ques- Stool guiac (maybe metastatic) tions on your H+P regardless of the way the scenario is given to you. Look for clues as to where the mass is (may Diagnostic Tests be given “left anterior neck”) and the age/sex of the patient to help guide you. FNA (critical here and helpful in neck masses) CXR (lung or mediastinal pathology) CT Scan of the face/neck (sinuses/oral cavity/nasophar- How to Answer? ynx/larynx) +/− MRI History +/− U/S of neck (useful to evaluate thyroid/parathyroid) Age (very important here) +/− Thyroid scan (again, useful to evaluate Location (again, very important) thyroid/parathyroid) Duration Blood tests (as always, complete labs, CBC with dif- Drainage (brachial cyst?) ferential, in select cases, calcitonin/calcium levels, Pain thyroid hormones, and examination of blood Tobacco/Alcohol use smear)

49 Part 1.qxd 10/19/05 2:51 AM Page 50

50 Endocrine—Neck Mass

Differential Diagnosis CT scan of neck/chest/abdomen/pelvis Bilateral mammograms Midline EGD Thyroglossal duct cyst BE/Colonoscopy Dermoid cysts If primary found, this represents Stage 4 disease and Pyramidal lobe of thyroid chemotherapy may be offered Lateral If no primary found, excisional biopsy + modified Lymph node—infected vs. metastatic radical neck dissection on that side Brachial cleft cyst Send for ER/PR receptors and mucin stain (r/o Supraclavicular breast melanoma/lymphoma) Lymph node—infected vs. metastatic Lymph node = thyroid Submandibular/Preauricular mass Thyroid U/S Lymph node Thyroid Scan Parotid gland Total thyroidectomy + Salivary gland Enlarged nodes for papillary CA Don’t forget inflammatory etiologies: Central lymph node dissection and Modified Lymphadenitis radical for medullary Tuberculosis Tularemia Lymph node = lymphoma Cat scratch Excisional biopsy of node Toxoplasmosis CT scan neck/chest/abdomen/pelvis Sarcoidosis Bone marrow biopsy (Stage IV disease) Viral Stage disease (number of nodal groups/which side of diaphragm) Treatment Staging laparotomy? Chemotherapy (CHOP) Thyroglossal duct cyst Post-op XRT should be considered to neck after radical U/S neck to confirm presence of normal thyroid neck dissection Excision with middle portion of hyoid bone and Important anatomy to remember follow any tissue to base of tongue (Sistrunk Anterior triangle boundaries procedure) Lateral = SCM Brachial cleft cyst—always surgical excision, be careful Medial = midline of neck of anatomic pathway! Superior = inferior edge of mandible First Brachial Cleft Posterior triangle boundaries Opening at angle of mandible, passes through Inferior = clavicle facial nerve Anterior = SCM Second (most common) Posterior = trapezius Opening anterior border of SCM, passes between Steps in radical neck dissection: carotid bifurcation T-incision Third Locate and protect mandibular and cervical Opening at lower border of SCM, passes behind branches of facial nerves carotid Divide anterior facial vessels Lymph node = squamous cell carcinoma Remove contents of submental and submandibu- NPL in your office lar triangles Excisional biopsy under anesthesia + exam under Ligate external jugular vein close to subclavian anesthesia with: Protect spinal accessory, phrenic, brachial Panendoscopy of upper aerodigestive tract: plexus while removing fat/lymphatic tissue in Direct laryngoscopy posterior triangle Rigid esophagoscopy Low division of omohyoid behind SCM Rigid bronchoscopy Division of SCM Biopsies of nasopharynx, base of tongue, Open carotid sheath and ligate IJ close to clavicle pyriform sinus Ligate submaxillary duct Excision of primary site (if found) and modified In modified radical neck dissection, the following are radical neck dissection preserved: Lymph node = adenocarcinoma Part 1.qxd 10/19/05 2:51 AM Page 51

Strikeouts 51

Spinal accessory nerve Damage to any nerve (phrenic, spinal accessory, vagus, Internal jugular vein hypoglossal) SCM Strikeouts Common Curveballs Not knowing the different algorithms between FNA Metastatic thyroid cancer yielding squamous cell carcinoma vs. lymphoma vs. FNA will be indeterminate adenocarcinoma Scenario will switch several times from squamous cell Not having a broad DDx carcinoma to adenocarcinoma to lymphoma Not performing FNA Melanoma overlying parotid gland (modified radical Not knowing surgery for: neck + superficial parotidectomy) Thyroglossal duct cyst Won’t be able to identify primary site Most common branchial cleft cyst Will find primary site and be asked how to perform Not being able to describe modified neck dissection or resection difference from complete radical Seroma under skin flap Chylous fistula post-op in left neck dissection Carotid artery blowout post-op Part 1.qxd 10/19/05 2:51 AM Page 52

Endocrine—Hyperparathyroidism

Concept (1) Check Cl/PO4 level (greater than 30 suggestive of 1˚ HP) One of many possible causes of hypercalcemia. Must rule (2) Get PTH level

out common causes of hypercalcemia first, then remember (3) Low PO4 (high in renal failure) primary, secondary, and tertiary types of hyperparathy- (4) +/− x-ray of hands to look for cystic/resorptive roidism. changes (1) Primary hyperparathyroidism, about 80% ade- (5) 24 h urinary calcium for rare hypercalcemic hypocal- noma; about 2% double adenoma ciuria (2) Secondary hyperparathyroidism from hyperplasia Third, localization studies: often secondary to renal failure (1) CT scan or MRI of neck to mediastinum (3) Tertiary hyperparathyroidism from autonomous (2) Thallium-technetium scan functioning glands after etiology causing secondary (3) Ultrasound hyperparathyroidism has been treated (most com- (4) Sestamibi scan mon in renal transplant patients) (5) Invasive studies like arteriography and selective venous sampling for recurrences Way Question May be Asked? (6) Many believe best localization study is experienced parathyroid surgeon (but don’t go there on Oral Exam unless you are one!) “61 y/o female with an elevated calcium level on routine blood tests with her only complaint of fatigue. What do Indications for surgery are two-fold: symptomatic you want to do?” Sometimes you will be given more hypercalcemia, or information like renal stones, or abdominal pain, consti- calcium > 11 with signs of bone disease pation, arthralgia, myalgia, depression, ulcers, pancreati- tis, osteitis fibrosa cystica, but rarely all of the symptoms Procedure associated with elevated calcium (renal stones, bone pain, constipation, fatigue, ulcer, depression, emotional (1) If single adenoma, excise the adenoma, be careful lability). not to rupture capsule, and biopsy 1–2 other glands to confirm they are normal (2) If hyperplasia, total parathyroidectomy with auto- How to Answer? transplantation in to forearm (3) If parathyroid Ca (very high pre-op Ca levels), en First, need to be able to rule out common causes of hyper- bloc resection with thyroid lobe on that side and calcemia. Many mnemonics here, but all include: malig- regional lymph nodes nancy (breast, lung, prostate, multiple myeloma), primary (4) Gold standard is to explore all 4 glands (most tradi- and secondary hyperparathyroidism, sarcoidosis, thi- tional answer) azides, immobilization, familial hypocalciuric hypercal- (a) If not going to do this, may mention intraoper- cemia, milk-alkali syndrome, and hyperthyroidism. ative PTH level Second, to document hyperparathyroidism, need to con- (b) Even 4 normal glands are found, continue neck firm hypercalcemia on repeat blood tests, then: exploration for rare presence of 5 or 6 glands

52 Part 1.qxd 10/19/05 2:51 AM Page 53

Strikeouts 53

(5) Consider cryopreservation in borderline cases espe- Pt sent to you after previously failed neck exploration cially if find only 3 hyperplastic glands elsewhere (a) You missed a gland—normal or abnormal Asked to comment on why four-gland exploration bet- (b) Patient has only 3 glands ter than exploration on just one side (justify what- ever position you offer) If you cannot find a gland: Pt may originally present very subtly with only fatigue (1) If upper gland, check paraesophageal, retrolaryn- or renal stones geal spaces, posterior mediastinum, and do thyroid Will be asked an innocent question like “How does lobectomy on that side PTH work?”(→increase bone resorption, increases (2) If lower gland, check tracheoesophageal groove, renal resorption of Ca and renal secretion of phos, carotid sheath, thymus, thyroid, anterior medi- and stimulates Vit D formation) astinum Management of hypercalcemic crisis (treat with IVF, (3) If you still can’t find gland, don’t do sternotomy lasix, steroids, calcitonin) first time around, but follow post-op Ca/PTH Parathyroid carcinoma levels

Common Curveballs Strikeouts Failing to rule out MEN syndrome Post-op has persistently elevated calcium/or comes back Failing to rule out common causes of hypercalcemia six months later with elevated calcium→ be method- Performing median sternotomy first time operation ical with complete work-up including calcium, phos- when find only 3 glands phorus, and PTH levels; MRI, sestamibi, U/S; one of Finding a single adenoma and stopping operation four choices: Going into long discussion about radioguided surgery (1) Missed adenoma (most likely)—could be a 5th for minimally invasive parathyroidectomy (very dan- gland, in mediastinum, or on same side gerous on the exam to describe any cutting edge sur- (2) Missed hyperplasia geries/technologies) (3) Parathyroid carcinoma Not knowing how to deal with post-op persistent (4) Parathyromosis hypercalcemia Can’t find 4 glands Not knowing how PTH works There will be more than 4 glands Not knowing indications for surgery Asked when you would consider cryopreservation of Not knowing percentage for finding single adenoma any of the glands (80%) Part of a MEN syndrome Not knowing how to deal with post-op complications Post-op hypocalcemia Not knowing where to look for “missing” upper or Post-op airway compromise lower gland Post-op hoarseness Part 1.qxd 10/19/05 2:51 AM Page 54

Endocrine—Thyroid Nodule

Concept Examination of vocal cords if new onset hoarseness Reflexes, pulse, BP (hyperthyroidism) May be “hot” nodule, benign adenoma, malignancy, or other neck mass (parathyroid, lymph node) Diagnostic Tests FNA—simple to perform in 1st office visit Way Question May be Asked? U/S—helps look for other nodules, determine solid vs. cystic “31 y/o female seen by PMD recently for a sore throat was Thyroid scan (cold vs. hot nodule) noted to have a mass in the left side of her thyroid. What Blood tests: T4, TSH, thyroglobulin level (for f/u), thy- do you want to do?” Could be presented with the mass roid antibodies, calcitonin and Ca++ level (if suspect found by a family MD and sent to you, could be given medullary CA) symptoms of hyperthyroidism. After your initial H + P, all Be ready for Ca++ to be elevated (change of scenario) patients get U/S and FNA. If suspect pheo, get calcitonin, serum calcium, phos- phate, urine studies for pheo, RET testing

How to Answer? Results of FNA Complete H+P (1) Clear fluid, nodule disappears (observe and send Questions related to hyper or hypothyroidism fluid to pathology—surgery if large number of fol- Tachycardia licular particles) Heat intolerance (2) Clear fluid, nodule doesn’t disappear or recurs Wt loss/gain more than twice→surgery) Fatigue (3) Bloody fluid—fluid to pathology but pt goes to Depression OR! Cold intolerance (4) Solid History of radiation to neck (breast/Hodgkin’s) (a) Clearly benign—thyroxin suppression for small History of new hoarseness/changes in voice 1 cm nodule non-toxic goiter, OR if nodule per- Any possibility of MEN syndrome (ask of pheochro- sists or enlarges over next 6 months mocytoma and hypercalcemia—about 10% (b) Suspicious—follicular cells→ to OR medullary’s associated with MEN) (c) Malignant—usually papillary as can can’t Family history: goiter, MENII, thyroid cancer differentiate malignant follicular neoplasm except Diarrhea (medullary) with final pathology showing capsular invasion

Physical Exam Surgical Treatment Attention to description of the mass Papillary CA Cervical lymph nodes Young pt and tumor less than 2 cm→ lobectomy

54 Part 1.qxd 10/19/05 2:51 AM Page 55

Strikeouts 55

All others get total thyroidectomy and removal of Common Curveballs any obvious enlarged nodes This allows easy follow-up with radioactive iodine, thyroid scans Be prepared for airway compromise post-op and thyroglobulin levels Be prepared for vocal cord paralysis post-op Follicular CA Asked about possible nerve injuries (recurrent laryngeal Total thyroidectomy and sample any obvious nodes and sup. laryngeal) and their consequences Be prepared for hypocalcemia post-op Medullary Part of a MEN syndrome + Total thyroidectomy Follicular cells on FNA Central lymph node dissection (from larynx to Justifying your reasoning for total thyroidectomy suprasternal notch) and modified RND on side of Will be nodules in both lobes palpable mass (if MEN, resect pheo first!) Will be goiter plus a nodule Anaplastic Thyroglobulin levels will increase several months post-op Total thyroidectomy, if possible, if not, split gland to Thyroid scan will show “hot nodule” relieve any tracheal compression Usually rapidly fatal Chemo/XRT Strikeouts

Know how to describe partial and total thyroidectomy Failing to rule out MEN syndrome and know your position on intra-op pathology con- Not knowing how to deal with post-op complications sults of follicular adenoma (controversial) Not performing FNA Post-op Not knowing when to follow calcitonin levels All pts get placed on Synthroid (enough to make (medullary carcinoma) and when to follow thy- TSH barely measurable) roglobulin levels Follow thyroglobulin levels (marker for recurrence) Not performing central node dissection in medullary Stop thyroxin for 2 weeks six weeks post-op and do carcinoma total body radioactive I131 scan to detect any Not placing pt on thyroxin post-op and following TSH residual tumor for ablation levels Repeat radioiodine scan q 6 months for next few years Medullary Ca is followed by calcitonin levels and DMSA scan (nuclear medicine) Part 1.qxd 10/19/05 2:51 AM Page 56

Esophagus—Zenker’s Diverticulum

Concept Diagnostic Tests Upper esophageal muscle dyscoordination with lower pha- Labs, EKG, CXR ryngeal constrictor contracting against an unyielding Don’t forget in elderly patients an assessment of the cricopharyngeus muscle. This causes an acquired (false preoperative status (pulmonary, cardiac, and renal diverticulum) mucosal out-pouching of the esophageal evaluation if necessary pre-op) wall between these muscles on the left posterolateral side UGI first maneuver—always before EGD, especially (Killian’s triangle). because EGD may perforate a Zenker’s as two lumens will be visualized on EGD

Way Question May be Asked? Surgical Treatment “A 78 y/o male presents on referral from his family doctor Left cervical incision over anterior border of SCM complaining of trouble swallowing with occasional regur- Diverticulum located in plane between carotid sheath gitation of undigested food, mainly at night.” Rarely will and trachea you get the patient with obvious bad breath, dysphagia to Bougie in esophagus to prevent narrowing when solids and liquids, regurgitation of undigested food, a sen- performing diverticulectomy with TA stapler sation of a lump in the throat, and gurgling in the neck. May invert and perform a diverticulopexy to Weight loss and aspiration are late symptoms. precervical fascia in elderly, high risk, with diverticulums greater than 3 cm to reduce risk of sta- ple line leak, but they will likely push you to perform How to Answer? resection May leave alone if less than 2 cm Must work through an algorithm for dysphagia and rule Always perform a cricopharyngeal myotomy—gentle out mainly achalasia and cancer cephalad traction on the diverticulum will expose fibers of the cricopharyngeus muscle which are History divided and bluntly dissected from the underlying mucosa and continued onto the esophagus for Solids vs. liquid dysphagia several centimeters Weight loss Always drain the incision! Smoking Don’t mention endoscopic alternatives to open GERD/Barrett’s resection Coughing up solid food Halitosis (bad breath) Gurgling in neck Common Curveballs

Physical Exam Pt develops a leak or wound infection post-op Performing a pexy leads to perforation of the Neck and lymph node basins (will never feel diverticulum) diverticulum

56 Part 1.qxd 10/19/05 2:51 AM Page 57

Strikeouts 57

Injury to the rec. laryngeal identified post-op Forgetting to perform the cricopharyngeal myotomy Diverticulectomy lead to narrowing of the esophagus Forgetting to perform UGI or not performing prior to after not using a bougie EGD Injury to esophagus intra-op Discussing the endoscopic treatment that you’ve never seen (never describe an operation you’ve never done. It’s better to say something like “I do not perform Strikeouts this operation, but I know the key aspects are the fol- lowing...”) Forgetting to leave a drain Not describing operative procedure properly Part 1.qxd 10/19/05 2:51 AM Page 58

Esophagus—Achalasia

Concept (2) Esophageal dilatation with pneumatic balloon under fluoroscopy to dilate and disrupt the Esophageal aperistalsis (loss of Auerbach’s plexus), failure fibers of the LES (be prepared for scenario to of the lower esophageal sphincter to relax, and resultant change to perforation here~4% risk). Dilatation esophageal dilation alone approx. 70% response rate. (3) BoTox injection with 80 units in four divided doses directed into the LES by — Way Question May be Asked? consider in older, debilitated pts (4) Surgical myotomy, can be done laparoscopically “37 y/o female presents on referral from her GI doctor or transthoracic. (only describe if you are familiar with difficulty swallowing liquids and solids and subster- with minimally invasive techniques otherwise, nal pain after meals for approximately 1 year.” Rarely will transthoracic approach.) you be given the diagnosis on referral or shown the typical “bird’s beak” on UGI. Patients may also have aspiration or Procedure referral for a megaesophagus. Left lateral transthoracic approach Double lumen ETT How to Answer? Esophageal bougie Longitudinal myotomy from a point 1 cm onto gastric Must be methodical in your approach to dysphagia cardia to inferior pulmonary vein because the scenario will end up being something Muscle edges should separate by 1–2 cm you leave out. DDx includes spasm, achalasia, stric- +/− antireflux procedure (controversy here but many ture, tumor-benign and malignant. surgeons will perform a partial wrap—Belsey, Dor, First, complete history including risk factors for malig- or Toupet) nancy as well as for the onset of the dysphagia. Post-op UGI with gastrografin prior to feeding Second, complete physical exam including epigastric masses and lymph node basins (will all be negative but if you leave out the PE, the pt will end up having Common Curveballs a pronounced supraclavicular node and the scenario will have changed to esophageal cancer with obvious Pt has a malignancy on endoscopy mets). Pt won’t have classic manometry Appropriate preoperative studies including full labs, Pt will perforate after EGD EKG, CXR. Pt will have “megaesophagus” Appropriate work-up of dysphagia which always Pt will have bad reflux post-op if you don’t do a wrap includes UGI, EGD (risk of malignancy increased in Pt will have bad PFTs and won’t tolerate a thoracotomy these pts), and manometry You perforate esophagus performing the myotomy (repair with absorbable suture and cover with wrap) Treatment should also be stepwise with: (1) Attempts at medical therapy with nitrates and calcium channel blockers 58 Part 1.qxd 10/19/05 2:51 AM Page 59

Strikeouts 59

Strikeouts You get stuck in referring pt back to GI doc and in describing medical therapy and won’t take pt to the You forget the UGI, EGD, or manometry OR for myotomy You can’t describe the myotomy or forget to mention extending onto stomach Part 1.qxd 10/19/05 2:51 AM Page 60

Esophagus—Esophageal Cancer

Concept Barium Enema/Colonoscopy if might use colon as conduit (don’t need angio pre-op) One of top leading causes of cancer deaths, squamous-cell Be complete, but don’t dwell on these as the examiner is has traditionally accounted for the majority of esophageal trying to get to more complicated issues like the indications cancers, but the frequency of adenocarcinoma is increas- for surgery/palliation, performance of the surgery, and ing. Risk factors include achalasia, Barrett’s, caustic management of complications after surgery. injuries, diverticula, leukoplakia, Plummer-Vinson syn- drome, smoking and alcohol use. Can discuss pre-op nutrition with J-tube feeds or TPN, but must be at least 2 weeks in duration pre-op to see any benefit

Way Question May be Asked? Contraindications for Resection “A 61 y/o male smoker presents to your office with a new Metastatic disease (must FNA to prove metastatic) onset of dysphagia. On review of symptoms, the patient Enlarged mediastinal/para-tracheal/celiac nodes has lost 15 pounds in the last month and the dysphagia is Fistula to the airway worse for solids compared to liquids.” Be careful of the adult patient that presents with an esophageal stricture— Non-Surgical Palliation must r/o malignancy with both biopsy and brushing of the stricture for cytology. Metallic stents Laser fulguration Feeding tubes How to Answer? Intraluminal tubes Chemo/XRT Complete H+P (wt. loss, vomiting, palpable mass, risk factors, check for supraclavicular node, enlarged Surgical Treatment liver) Laboratory tests (full labs, nutritional status pre-op) Can describe Ivor-Lewis procedure or Transhiatal pro- cedure Appropriate Diagnostic Tests Remember that cervical anastomosis is safer than intrathoracic UGI Any transhiatal might need to be converted to thoraco- Endoscopy and biopsy tomy if tumor is fixed to adjacent structures Bronchoscopy (if CA in upper 2/3 of esophagus to Can perform anastomosis as running or interrupted, r/o esophagobronchial fistula) single or double layer—staplers associated with CT scan chest and abdomen (enlarged LN’s- increased risk of stenosis here celiac/mediastinal), metastases to liver/lungs) If performing thoracotomy, remember pre-op tests Endoscopic U/S may be used to provide more accu- (PFTs) rate staging of tumor invasion and regional node Stomach is best organ to replace esophagus with status Always send to pathology to check intra-op margins

60 Part 1.qxd 10/19/05 2:51 AM Page 61

Strikeouts 61

Post-op UGI day 5–7 depending on preference Recurrent laryngeal nerve injury during the surgery Maintenance of chest tube/J-tube Management of late anastomotic stricture Questions regarding neoadjuvant chemo/XRT Key features of Ivor-Lewis: Pt may have feature suggestive of unresectability Abdominal portion first then right posterolateral Injury to trachea during cervical or thoracic dissection thoracotomy (usually to membranous portion of trachea and can Abdominal exploration usually advance ETT so that balloon distal to tear Create gastric tube and then perform repair (may need to split upper Preserve right gastric and right gastroepiploic sternum to accomplish)) Kocher maneuver to allow gastric pull-up Suture line recurrence Pyloroplasty/ Check frozen section of esophageal margin before mak- Double lumen tube to deflate right lung ing anastomosis Anastomosis in left neck If positive celiac node found during surgery, then what? Key features of Transhiatal: (this is unresectable disease) Be prepared to do thoracotomy Blunt mediastinal dissection Order = abdominal/cervical/mediastinal/anastomotic Strikeouts

Not treating a stricture as a possible malignancy and Common Curveballs performing simply an anti-reflux procedure (getting scenario wrong) Cancer presents as a new stricture in an adult Placing G-tube for pre-op nutrition and destroying Anastomotic leak—pt may become septic or may be a stomach as potential gastric tube silent leak only seen on post-op UGI—management Mentioning the use of VATS/ in perform- will depend on whether it is in the chest or the neck ing the surgical resection Necrosis of the gastric tube Trying to resect someone with obvious evidence of non- Saliva will come out of your chest tube (leak as above) resectability Being asked to describe diversion for total disruption of Not performing pyloroplasty/pyloromyotomy with gas- anastomosis post-op tric pull-up Pt has fever post-op Not being able to describe the operation Being asked how to boost nutritional status pre-op Not knowing non-operative methods of palliation Pt develops a chylothorax post-op Not being able to manage the possible complications of Delayed gastric emptying post-op your procedure (always a strikeout!) Part 1.qxd 10/19/05 2:51 AM Page 62

Esophagus—Esophageal Perforation

Concept Complete Physical Exam

Potentially lethal condition that may be spontaneous Vital signs (Boerhaave’s syndrome), result of trauma, swallowed for- Cervical exam (SQ emphysema=more with cervical eign bodies, ingestion of caustic substances, malignancy, perforations), or iatrogenic (NGT/endoscopy). Most common cause Diminished breath sounds in left chest today is iatrogenic related to endoscopic maneuvers (biopsy/dilatations/cautery). Appropriate Studies Full labs (amylase to r/o pancreatitis), Way Question May be Asked EKG (r/o MI), CXR (pleural effusion, hydropneumothorax, mediasti- “28 y/o male presents to ED with acute onset of epigas- nal air), tric/chest pain after several episodes of vomiting from Lateral neck x-ray (SQ emphysema) alcohol abuse.” Could also see after dilatation of stricture, Gastrografin swallow should identify leak. If not, may achalasia, biopsy for Barrett’s, dysplasia, or malignancy. then get barium swallow. You’ll be lucky if they mention to you any of the follow- No endoscopy! ing: crepitance in the suprasternal notch, diminished Treatment should also be stepwise with consideration breath sounds in the left chest, a left pleural effusion or air given to: in the mediastinum on CXR, or triad of vomiting/low tho- Time since perforation racic pain/cervical emphysema. Location/size of the perforation Pt’s overall clinical status Degree of contamination How to Answer? Underlying esophageal disorder (Barrett’s/malignancy)

As usual, history and physical exam, but make assessment All pts: of stability of the patient. A patient in shock needs urgent NPO, broad spectrum Abx, NGT treatment. Non-operative therapy only for pt with walled-off small perforation, minimal symptoms, no sepsis, with fre- quent reassessments and low threshold to take to OR History Once taken OR, all pts get same basic plan: Previous esophageal disorders Debridement Any history of ulcer disease Closure (of perforation) Pancreatitis Drainage Heart disease Ingestions Timing of pain to vomiting

62 Part 1.qxd 10/19/05 2:51 AM Page 63

Strikeouts 63

(1) If within 24 h of perforation, attempt primary the pt is unstable, or can be performed immediately repair with stomach and a left cervical anastomosis (a) for perforations in upper two thirds of esopha- Post-op UGI with gastrografin prior to feeding once gus, right thoracotomy, debridement of all non- resolution of sepsis and repair has chance to heal! viable tissue, myotomy to define extent of mucosal injury, closure in two layers over NGT, cover with tissue flap (pleural/pericardium/inter- Common Curveballs costal muscle), and place chest tube. Pt is then kept NPO, on TPN or enteral feeds through J- No leak seen on gastrografin swallow tube, and continue abx. CXR/neck x-ray will be negative (b) for perforation in lower third of esophagus, left Pt will be unstable thoracotomy and same procedure as above with Location of the leak will change after you suggest an ability to now cover repair with Thal patch, algorithm diaphragm muscle, fundoplication Time to detection of the leak will change after you sug- (c) for perforation in abdominal esophagus, upper gest an algorithm midline with low threshold to making left tho- Examiners pushing you towards non-operative therapy racotomy (make sure to prep left chest) and Nonoperative therapy will fail cover repair with fundoplication Pt will have leak after your repair (esophageal diversion) (d) for cervical perforation, incision in left neck Pt will have perforation during dilatation of achalasia along SCM or during examination of Barrett’s ( (2) If after 24 h unstable, perform esophageal exclusion appropriate here) (a) debridement of mediastinum/esophagus/lung Management late stricture (b) establish effective drainage with two chest tubes (c) leave esophagus open (d) ligate GEJ with 2 ties of #2 chromic Strikeouts (e) place NGT above perforation (f) high volume irrigation through NGT Performing endoscopy to identify leak (and spread (g) NPO/TPN or J-tube feeds/abx infection/create tension pneumothorax) (c) or segmental esophagectomy Not performing water soluble contrast enema (d) /J-tube Attempting to perform immediate resection/reconstruc- (e) cervical esophagostomy tion with unprepped colon (f) NPO/TPN or enteral feeds/abx Not ruling out distal obstruction Not attempting primary repair on early perforation (3) Esophageal resection is appropriate if there exists Not r/o MI/pancreatitis/perforation PUD/aortic pathology in the esophageal wall, this is delayed if dissection Part 1.qxd 10/19/05 2:51 AM Page 64

Esophagus—Esophageal Varices

Concept Resuscitation Life-threatening complication of . A IV access, CVP, labs (especially coags), T+C, transfu- major concern is often stabilizing these patients and know- sion pRBC/FFP ing appropriate timing of surgical intervention. Also, NGT, lavage stomach make sure to rule out other potential sources for UGIB. Treatment Way Question May be Asked? Can consider Sengstaken-Blakemore tube after intubat- ing pt (be prepared to describe technique) “A 51 y/o male presents to the emergency department with Start Pitressin drip 0.4 U/min (add nitroglycerin gtt if three episodes of massive hemoptysis. His social history is h/o CAD and consider SGC) remarkable for extensive alcohol abuse and his past med- Start Somatostatin gtt (25 micrograms/hr) ical history is remarkable for multiple admissions for alco- Beta blocker to lower HR if not lower than 100 (give holic pancreatitis.” May be referred to you with the slowly as may precipitously drop SBP) diagnosis of bleeding varices from another hospital, or (at the back of your mind, should be considering Child’s simply a patient with an UGIB. class as Child’s C pts need liver txp—Bilirubin > 3, albumin < 3, severe ascites) How to Answer? Endoscopy (once hemodynamically stabilized) At EGD, can consider Sclerotherapy Brief, focused H+P while resuscitating the patient Banding (neither will be available or will work!) History If EGD fails, can consider TIPS (won’t be available!) Alcohol use Episodes of encephalopathy, bleeding varices History of pancreatitis Surgical Treatment Indications—Uncontrolled History of PUD Bleeding (you must do this while resuscitating, or you will have Emergency portosystemic shunt (mesocaval 8 mm PTFE great history on a dead pt!) shunt b/w SMV and IVC) →identify middle colic vein, follow distally to SMV (to right of SMA), iden- Physical Exam tify IVC through right colonic mesentery adjacent to duodenum, anastomose to IVC first, then to side of Stigmata of liver disease SMV (can use left IJ if contaminated field). This Ascites

64 Part 1.qxd 10/19/05 2:51 AM Page 65

Strikeouts 65

shunt doesn’t dissect in porta hepatis so doesn’t com- Pt will develop hepatic failure or hepatorenal syndrome promise potential for future liver txp post-op Asked to describe other shunting procedures Other choices: Gastric devascularization EEA limited esophagectomy with ligation of the Strikeouts left gastric vein Gastrostomy and suture ligation Performing distal splenorenal shunt Not being able to describe your surgical procedure Common Curveballs Describing the Suguira procedure (you don’t want to do something you’ve never done before and this is rarely done in the U.S.) UGIB secondary to PUD, esophagitis, gastric varices Rushing to the operating room Bleeding will continue post-op Not performing EGD/trying sclerotherapy/banding Pt will become encephalopathic post-op Not knowing how to use Sengstaken-Blakemore tube Pt will have had prior abdominal surgery Not resuscitating the pt properly Pt will be Child’s class C Pt will aspirate or perforate after balloon tamponade Pt with thrombosed splenic vein and bleeding gastric varices (needs only a splenectomy) Part 1.qxd 10/19/05 2:51 AM Page 66

Esophagus—GERD

Concept Appropriate work-up of GERD which always starts with Barium UGI Incompetent lower esophageal sphincter related to inade- Look for reflux, hernia, shortened esophagus, diver- quate pressure (< 6 mmHg), inadequate length (< 2 cm), ticula, other motility disorders insufficient intraabdominal esophagus (< 1 cm). Hiatal The next test should be an upper endoscopy to evaluate hernia, delayed gastric emptying, and bile reflux may com- the severity of the reflux plicate the picture. Stage I erythema and edema II ulcerations III stricture Way Question May be Asked? The rest of the work-up must include: Manometry to r/o ineffective motility that will affect “45 y/o man presents to your office with a history of heart- your type of anti-reflux procedure, to document burn, choking at night, and recent onset of asthma.” the low LES pressures, and determine location of Symptoms may be many and can include chest pain, water LES brash, adult onset asthma, dysphagia, odynophagia, 24 h pH monitoring to document the relationship sinusitis, aspiration pneumonia, choking feeling at night, between pt’s symptoms and reflux as well as pro- regurgitation, excessive salivation, and chronic hoarseness. vide a baseline for post-op evaluation of success Also, patient may present with a complication of their of surgery reflux disease: Barrett’s, stricture, or ulcer. (A gastric emptying study should be added in any pt with a history of significant belching or bloat- How to Answer? ing after meals and/or history of duodenal ulcer as a delay in gastric emptying contributes to 10% of Nissen failures) Must be methodical in your approach to not get Remember, the indications for surgery are failure of caught by the pt with abnormal motility who you medical therapy or complications of reflux disease decide to perform a complete wrap on. (young age is a relative indication) First, complete history including relationship to meals, Procedure = (usually performed solids versus liquids, maneuvers tried (loose clothing, laparoscopically) caffeine cessation, trial of H2 blockers or PPIs) Lithotomy position Second, complete physical exam including epigastric 5–6 ports masses and lymph node basins (will all be negative Nissen performed over a bougie (54–56) but if you leave out the PE, the pt will end up having Start dissection at gastrohepatic ligament a pronounced supraclavicular node and the scenario Mobilize esophagus well into mediastinum will have changed to esophageal cancer with obvious Divide short gastrics down 1/3 along greater curve mets). Crural repair Appropriate preoperative studies including full labs, 3 cm anterior wrap EKG, CXR.

66 Part 1.qxd 10/19/05 2:51 AM Page 67

Strikeouts 67

Take care not to injure the vagus nerve, perforate Fundoplication falls apart post-op (technical failure) stomach, esophagus, injury spleen, or make wrap Pt has “gas bloat” syndrome post-op (inadequate gas- too tight/too long/ or twist the stomach when tric emptying) passing it around esophagus Pt has difficulty swallowing post-op (made wrap too Belsey, Dor, or Toupet can be performed in pts with tight) ineffective esophageal motility Post-op UGI with gastrografin prior to feeding Strikeouts

Common Curveballs You forget the UGI, EGD, or manometry You can’t describe the fundoplication or forget to men- Pt has a malignancy/Barrett’s/stricture on endoscopy tion taking the short gastrics (non-dilatable stricture=OR) You take pt to surgery right away without trying Pt won’t have normal manometry (don’t Nissen) medical therapy (only 5 to 10% GERD pts need Perforation during procedure by you, by advancing surgery) bougie, or seen post-op on UGI You don’t take an adequate history or order manome- Pt will present with a stricture where first you must r/o try and perform Nissen on pt with achalasia malignancy and dilate prior to any studies Discussing endoscopic measures to treat Barrett’s Pt will have “shortened esophagus” (be prepared to (cryotherapy or phototherapy) describe Collis gastroplasty) Discussing endoscopic measures to treat GERD Pt will develop pneumothorax or bleeding from (“Plicator,” “Stretta,” or newly approved injectable liver/spleen during procedure agents) Fundoplication herniates into chest post-op (poor hiatal closure, didn’t mobilize esophagus enough) Part 1.qxd 10/19/05 2:51 AM Page 68

Esophagus—Hiatal Hernia

Concept Physical Exam Four types: Abdominal masses I Sliding hernia, GEJ in chest Lymphadenopathy II Paraesophageal, GEJ in abdomen III Type II with shortening of esophagus and GEJ Diagnostic Tests in chest IV Additional abdominal organs (spleen, colon) in Usual labs (check H/H as often pts are anemic) hernia defect CXR (may see gastric bubble behind heart shadow) Oral Exam questions seem to be surrounding the treat- Barium Swallow ment of Type II and III paraesophageal hernias that are Upper endoscopy (examine for ulcerations, malignan- symptomatic. cies, diverticula) Manometry (helpful when deciding what degree of wrap to perform) Way Question May be Asked? Indications for surgery: (all paraesophageal hernias get surgery) “53 y/o female presents to the office complaining of full- Volvulus ness in her chest immediately after eating and associated Ulcerations weight loss.” May also have postprandial pain, may not be Anemia related to any specific solid or liquid foods. Pain likely epi- gastric. Surgical Treatment Abdominal incision (can consider thoracic if believe How to Answer? shortened esophagus or prior abdominal operations) Reduction of stomach (herniated contents) and Complete history and physical exam (may have been given inspection all this already): Excision of hernia sac Closure of diaphragmatic defect (interrupted, non- + − History absorbable sutures / mesh) Anchoring stomach (Stamm gastrostomy or Character of pain to abdominal wall or to arcuate ligament) Relation to meals “Floppy” Nissen (unless severely abnormal manometry) Relation to solids/liquids Previous esophageal problems/diagnoses Heartburn Common Curveballs Dysphagia Vomiting Pt has a malignancy on endoscopy Anemia Pt will have shortened esophagus (→Collis gastroplasty) Early satiety Pt will have esophageal/gastric mass on UGI

68 Part 1.qxd 10/19/05 2:51 AM Page 69

Strikeouts 69

Incarcerated contents (stomach) will be strangulated Strikeouts and need resection Can’t close crura without the use of a mesh You forget the UGI or EGD Abnormal manometry so can’t do 360 degree Nissen You forget to anchor the stomach or perform a wrap wrap You try to treat the pt medically/non-operatively once Being asked if it is necessary to perform anti-reflux you’ve diagnosed a paraesophageal hernia procedure You treat your pt with strangulation in the hernia sac as Pt will have pneumothorax from dissection of adhe- having angina or an MI sions from sac to pleura Post-op recurrence of hernia Being asked how tight to reapproximate the crura Injury to stomach or esophagus during procedure (change scenario!) Part 2.qxd 10/19/05 2:52 AM Page 70

Genitourinary—Renal Mass

Concept CT guided needle bx—rarely done for solid lesions as risk of bleeding and tumor seeding Unusual question, but more important in everyday surgical Angiogram—pre-op embolization of large lesions is a life. Variety of renal masses including abscess, cyst, benign consideration tumors, and malignancy—primary or metastatic. Look for suggestive history. Don’t fall prey to percutaneous biopsy of Surgical Treatment suspected malignancy, and don’t confuse with adrenal lesion! Dependent on lesion RCC gets radical nephrectomy How to Answer? Transitional Cell Carcinoma (TCC) gets radical nephroureterectomy History Don’t go into describing these—say you would refer to Flank pain a urologist! Hematuria Make sure to assess renal function and contribution Fever, chills from side you are planning to remove—pt may need Family history of Renal Cell Carcinoma (RCC) post-op hemodialysis Prior malignancy (lung or breast can metastasize to kidney) New varicocele (especially left renal mass) Common Curveballs Hypercalcemia Tuberous sclerosis Lesion will be cystic with internal echoes/septae Renal insufficiency Pt will need hemodialysis post-op Lesion will be TCC if you don’t rule it out with Physical Exam ureteroscopy/bx Pt will bleed from any percutaneous bx attempt Unlikely to feel the mass Pt will be symptomatic from simple cystic lesion (just Varicocele need to unroof) Evidence of metastatic disease Will actually be an adrenal lesion (changing scenario)

How to Answer? Strikeouts Need complete labs, CXR, U/A Diagnostic tests can include: Offering laparoscopic kidney resection IVP—usually the first test to evaluate a pt with hema- Percutaneous bx of solid lesion turia Not referring to urologist (at least try before taking on U/S—can determine if cystic or solid, can evaluate for a case outside of your specialty) simple cysts with no septa or calcifications (these can Not ruling out metastatic disease IVC thrombosis—can be symptomatic but almost always benign) extend to right atrium CT—can evaluate solid lesion, inspect renal vein and Not checking for possible lung/breast CA with met to IVC, look for metastatic lesions kidney MRI—if any doubt about IVC thrombus Performing radical nephrectomy only for TCC 70 Part 2.qxd 10/19/05 2:52 AM Page 71

Genitourinary—Scrotal Mass

Concept Surgical Treatment On the oral exam, most likely to be a testicular mass. Don’t (1) Inguinal orchiectomy (control spermatic cord early to forget however that hydrocele, varicocele, inguinal hernia, minimize tumor spread testicular torsion, and spermatoceles can all present as (2) Seminoma (check HCG here) scrotal masses. Important points for your DDx are: is it (a) negative markers + minimal enlargement paraaor- painful and does it transilluminate? tic nodes→ radiation to paraaortic and ipsilateral pelvic nodes (b) bulky retroperitoneal nodes, positive markers, or Way Question May be Asked? distant mets→ platinum based chemotherapy “You are called to the ED to evaluate a 27 y/o male res- (3) Non-seminomatous tumors (embryonal, teratoma, ident who has noticed a 2 cm mass in his left testicle. teratocarcinoma, choriocarcinoma) What do you do?” Age group most at risk is young (a) retroperitoneal lymph node dissection followed by males. chemotherapy unless enlarged retroperitoneal nodes, then chemo first (b) chemotherapy with bleomycin, etoposide, and How to Answer? platinum History Pain (timing) Common Curveballs Hernia Does patient perform own testicular exams? Scenario will change from seminoma or non-semino- matous tumor Physical Exam Pt will have enlarged retroperitoneal nodes on CT Pt will have evidence of metastatic disease Hernia Pt will have post-op ejaculatory dysfunction Transillumination Tumor markers will be positive or rise in post-op follow up Tenderness Post-op recurrence—abdomen/chest Lymphadenopathy

Diagnostic Tests Strikeouts Labs (AFP, HCG, LDH) Not performing inguinal orchiectomy CXR (r/o metastatic disease) Not ordering tumor markers U/S of scrotum (all pts) Not knowing that seminoma is very radiosensitive CT scan (look for paraaortic lymph node enlargement) Not knowing what to do with regards paraaortic lymph nodes Not performing CT scan to evaluate retroperitoneum

71 Part 2.qxd 10/19/05 2:52 AM Page 72

Hepatobiliary—Gallstone Ileus

Concept Obstruction series CT scan (not usually necessary) Mechanical obstruction in the terminal ileum from a large gallstone that has eroded through the into the Surgical Treatment duodenum. Seen in elderly patients with SBO who have no hernia and no previous . Resuscitate the pt, NGT, IVF, then: OR for exploration: Full ex lap (be prepared to describe this) Way Question May be Asked? Check status of RUQ (extensive scarring prevents definitive procedure) “A 74 y/o female is seen in the emergency department for a Enterotomy is performed proximal to palpable stone small bowel obstruction. Obstruction series confirms the lodged in the terminal ileum small bowel obstruction with air in the biliary tree. What do “Milk” stone back gently you want to do?” May be given AXR with stone in the RLQ Close enterotomy in two layers or air in the biliary tree or patient with episodes of SBO. Check rest of intestine for additional stones (~5%) Attention to RUQ (mortality less in retrospective series if done in separate procedures!) How to Answer? → takedown of fistula and closing the bowel in two layers, cholecystectomy and cholan- H+P while resuscitating the pt giogram (to look for other stones) (only if inflammation not severe, pt is stable, History and scarring will not preclude safe dissection) Prior surgery Malignancy history Common Curveballs Overall medical condition History suggestive of gallbladder disease Pt will have history of malignancy History of intermittent obstruction classic Pt will have associated intraabdominal process Pt will have severe scarring in RUQ precluding defini- tive procedure Physical Exam Pt will have bowel obstruction post-op (missed a second Vital signs (pt may be unstable) stone) Check for surgical scars Pt will be septic/unstable Check for hernias! Stone will have eroded through hepatic flexure of colon rather than duodenum Diagnostic Tests Being asked how to close the fistula Gallbladder cancer that led to the perforation (change Full lab panel (including LFTs—may be other stones) scenario)

72 Part 2.qxd 10/19/05 2:52 AM Page 73

Strikeouts 73

Pt will develop cholangitis or intraabdominal abscess Not checking for prior surgeries post-op Not recognizing the problem Post-op biliary leak Not “milking” back the stone but making enterotomy in terminal ileum Performing takedown of fistula in unstable pt Strikeouts

Not checking for hernias Not getting obstruction series but skipping to CT scan Part 2.qxd 10/19/05 2:52 AM Page 74

Hepatobiliary—Liver Abscess

Concept CT scan abdomen/pelvis Agglutination/compliment fixation tests to r/o amebic Usually a complication of an underlying disease process abscess (appendicitis, biliary disease, diverticulitis). Less likely the result of amebic infection. More commonly today associated Surgical Treatment with immunosuppression (HIV) or IVDU (endocarditis). Amebic abscess (Entamoeba histolytica) →Flagyl unless: Secondary infection Way Question May be Asked? Rupture into biliary tree or abdominal cavity Failure to initially improve on abx (may need to be “A 35 y/o male is evaluated in the ED for fever, chills, and on abx for months if see initial improvement) a constant, dull ache in the right flank. He has a history of Pyogenic abscess IVDU, and his CT scan shows multiple liver abscesses. (Either from biliary tree or from portal venous system What do you want to do?” from direct extension from adjacent organ) → Percutaneous drainage and IV Abx (can try to treat multiple small abx with IV abx) How to Answer? → Open drainage if percutaneous is not possible: (depends on location of abscess) Full H + P posteriorly through bed of 12th rib and extraperi- toneal approach interiorly through subcostal inci- sion and extraperitoneal approach transperitoneally History IVDU Common Curveballs HIV Recent abdominal infections Pt will have history of malignancy Travel hx Pt will have associated intraabdominal process History of malignancy (could this be presentation of Pt will have history of IVDU/HIV metastatic disease) Pt will have amebic abscess Will have multiple abscesses Physical Exam Will need to perform open drainage and be asked to describe your approach Full physical especially abdominal exam (liver enlarge- Amebic abscess will rupture into abdominal cavity or ment, tenderness) biliary tree Lymphadenopathy Will need to describe treatment of diverticulitis or cholangitis (change of scenario) once you take care Diagnostic Tests of abscess Hepatitis panel/LFTs (still working up RUQ pain) CBC Ultrasound RUQ 74 Part 2.qxd 10/19/05 2:52 AM Page 75

Strikeouts 75

Strikeouts (Ecchinococcal/hydatid cysts identified by electropho- resis, initially treated with mebendazole, failure to resolve Not checking for pyogenic abscess from abdominal source demands first ERCP to r/o communication with biliary Not ruling out amebic abscess tree, then surgery and injection of cyst with hypertonic Not getting biopsy of abscess wall to r/o malignancy saline, avoiding any spillage—anaphylaxis—and perform- Mixing up treatment of Ecchinococcal cysts and amebic ing pericystectom y) abscess Part 2.qxd 10/19/05 2:52 AM Page 76

Hepatobiliary—Liver Mass

Concept +/− CEA if suspect colorectal recurrence CBC Usually found during an exploratory laparotomy per- Ultrasound RUQ (used to r/o solid lesion from formed for colon cancer, GI bleed, or other unrelated rea- cyst/abscess—different scenario) sons. May also be diagnosed incidentally on CT or U/S CT scan abdomen/pelvis (central scar associated with performed on a patient with abdominal pain. Make sure to FNH) differentiate solid from cystic lesions here. Hemangioma is MRI most common benign tumor. Half of adenomas present +/− Tagged RBC technetium scan (r/o hemangioma) with spontaneous bleeding. +/− Angiography—helpful in evaluating primary malig- nancies

Way Question May be Asked? DDx “A 37 y/o female is evaluated in the ED for abdominal pain Hemangioma and the CT scan shows a 3 cm mass deep in the right lobe FNH of the liver. What do you want to do?” May be in the sce- Adenoma nario of doing an ex lap and finding an incidental lesion in Malignancy (primary or metastatic) the periphery of the liver or patient may present hypoten- sive with abdominal pain. Surgical Treatment (1) FNA under CT guidance—helpful if diagnoses How to Answer? malignancy (don’t do if suspect hepatocellular can- cer (elevated AFP, hepatitis B positive, cirrhosis) History (2) If FNA negative, need core needle bx by Hepatitis laparoscopy or open surgery Previous malignancy (colon CA) (3) Treatment OCP use (a) FNH—observation unless becomes sympto- Weight loss/anorexia matic Race (Africa/Southeast Asia associated with HCC) (b) adenoma—stop BCP and observe for 6 months, Abdominal pain resect if: becomes symptomatic Physical Exam increases in size during observation period pt intends on becoming pregnant Full physical especially abdominal exam (liver enlarge- (c) malignancy— ment, tenderness) (1) Can resect metastatic disease if colon or neuroendocrine malignancy as long as pri- Diagnostic Tests mary site controlled (5 yr survival ~30% from metastectomy for Hepatitis panel/LFTs colorectal ancer if < 5 mets and less than +/− AFP (if suspect HCC) 5 cm in size) 76 Part 2.qxd 10/19/05 2:52 AM Page 77

Strikeouts 77

(2) Hepatocellular—be prepared to describe Adenoma/hemangioma will bleed spontaneously dur- liver resection ing your observation period and pt will present in (d) hemangioma—dx by CT, MRI, or tagged RBC hemorrhagic shock scan FNA will be negative but pt. has bleeding when doing observe unless very large or symptomatic open biopsy can cause pain, hemolysis, CHF You get into bleeding during open biopsy (change spontaneous rupture rare (1–2%) scenario) embolization 1st choice if symptomatic often surgically treated by enucleation (4) Incidental Liver Lesion Strikeouts biopsy necessary FNA to make sure not cystic or hemangioma Not ruling out a cystic lesion Consider intra-op U/S Not knowing treatment for FNH or hepatoma Can perform wedge resection if small and Sticking needle into hemangioma peripheral Not knowing how to describe your liver resection Not trying to biopsy a liver lesion found during an exploratory laparotomy Common Curveballs Not performing CT scan with contrast Performing metastatectomy for breast, stomach cancer Pt will have history of malignancy Performing FNA on hepatocellular cancer Being asked when you will perform resection for Performing liver resection laparoscopically metastatic disease Getting lost in a discussion about angiographic Liver nodule found during exploratory laparotomy— embolization when pt clearly needs to go to OR “what would you do?” (resuscitate/check coags first) Change scenario and pt will have cystic rather than solid lesion in the liver Part 2.qxd 10/19/05 2:52 AM Page 78

Hepatobiliary—Post-cholecystectomy Cholangitis

Concept Plain x-rays of abdomen (look for number/position clips) Suggestion here is that a stone was missed and the patient RUQ U/S (look for dilated ducts, evaluate CBD, biloma returns with an obstructed biliary tree and septic from mass?) ascending cholangitis. Prompt treatment is important here as well as stabilizing patient in ICU setting. Surgical Treatment (1) ABCs Way Question May be Asked? (2) IVF resuscitation (3) ICU “55 y/o male seen in the ER one year after a laparoscopic (4) CVP cholecystectomy with fever, chills, RUQ pain, and jaun- (5) Abx (broad spectrum) dice. What do you want to do?” May be a variable interval (6) U/S RUQ after lap chole. Your DDx needs to include: retained stone, (7) ERCP→to drain CBD, remove stone if possible new stone, stricture, tumor, extrinsic compression. (needs to be under 1.5 cm in size), biopsy any mass (8) PTC if ERCP fails→to drain biliary tree (9) OR if ERCP/PTC both fail after best attempts to How to Answer? stabilize pt: Goal here is to drain biliary tree by whatever means Start with ABCs here if pt is septic from cholangitis obtain- possible! ing any relevant H + P while resuscitating the pt (a) CBD exploration, extract any stone, if cannot, place T-tube and close remember: History always to do cholangiogram when placing a T-tube and use absorbable sutures in CBD Time course closure History of hepatitis use choledochoscope History of hemolysis biliary fogarties Malignancies irrigation Previous operative indications/report a must stone forceps bring out T-tube through abdominal wall Physical Exam with as straight a course as possible (b) other options: Abdominal exam choledochoduodenostomy (and leave stone Confirm jaundice behind) sphincteroplasty (description below) Data choledochojejunostomy as possible Full laboratory panel including LFTs

78 Part 2.qxd 10/19/05 2:52 AM Page 79

Strikeouts 79

Common Curveballs Strikeouts

Porta hepatic severely scarred down and you cannot Not trying ERCP or PTC but rushing to surgery safely mobilize the duodenum Not knowing other options besides CBD exploration GI doc not available for ERCP and placing T-tube Stone cannot be removed by ERCP Not being able to describe CBDE PTC won’t work to decompress biliary system Trying to perform anything laparosocpically May be iatrogenic injury to CBD (change scenario) Not stabilizing pt prior to any maneuvers Being asked to describe transduodenal sphincteroplasty Discussing endoscopic Open duodenum longitudinally, open medial wall of Seeing stone on cholangiogram and doing post-op ampulla directly onto stone itself, identify pancreatic ERCP rather than CBDE duct orifice (may need to give IV glucagon here), suture ductal mucosa to duodenal mucosa with fine absorbable suture Remember Indications for CBDE: Pt will have a malignancy Pt will have a stricture Positive intraoperative cholangiogram Pt will have other comorbidities (recent MI, ARF, and Large stone scenario will change to ICU management) Impacted stone (usually distally impacted) Multiple stones Cholangitis and failed ERCP/PTC Part 2.qxd 10/19/05 2:52 AM Page 80

Hepatobiliary—Post-cholecystectomy Jaundice

Concept Hepatobiliary scan (HIDA) Look for CBD occlusion or leak Variety of postcholecystectomy problems you could be Look for cystic duct/duct of Lushka leak faced with. Always be methodical in your work-up of CT scan these patients and don’t rush to the OR. Look for biloma (extrinsic compression of biliary tree) Drain percutaneously Way Question May be Asked

“A patient returns to your office one week after an Surgical Treatment uneventful laparoscopic cholecystectomy performed by Admit to hospital your partner for symptomatic cholelithiasis and jaundice. Start antibiotics What do you want to do?” May happen more immediately Get appropriate studies as above after surgery. May be given a difficult intra-op dissection. Surgical management based on HIDA/ERCP May be given a history of CBD stones. findings (1) CBD leak stent across with ERCP How to Answer? drain biloma percutaneously History (2) Cystic duct leak stent across with ERCP Hepatitis drain biloma percutaneously Hemolysis all will close if no distal obstruction Indications for surgery (3) CBD occlusion Operative report (was IOC performed?) ERCP to determine nature of obstruction, Timing of Jaundice retrieve stone, phincterotomy Change color of urine/stool If iatrogenic, schedule pt for exploration (wait Sxs/sgs of cholangitis 3–6 weeks as long as pt not septic) for clip removal, repair of CBD over T-tube or Physical Exam choledochojejunostomy) (4) Duct of Lushka leak Check for icterus drain biloma Check incisions ensure no distal obstruction Full abdominal exam observe (these may need a return to OR for suture ligation) Data (5) If create choledochojejunostomy, use a roux limb, end to side with end of jejunum against General labs especially LFTs, amylase abdominal wall so you have some access to Abdominal x-rays biliary tree if need be U/S of RUQ

80 Part 2.qxd 10/19/05 2:52 AM Page 81

Strikeouts 81

(6) Don’t forget other options appropriate in certain Asked how to construct choledochojejunostomy scenarios: Won’t be enough length to perform primary repair to hepaticojejunostomy CBD choledochoduodenostomy Simple drainage of biloma won’t work Electrolyte abnormalities from high drain output Common Curveballs Strikeouts ERCP not available (don’t forget about PTC) Pt develops cholangitis Rushing back in to reoperate Being asked how you will discuss with your pt an iatro- Not getting U/S, HIDA, or ERCP genic injury Not knowing ways to treat CBD leak or occlusion Asked to treat injury to CBD (can repair primarily if < Not recognizing possible iatrogenic injury to CBD 50% over T-tube) Not being honest about injury if asked the ethical ques- Asked how to treat retained stone that fails ERCP tion of how do you discuss the situation with your pt (describe CBDE → change scenario) Part 2.qxd 10/19/05 2:52 AM Page 82

Pancreas—Acute Pancreatitis

Concept Abdominal exam (peritonitis, evidence of hemorrhagic pancreatitis) Life-threatening condition representing a massive retroperi- Prior incisions toneal burn with tremendous amounts of third-spacing. Hernias (could this be simply obstruction) May be secondary to alcoholism, gallstones, tumor, elevated triglycerides, medication, or even perforated ulcer. Diagnostic Tests Complete laboratory panel including amylase, lipase, Way Question May be Asked? calcium, albumin, LDH, ABG (depending on sever- ity of illness) “Called down to the ER to evaluate a patient with severe CXR (r/o esophageal rupture) epigastric pain, vomiting and history of recent alcohol use. AXR (look for ileus/sentinel loop/obstruction/aortic What do you want to do?” May or may not be given his- calcifications/free air/gallstones) tory of gallstones or alcohol use. May be given elevated Ultrasound (look for gallstones, ductal dilatation, amylase. Need to rule out other causes of severe epigastric examine pancreas, r/o AAA) abdominal pain including gastritis, perforated ulcer, rup- ture of esophagus if history of emesis, AAA, ischemia, Ranson’s criteria and gastric volvulus. On admission: age > 55 WBC > 16 How to Answer? Glucose > 200 SGOT > 250 Key parts: assessment of severity, determining etiology, LDG > 350 aggressive volume resuscitation, appropriate support During first 48 h: (nutrition/vent), operative intervention when appropriate, Hct fall by 10 recognition of possible complications BUN rise by 5 Calcium < 8 History Fluid sequestration > 600 mL PO2 < 60 Gallstones Alcohol use Surgical Treatment Recent new medications Timing of pain with retching/vomiting (1) supportive care with: History of PUD/AAA IVF, NPO, NGT, serial labs, H2 blockers, IV anal- gesics Physical Exam (2) CT scan to evaluate for complications, necrosis (3) Ventilatory/nutritional support where appropriate Complete PE including: (pt will likely deteriorate on Oral Boards and will Crepitus over chest/neck (r/o Boerhaave’s) need txfr to ICU, CVP, intubation, TPN)

82 Part 2.qxd 10/19/05 2:52 AM Page 83

Strikeouts 83

(4) Most pts should improve in next several days (on Pt will develop pancreatic ascites the exam, of course, your pt will not) Pt will develop ascending cholangitis (change scenario (5) If fails to improve, check repeat CT to discussion of how to drain dilated biliary tree) (a) FNA if any necrosis present and pt having fever Pt will be pregnant (can observe sterile necrosis) Pt will have gallstone pancreatitis (b) if FNA +, necrosectomy: Pt will later present with symptoms of chronic pancre- chevron incision atitis open up lesser sac Pt will develop UGIB related to splenic vein debride all devitalized necrotic tissues thrombosis large volume lavage of abdomen Asking you to discuss use of antibiotics/ leave large drains in lesser sac somatostatin Discussing Ranson’s criteria (6) Pt will develop epigastric mass Pt will have obstructing tumor/gallstone as cause for (a) CT or U/S to confirm pseudocyst pancreatitis (b) Do not do FNA Pt will need nutritional/respiratory support and asking (c) +/− ERCP to r/o ductal communication you to discuss these modalities (d) feed past ampulla or TPN for 8 weeks and Pt will develop DTs as result of alcohol withdrawl reassess (allow wall to mature) (e) various options for internal drainage depending on location Strikeouts (7) Pt gets better and cause was gallstones (a) need MRCP or ERCP pre-op if suspect Trying to percutaneously drain infected necrosis choldecholithiasis, possible sphincterotomy and Not recognizing/appropriately treating complications stone retrieval of pancreatitis (b) be prepared for intra-op cholangiogram and Failing to rule out other possible causes of severe epi- CBDE gastric/abdominal pain (c) lap chole same admission Not being aggressive in your supportive care for the pt Not trying to identify etiology of pancreatitis Not performing MRCP/ERCP pre-op or IOC intra-op Common Curveballs when doing lap chole after bout of pancreatitis

Pt will develop pancreatic necrosis Pt will develop pseudocyst Part 2.qxd 10/19/05 2:52 AM Page 84

Pancreas—Chronic Pancreatitis

Concept Stigmata of alcoholic liver disease Abdominal exam (ascites, epigastric tenderness c/w Etiology: alcohol abuse, hyperparathyroidism, cystic fibrosis, acute pancreatitis) pancreatic divisum, trauma. Alcohol related most common in developed countries. Variety of suggested mechanisms: Diagnostic Studies hypersecretion of protein from acinar cells, plugging of pan- creatic ducts with protein precipitates, and pancreatic ductal Lab tests (only for completeness—IV secretin and CCK hypertension. Pathology: acinar loss, glandular shrinkage, stimulation with collection of pancreatic effluent, 72 proliferative fibrosis, calcification, ductal stricturing h fecal fat, glucose tolerance testing to measure endocrine function) AXR: pancreatic calcifications 95% specific if seen Way Question May Be Asked? CT: evaluate parenchymal disease, pseudocyst, ductal dilatation “45 y/o alcoholic with a history of several episodes of pan- ERCP: ductal dilatation, strictures, calculi, chain of creatitis presents now with worsening abdominal pain and lakes pancreatogram is taking narcotics around the clock. What do you want to do?” Could be presented with any of the complications of Surgical Treatment chronic pancreatitis. Be careful to rule out other complica- tions of pancreatitis (ascites, pseudocyst, acute pancreati- (1) Nonoperative therapy tis) before diving into discussion of the management of (a) Control abd pain: abstinence from alcohol, chronic pancreatitis. dietary manipulation (low fat, small volume meals, non-narcotic analgesics 1st ....often failure of this is indication for surgery How To Answer? (b) Tx for endocrine insufficiency: exogenous insulin carefully,’ (hypoglycemia can arise as History result of poor nutrient absorption) Abdominal pain, epigastric, radiation to back, continu- (c) Tx for exocrine insufficiency: low fat diet, ous or relapsing, exogenous pancreatic enzymes, Anorexia If medical therapy fails (which of course it always will Weight loss on the Oral Exam!): IDDM in 1/3 pt Steatorrhea in 1/4 pt. (2) What is size of pancreatic duct → Classic Tetrad: abd pain, wt loss, DM, steatorrhea (a) large Peustow procedure Narcotic use Peustow/Gillesby 1958—side to side pancreatico- Flares of pancreatitis jejunostomy, success rates 60–90%, decom- Etiology of pancreatitis presses entire duct, need duct greater than 7 mm in diameter, pancreatic calcifications, pan- creatic-jejunal anastomosis longer than 6 cm, Physical Exam does not affect endocrine/exocrine insuffi- Palpable mass (pseudocyst) ciency 84 Part 2.qxd 10/19/05 2:52 AM Page 85

Strikeouts 85

Chevron incision, divide gastrocolic ligament to Strikeouts enter lesser sac, expose entire anterior surface of pancreas, create Roux-en-Y and anastomose Not getting ERCP/CT scan to entire pancreatic duct Not knowing what operation to offer for “large/small” → (b) small pancreatic resection pancreatic duct Pylorus—preserving Not knowing complications of chronic pancreatitis for pt with chronic pancreatitis, no ductal Not being able to describe the Peustow procedure dilatation, and disease primarily in head of Offering pt total pancreatectomy or 95% pancreatec- gland, preserves endocrine function in tomy in favor of the more standard options body/tail (3) Ampullary stenosis→ (a) ampullary procedures: transduodenal sphinc- teroplasty helpful if focal obstruction at ampullary orifice, in pt with pancreatic divi- sum and stenosis of minor pancreatic duct papilla (these procedures have generally fallen out of favor) (4) Celiac block considered for pts who fail operative interventions

Common Curveballs

Pt will present with complication of chronic pancreatitis: Pain Pseudoaneurysm Splenic vein thrombosis Obstruction (GI or biliary tract) Exocrine/endocrine deficiency Pseudocyst Anastomotic leak after Peustow procedure Asked to describe Peustow procedure Pancreatic duct will be “large” initially, then asked to comment on surgery for “small” duct Pt will develop hepatic failure post-op (alcoholic liver disease) Pt will have DTs post-op Part 2.qxd 10/19/05 2:52 AM Page 86

Pancreas—Pancreatic Cancer

Concept look for metastases, enlarged lymph nodes → don’t do percutaneous bx of possibly 90% of pts will be unresectable with mean survival of 3 resectable tumor risk dissemination along months. Key is to determine who is a candidate for resec- tract tion. You will likely be given a pt you will need to explore ERCP—obtain biopsy/brushings/cytology to determine resectability. Stent pt only if severely jaundiced, unrelenting itching or abnormal L LFTs (especially coags) Angiogram with venous phase—look for encasement of Way Question May be Asked? SMA, SMV, portal vein and r/o replaced right hepatic artery “61 y/o male comes to your office with recent weight loss Endoscopic U/S—not necessary but can help stage and a CT scan ordered by his family doctor shows a mass tumor and assess resectability in the head of the pancreas. What do you want to do?” May present as just weight loss, obstructive jaundice, or DDx examiners may be direct and get right into the thick of it. Remember other causes of obstructive jaundice if this is what you are presented with: How to Answer? Stricture Stone Be methodical! Extrinsic compression Malignancy (duodenal, ampullary, cholangio) History Surgical Treatment Smoking Anorexia (1) Can laparoscope pt before you open to look for Alcohol use peritoneal implants Weight loss (a) if find, then do biliary and gastric bypass Back pain/abdominal pain (classic is painless jaundice) laparoscopically (2) Chevron incision Physical Exam (3) full abdominal exploration and evaluate for respectability (check for hepatic mets, lymph node Mass in RUQ (liver or distended gallbladder) mets outside of resection zone and liberal use of frozen section) Data Clockwise Resection Full laboratory panel including LFTs, Albumin, CA19- 9, CEA (4) Cattell–Braasch maneuver Routine pre-op studies (EKG, CXR) ligate Middle colic vein CT scan—with thin section through pancreas exposing SMV

86 Part 2.qxd 10/19/05 2:52 AM Page 87

Strikeouts 87

(5) Extended Kocher maneuver Can’t determine it is malignancy even with intra-op ligate right gonadal vein biopsies→“will you do a Whipple?” (6) Portal dissection Complications of Whipple ligate gastroduodenal Leak at any of the anastomoses dissect out gallbladder Abscess Transect CHD just proximal to cystic duct Delayed gastric emptying (careful, a hepatic artery can course posterior to Marginal ulcer portal vein) Pancreatic fistula (7) Transect stomach Bile leak at level of third/fourth transverse vein on lesser Intra-op injury to middle colic vein curve and confluence of gastroepiploic veins on Peritoneal implants and asked what type of bypass greater curve operation you will perform +/− pylorus preserving Tumor may be in tail of pancreas (→distal pancreatec- (8) Transect jejunum tomy) 10 cm distal to ligament of Treitz May present as acute pancreatitis (change scenario) (9) Transect pancreas at level of portal vein How to determine resectability? if adherent, proximal and distal control and Pt will be malnourished and asked to discuss TPN resect anterior wall and repair with vein patch Asked when will you place biliary stent pre-op? frozen section to check pancreatic/biliary margins (10) Strikeouts Counter-Clockwise Reconstruction Not performing adequate staging work-up to r/o unre- (11) End to side pancreaticojejunostomy sectable disease 2 layers over a stent Not knowing how to describe Whipple operation (12) End to side choledochojejunostomy Not knowing how to describe bypass operation (13) End to side gastrojejunostomy antecolic in two Performing percutaneous biopsy of pancreatic mass in layers potentially resectable lesions (14) Gastrostomy Performing total pancreatectomy (15) Jejunostomy (16) Lots of drains

Common Curveballs

Replaced right hepatic artery—what is its course? Tumor invading portal vein (discovered during course of operation) Part 2.qxd 10/19/05 2:52 AM Page 88

Pancreas—Pancreatic Pseudocyst

Concept Differentiate pseudocysts based on size and symptoms. Non-symptomatic less than 4 cm in size should be fol- A walled-off collection of pancreatic enzymes and inflam- lowed by serial U/S/CT scans—can continue to follow matory fluid typically in the lesser sac (with the bound- as long as decreasing in size. aries formed by the lesser sac) or within the pancreas itself Be prepared for pseudocyst to: rupture, obstruct, bleed, that is bounded by a nonepithelialized wall of fibrotic tis- get secondarily infected, or increase in size. sue. Can develop symptoms related to size (obstruction, Cysts with duration greater than 6 weeks, enlarging on CT, pain) or erosion into other structures (bleeding). greater than 4 cm, or are associated with chronic pancreati- tis are unlikely to resolve without operative intervention Way Question May be Asked? 6 cm or greater pseudocysts or symptomatic ones should undergo interventional treatment: “A 48 y/o male with history of alcohol abuse and pancreati- tis presents to ER with abdominal pain and work-up reveals TREATMENT a 4 cm pancreatic pseudocyst.” Question could be abdomi- (1) ERCP to see if communicates; nal pain in a patient with a history of pancreatitis, or could (a) if doesn’t, can consider CT aspiration (~40% be given the formation of a pseudocyst after an initial bout success rate) or leaving a catheter in cyst cavity of pancreatitis with the patient still in the hospital. (these options will fail or the cyst will get secondarily infected) (b) if does, and symptomatic or larger than 4 How to Answer? cm→surgical drainage Complete H+P (2) Choices for internal drainage (remember cyst wall Weight loss takes about 6 weeks to mature): Vomiting (a) cystgastrostomy (anterior gastrostomy, palpa- Abdominal mass tion and needle aspiration to find cyst in back Trauma wall of stomach and then open cyst, send part Alcoholism of wall for bx, and suture posterior wall of Bouts of acute or chronic pancreatitis stomach to mature cyst wall, opening should be Palpable mass 5 cm, use interrupted absorbable sutures) (b) cystojejunostomy (use Roux loop when cyst is not adherent to posterior wall of stomach (can check Diagnostic Tests by opening gastrocolic omentum and seeing if Appropriate laboratory tests (amylase, WBC) there is plane b/w posterior stomach and cyst) or U/S good for screening multiple cysts (using side-to-side anastomosis) CT gold standard (c) cystoduodenostomy (if in head of pancreas close to duodenum, Kocher maneuver to check, 3 cm Be complete, but don’t dwell on these as the examiner is opening into first or third portion of the duode- trying to get to your management here: num, transduodenal approach)

88 Part 2.qxd 10/19/05 2:52 AM Page 89

Strikeouts 89

(d) distal pancreatectomy (option if pseudocyst in for DDx; angiogram to embolize bleeding vessel, pancreatic tail or has eroded into surrounding otherwise ex lap, ligation splenic or gastroduodenal, structures) then open and pack cyst and ligate bleeders within cyst wall) (3) External drainage only for The pseudocyst will get infected if you try to aspirate it (a) infected pseudocysts If you leave a drainage catheter for non-communicating (b) unstable pt with free rupture or bleeding cyst, the pt will get a pancreatic fistula (4) bleeding in pt with a pseudocyst can be from: GI doctor will not be available for ERCP or endoscopic (a) bleeding into the bowel from erosion into bowel cystgastrostomy or endoscopic cystgastrostomy will wall result in free perforation/bleeding at the anastomotic (b) bleeding from gastric varices (splenectomy site treatment of choice here as varices form Biopsy of the wall will reveal malignancy secondary to splenic vein thrombosis) Pt will have pancreatitis flare after ERCP (c) bleeding into the cyst (erosion into one of the Pseudocyst will actually be cystic neoplasm by intra-op pancreatic vessels) frozen section biopsy (change of scenario) (d) bleeding from a ruptured pseudoaneurysm Pt that you decide to follow will develop complication (usually splenic artery) from pseudocyst such as: (e) bleeding may occur into cyst, into bowel, or free cyst rupture—pancreatic ascites into peritoneal cavity infection (fever, inc. WBC, inc. abd pain→open surgical drainage) Angiogram helpful here if pt stable enough, bleeding (hemorrhagic shock) otherwise, laparotomy, ligate offending vessels, duodenal obstruction open cyst, pack and return or go to angiogram pseudoaneurysm as necessary splenic vein thrombosis

Common Curveballs Strikeouts

There will be multiple cysts Forgetting to biopsy the wall of the pseudocyst Try to get you to change your management strategy so Not waiting for wall to mature the size may change during the questioning from 4 Not obtaining a CT scan cm to 8 cm Not knowing how to perform internal drainage proce- Try to get you to operate before a mature wall has formed dure Pseudocyst will rupture into thoracic cavity (pancreatic Not getting ERCP pre-op to determine if communi- hydrothorax) cates with pancreatic duct Your first choice of internal drainage will not be an Taking pt with bleeding from pseudocyst to OR rather option (prior surgery,...) than angiogram to embolize offending vessel The pt with known pseudocyst will develop bleeding While has been performed by several authors, don’t into the pseudocyst and present in shock (see above mention laparoscopic cystogastrostomies Part 2.qxd 10/19/05 2:52 AM Page 90

Pediatric Surgery—Neonatal Bowel Obstruction

Concept Diagnostic Tests Bilious vomiting is always a surgical emergency in the “Babygram”—look for pattern of the gas newborn. Multiple possible etiologies: “double bubble”—duodenal atresia or malrotation with volvulus Annular pancreas dilated SB loops—jejunoileal atresia Duodenal web UGI if suspect proximal obstruction or malrotation Malrotation BE if suspect distal obstruction Jejunoileal atresia Meconium ileus Hirschprung’s Surgical Treatment Infection/NEC ClϪ (1) NPO/IVF/NGT/Correct electrolytes Metabolic abnormalities (K+,Mg++) (2) determine if obstruction is proximal or distal Always look for associated anomalies such as cystic (3) OR for any evidence peritonitis fibrosis (meconium ileus) and Trisomy 21 (duodenal atre- (4) Duodenal Atresia→OR once resuscitated sia, malrotation). (a) duodenojejunostomy through transverse RUQ incision (b) obstruction usually immediately post- Way Question May be Asked? ampullary (c) G-tube “Called to NICU to evaluate a baby that has had bilious (5) Malrotation (often associated with diaphragmatic vomiting since birth. What do you want to do?” Always hernia, abdominal wall defects, and jejunoileal look for congenital anomalies, Down’s stigmata, and atresia) remember that this is a surgical emergency. (a) counterclockwise detorsion if volvulus present (b) second look for questionable viability How to Answer? (c) Ladd’s procedure dividing peritoneal bands crossing duodenum History (extend from ligament of Trietz) Maternal polyhydramnios positioning duodenum and jejunum to right of Onset of bilious emesis (w/every feeding) midline Delayed meconium passage positioning colon to left of midline Prematurity incidental appendectomy Family history (d) treat other anomalies if present (e) cecopexy/duodenopexy not necessary Physical Exam (6) Jejunoileal atresia (the more distal the obstruction, the more abdomi- Evidence dehydration (sunken fontanelle, skin turgor) nal distension child will have) Abdominal distension or scaphoid abdomen (a) BE to document normal colon Any congenital anomalies (perforate anus?) (b) resect atretic portion 90 Part 2.qxd 10/19/05 2:52 AM Page 91

Strikeouts 91

(c) inject saline to make sure no distal obstruction Common Curveballs (web/atresia) (d) end to end anastomosis Double bubble on x-ray will be malrotation and not (7) Duodenal web duodenal obstruction (a) longitudinal duodenotomy Scenario will change from proximal to distal obstruc- (b) partial membrane excision tion (8) Meconium ileus—failure to pass meconium < 24 h Pt will have multiple atretic areas in jejunum/ileum with bilious emesis, abdominal distension, perforate Pt will have appearance of total small bowel infarction anus Pt will have associated anomalies (only cardiac affects (produces obstruction from inspissated meconium your decision to operate) secondary to pancreatic exocrine insufficiency) (a) evaluate for Cystic Fibrosis (b) ground glass appearance on AXR instead of Strikeouts A/F levels (c) gastrografin enema, pancreatic enzymes Not identifying malrotation by NGT, and mucomyst for uncomplicated Not knowing Ladd’s bands or details of Ladd’s procedure presentation Not knowing what “double bubble” means on “baby- → (d) complicated meconium ileus OR gram” resect nonviable bowel Not looking for associated anomalies repair perforations Not treating bilious vomiting as surgical emergency drain any abscesses enterotomy + injection mucomyst Part 2.qxd 10/19/05 2:52 AM Page 92

Pediatric Surgery—Pyloric Stenosis

Concept U/S

Thickening of the muscle of the pylorus resulting in func- Elongated pyloric channel tional outlet obstruction. Most common surgical cause of Thickened pyloric diameter emesis in infants. Unknown etiology. Increased pyloric wall thickness Barium UGI Elongated narrow pyloric channel (“string sign”) Way Question May be Asked? Gastric outlet obstruction

“Called to ED to evaluate a 9 week old infant with a his- Surgical Treatment tory of intermittent non-bilious emesis that is now projec- tile vomiting. What do you want to do?” May be given an (1) Correct electrolyte abnormalities (this is an elective infant with clear signs of dehydration and may be an older surgical procedure) infant (up to 2 years old). Key is whether or not the vom- (2) D51/2NS iting was bilious. (3) Pyloromyotomy (Ramstedt technique) general anesthesia transverse epigastric or RUQ incision How to Answer? grasp pylorus between two fingers incision with scalpel into serosa/muscle History back of scalpel handle to blunt complete pyloromy- FHx of pyloric stenosis otomy Bilious vs. non-bilious vomiting should see bulging mucosa Vomiting of undigested formula shortly after feeding careful not to perforate underlying mucosa Intermittent emesis progressing to projectile (if perforate, close and cover with omental patch, or Infant hungry between episodes of vomiting close myotomy and rotate pylorus 45˚ and per- form pyloromyotomy again) (4) Can start feeding 6–12 h post-op with dilute milk Physical Exam and advance as tolerated Sunken fontanelle (5) Small episodes of emesis not uncommon in imme- Dry mucous membranes diate post-op period, pursue with UGI for incom- Decreased skin turgor plete pyloromyotomy if extends past POD#2. Abdominal exam: Thickened pylorus or “olive” in epigastrum Common Curveballs (need infant to be quiet and stomach empty) Observation of gastric peristaltic waves Pt will have low K+ and asked how you will manage Being asked when you will start feeding the child Diagnostic Tests Mucosal perforation during your pyloromyotomy, now what? Full labs especially K+ (hypokalemic, hypochloremic Scenario will change and pt will have malrotation, metabolic alkalosis) antral web, or duodenal stenosis 92 Part 2.qxd 10/19/05 2:52 AM Page 93

Strikeouts 93

Incomplete pyloromyotomy Strikeouts Asked to describe Paradoxic Aciduria Mistaking diagnosis for one of the many etiologies for + + (vomiting leads to loss of fluid with high K ,H , and neonatal bowel obstruction Cl-concentrations. Volume deficit leads to aldos- Not being able to describe how to resuscitate the pt pre- + + terone mediated Na resorption with loss of K op + and body tries to hold onto K leading to excretion Describing laparoscopic pyloromyotomy + → of H ions leading to paradoxic aciduria) treat by Not being able to explain the hypokalemic, hypochloremic + replacing volume before administering K ! metabolic alkalosis that typically accompanies these pts Being asked how to calculate volume of fluid to be administered to the infant and given a weight in kg Part 2.qxd 10/19/05 2:52 AM Page 94

Pediatric Surgery—Tracheoesophageal Fistula

Concept Imperforate anus or limb abnormalities Cardiac exam Common cause of respiratory distress in the infant. Several variants: Diagnostic Tests (A) esophageal atresia and distal TEF (most common) “Babygram” (B) atresia without fistula air in GI tract rules out atresia without TEF (C) H-type TEF r/o duodenal atresia, vertebral anomalies (D) atresia with proximal and distal TEF 0.5 cc barium down NGT (identify blunt pouch) Approximately 50% of infants will have other congenital Pre-op echo to r/o cardiac anomalies (affects anesthesia defects (VACTERL—vertebral, anorectal, cardiovascular, management) TEF, renal, limb) and you need to rule these in/out prior Renal U/S before or after repair to operation. Surgical Treatment Way Question May be Asked? (1) NGT in pouch (2) Elevate head of bed “Called to NICU to evaluate a newborn who is small for (3) abx if pneumonia his gestational age that had an episode of choking and (4) OR in first 24 h for repair: desaturation with his first two feedings. What do you want (a) extrapleural approach/right thoracotomy to do?” Should consider any newborn with respiratory dis- (b) divide fistula tress. Key is that problems are associated with feeding. (c) close trachea Most should be identified preterm by U/S. (d) end to end esophageal anastomosis (e) gastrotomy for early post-op feeding (f) leave drain next to esophageal anastomosis How to Answer? (g) alternative is gastrotomy only and spit fistula in neck and delay repair until one year old History (colon interposition→usually in cases of atresia Maternal polyhydramnios without TEF) Respiratory distress with first feeding (choking, cough- ing, regurgitation) Desaturation with eating Common Curveballs

Physical Exam Post-op complication: Leak Can’t place NGT Stricture Small for gestational age infant Recurrent fistula Scaphoid abdomen (if atresia without TEF) Reflux

94 Part 2.qxd 10/19/05 2:52 AM Page 95

Strikeouts 95

Will enter pleura during extrapleural approach Strikeouts Will be associated anomalies (Down syndrome, valvu- lar defect, etc.) Not making the diagnosis Won’t be able to perform primary end to end anasto- Not knowing most common type/how to repair most mosis because of long “gap” common type Pt will present with H-type fistula (repeated episodes of Not ruling other associated anomalies pre-op (espe- pneumonia in infancy) cially cardiac!) Scenario will switch to management of associated Not trying to place NGT (alternatively, continuing to imperforate anus try to advance when meet resistance) Not placing G-tube at operation Part 2.qxd 10/19/05 2:52 AM Page 96

Perioperative Care—Hypotension in the Recovery Room

Concept Circulation Easy to get lost in the myriad of possible diagnosis. The Pulses key is to be methodical and stepwise. Approach patient Cold extremities (hypovolemic shock) vs. warm like a trauma patient working through your ABCs. DDx (anaphylactic) includes any form of shock: hypovolemic shock (inade- quate fluids intra-op), hemorrhagic shock (pt still bleed- “AMPLE” History ing), cardiogenic shock (MI, pneumothorax), also sepsis (unlikely so quick), transfusion reaction, malignant hyper- Type of procedure thermia, Addisonian crisis, air/fat embolism. Length of surgery Fluids/blood Previous PMHx Way Question May be Asked? CVP placed intra-op?

“You are called to evaluate a 63 y/o male status post a Physical Exam AAA repair who was stable in the recovery room for about 2 h and now his BP has dropped to 80/40. What do you Vital signs (fever very suggestive) want to do?” Question could be asked in many different Neck veins (flat or distended) ways with the patient status post any major abdominal Heart rate (arrythmia) operation, had received blood intra-op, may or may not be Rash (petechiae with txn reaction) given other vital signs at the start. Generalized oozing (DIC) Pulses in extremities Abdominal exam How to Answer? Surgical Treatment Be methodical ABCs while resuscitating the pt (1) Order: CXR, EKG, ABG, complete laboratory panel, U/A Airway Send pt’s blood along with transfused bags if sus- pect txn reaction (also check urine for Hgb) Is pt on ventilator? (2) CVP or SGC to direct fluid management What is RR and Pulse ox? (3) Treat specific underlying problem Does pt need to be intubated? (a) for hypovolemic shock, fluid resuscitation (b) for cardiogenic shock, SGC plus pressors Breathing (c) for pneumothorax, chest tube (d) for malignant hyperthermia, cooling, support- Are both lung sounds present? ive care, dantrolene Does patient need chest tube? (e) for Addisonian crisis, bolus steroids (100 mg hydrocortisone)

96 Part 2.qxd 10/19/05 2:52 AM Page 97

Strikeouts 97

(f) air/fat embolism, supportive care Pt will need to be intubated (g) transfusion reaction (key is to keep up urine Pt will need CVP/SGC output and avoid precipitation of Hgb in renal You’ll be given a set of SGC parameters to interpret tubules) Pt will develop renal failure (change scenario) fluids to maintain UO 100 cc/hr 2 amps bicarb plus add to IVF to alkalinize urine (check pH > 7) Strikeouts mannitol (1–2 mg/kg) (osmotic diuretic) Taking pt back to OR (usually, they are not trying to get you to take pt back to OR, Common Curveballs Not being methodical and going through ABCs You’ll miss a mucus plug) Pt will have MI and you’ll be asked your manage- Miss a pneumothorax or tamponade from CVP line ment/pressors anesthesiologist placed Pt will have refractory hypotension to anything you do Miss a kinked ETT Pt will have txn reaction and you’ll be asked your spe- Missing vital signs (fever and hypotension point you in cific management including how to alkalinize the some specific directions) urine Part 2.qxd 10/19/05 2:52 AM Page 98

Perioperative Care—Postoperative Fever

Concept Immunosuppression (txp pt, HIV+, chemotherapy, steroid use?) Multiple causes, but should be systematic. Remember the number of days post-op and the most common causes. Physical Exam Never forget to check the wound for a necrotizing soft tis- sue infection or change the CVP for possible line sepsis. Vital signs (shock?) DDx: Complete physical exam Days 0–2: atelectasis, necrotizing soft tissue infection Wound Days 3–5: UTI, pneumonia IV sites Days 5–7: wound infection/abscess Foley? Day 7–10: DVT, anastomotic leak, C. diff Immediate post-op: Addisonian crisis, thyrotoxicosis Diagnostic Tests Anytime: line sepsis, drug fever, transfusion reaction (soon after txn pRBC, platelets, FFP, cryo) Complete labs (including CBC w/diff, LFT’s, amylase, U/A) Culture and Gm stain any wound drainage Way Question May be Asked? Low threshold to open up any wounds Sputum Gm stain/culture “You are called to see a pt 3 days s/p a right hemicolec- Blood cultures (useful only if your pt is still alive 48 h tomy for an adenomatous polyp who is febrile to 101.4. later when the results return!) What do you want to do?” Question may be after any CXR operation and any number of days post-op. Key is to +/− CT scan (to r/o abscess) resuscitate the unstable pt in the ICU, perform a complete +/− Duplex U/S of lower extremities exam with attention to the wound and IV sites, to get +/− thyroid hormone levels appropriate diagnostic tests, and then be aggressive with +/− stool for C. diff toxin assay the management when appropriate. Common scenarios will include necrotizing soft tissue infection, anastomotic Surgical Treatment leak, enterocutaneous fistula, and intraabdominal abscess. (1) Low threshold to open wound (2) Low threshold to change CVP and send tip for Cx How to Answer? (3) Low threshold to transfer unstable pt to ICU History (4) For necrotizing wound infection (a) open wound Type of surgery (how dirty was the case?) (b) gm stain, C+S Antibiotic use (c) take immediately to OR Recent transfusions (d) wide debridement to viable tissue Associated symptoms (cough, chills, rigors, pain, (e) close with VAC sponge, Bogata bag, cadaveric dysuria, diarrhea) skin, or pack IV sites (how old ?) Diarrhea 98 Part 2.qxd 10/19/05 2:52 AM Page 99

Strikeouts 99

(f ) return to OR following day to repeat debride- Being asked your antibiotic selection and why ment and then daily until only viable tissue Scenario will change on you from POD #2 to #5 to #10 remains Pt will have anastomotic leak (g) treat with PCN G or broad spectrum abx Pt will have fistula based on Gm stain, Cx’s Pt will have intraabdominal abscess (h) typical organisms: Clostridia, Strep, and mixed Pt will have transfusion reaction infections Pt will have Addisonian Crisis (5) For retained foreign body Pt will have thyrotoxicosis (a) need AXR/CXR +/− CT scan to identify Pt will have DVT (b) promptly take to OR (c) may be asked how you will describe situation to pt and family (honesty is the best policy!) Strikeouts

Getting lost in zebras like paraneoplastic syndromes, Common Curveballs SBE, drug fever, parotitis, otitis Pursuing a work-up for sinusitis and acalculous chole- Pt will have necrotizing wound infection and you’ll be cystitis in any pt other than the debilitated ICU pt asked how to manage Not considering line sepsis Pt will have retained foreign body left behind from sur- Not evaluating wound gery Not recognizing and appropriately treating necrotizing How will you discuss with pt/family a retained foreign wound infection body Waiting for blood culture results before making any Pt will have necrotizing infection and asked how to definitive management decisions (usually are waste treat/close wound of money as results take >24 hrs.) Pt will have C. diff enterocolitis and need emergent abdominal surgery Part 2.qxd 10/19/05 2:52 AM Page 100

Perioperative Care—Recent MI

Concept Pre-op SGC and NTG gtt and maximize hemodynamics with invasive monitoring Many patients will have elevated operative risk given car- Pt with conduction system disease may require tempo- diac history. Risk factors include HTN, DM, angina, vas- rary pacemaker support during surgery cular disease, + family history. ~10% perioperative mortality with: (a) recent MI (risk, ~30% if less than 30 days, 6% if Way Question May be Asked? less than 3 months, 2 % if 3–6 months) (b) decompensated heart failure (c) unstable angina Any common general surgery issue, with the patient hav- (d) severe valvular disease (less than 0.9 cm2 for aor- ing had a recent MI. tic valve and 1.5 cm2 for mitral valve) “56 y/o male, heavy smoker, with recent MI, now pres- ents with signs and symptoms c/w acute cholecystitis.” The examiners may actually throw the scenario at you where the patient has multiple problems, like obstructing or Common Curveballs bleeding rectal cancer, and will have had a recent MI. Intraoperative ischemia Intraoperative arrhythmias How to Answer? Post-op ischemia Post-op arrhythmias (a. fib particularly popular as a complication in any scenario whether recent MI or Goldman criteria not—don’t forget your ACLS!) aortic stenosis, MI within 6 months, emergency surgery, Post-op pulmonary edema nonsinus rhythm, age > 70 years, JVD, poor medical How will you manage pt pre-op condition (PO2 less than 60, CR > 3.0, chronic liver dx Emergency operations performed without cardiac prepa- ration have an up to 5% perioperative risk of MI Clean Kills No type of anesthesia (local, epidural, or general) is Not knowing any of Goldman criteria (don’t need to better than any other when administered by good know all of them) anesthesiologist Not adequately working up pt pre-op Preoperative work-up as best as possible to determine Forgetting about intra-op monitoring cardiac status (EKG, CXR, ECHO—ejection frac- Believing one type of anesthesia superior to another— tion) risk itself is just anesthesia so do the surgery you If find reversible defect on stress thallium→ cardiac need to do cath Not appropriately dealing with post-op complications If find a lesion, have bypass performed

100 Part 2.qxd 10/19/05 2:52 AM Page 101

Perioperative Care—Renal Failure

Concept How to Answer?

Multiple causes, but can be broken down into prerenal, History intrarenal, and postrenal causes. Certain information on I/Os H+P and your diagnostic tests will help you here. Intra-op fluids Clamp time on AAA (supra or infrarenal) Prerenal History of renal disease Nephrotoxic meds Hypovolemic shock Diuretic use Hemorrhagic shock Recent transfusions (hemolysis with precipitation in Septic shock renal tubules) Third space losses (burns, pancreatitis, long operation, Trauma with major muscle injury (myoglobinuria) cirrhosis) Vascular (emboli, renal artery occlusion) Physical Exam Abdominal compartment syndrome Pump failure Vital signs (shock?) Skin (turgor?) Intrarenal Mucous membranes Chest (CHF?) ATN—from ischemia—secondary to inadequate perfu- Abdomen (distended bladder?) sion (from prerenal cause above) Check Foley (is one in place? has it been flushed?) Acute interstitial nephritis—secondary to medication Bladder pressures (compartment syndrome)

Postrenal Diagnostic Tests Urethral obstruction (catheter/prostate) Complete labs Bilateral ureteral obstruction (intra-op injury, retroperi- BUN/Cr ratio toneal fibrosis) U/A (protein with glomerular disease, eosinophils with interstitial nephritis) Always want to convert oliguric renal failure into non- Urinary electrolytes (urinary Na < 20 suggests prerenal oliguric. Most often cause will be hypovolemia in etiology) surgical patients. U/S to evaluate kidneys Obstruction Way Question May be Asked? Confirm two kidneys CVP or SGC to determine volume status “You are called to see a patient 6 h s/p AAA repair whose urine +/− IVP to evaluate kidney function output has been 15 cc the past 3 h. What do you want to do?” Confirm no postrenal obstruction Question may be after any operation or in the management of (careful—die load—use non-nephrotoxic contrast any patient for example, s/p multiple trauma, burns, APR. agents)

101 Part 2.qxd 10/19/05 2:52 AM Page 102

102 Perioperative Care—Renal Failure

+/− Renal Scan—MAG3 scan, useful to assess kidney Patient will have myoglobinuria and asked your man- perfusion agement (alkalinize urine) Patient will have transfusion reaction and development Surgical Treatment renal failure and you’ll be asked how to manage Patient will have only one kidney (1) Low threshold for CVP, txfr to ICU, SGC Patient won’t respond to fluid boluses (2) D/C any nephrotoxic drugs and supplemental K+ Patient will be elderly with brittle heart/prone to (3) Go through your DDx for renal failure CHF/low EF (4) Fluid resuscitation and monitor hourly UO Won’t be able to place foley catheter (5) +/− Dopamine at renal doses Patient will have hematuria (6) Lasix to convert to non-oliguric renal failure (7) +/− Mannitol (8) Monitor electrolytes closely Strikeouts (9) Consider dialysis Not breaking DDx down into prerenal, intrarenal, and postrenal causes Common Curveballs Not placing at least CVP after two fluid boluses with- out a response Indications for dialysis Not being aggressive with resuscitation of patient Asked to describe how to measure bladder pressures? Not identifying abdominal compartment syndrome Their significance? Performing angiogram acutely Asked how to treat abdominal compartment syndrome Performing renal biopsy acutely Patient will be unresponsive to all resuscitative meas- ures Part 2.qxd 10/19/05 2:52 AM Page 103

Skin and Soft Tissue—Melanoma

Concept Physical Exam Four main subtypes—superficial spreading, nodular, Examination of lesion (color, size, symmetry) lentigo maligna, and acral lentigenous—with several spe- Examination of regional lymph node basins cial situations (anal, subungal). Work-up should be sys- tematic—establish risk factors, biopsy, stage the patient, Consider the Differential sample lymph nodes, and then determine any adjuvant treatment. Expect questions regarding lymph node sam- Benign nevus pling, especially groin dissections. Seborrheic keratosis Pigmented wart Squamous cell cancer Way Question May be Asked? Basal cell cancer

“52 y/o male presents to your office with a skin lesion on Biopsy Lesion his leg (or arm, or abdomen, or back) that has recently changed in both color and size.” You may also be shown a If not in cosmetically sensitive place, excise with 1–2 picture of an obvious melanoma, or get the patient sent to mm margin you after a biopsy performed by a dermatologist. If large, punch bx through thickest portion of lesion or incisional bx Always orient the specimen How to Answer? If subungal, split open nail—only need diagnosis here History Staging the disease (Clark’s system has really Changing skin lesion (A,B,C,D,E mnemonic) Bleeding lesion fallen out of favor—mostly based on Breslow Ulceration depth) Itching TNM system: I primary < 1.5 mm depth, no nodes II > 1.5 mm depth, no nodes Establish Risk Factors for Melanoma III regional nodal disease or Excessive sun exposure in-transit mets Fair skin IV distant metastases + FHx Histological staging (Breslow) Hx of melanoma Thin 0–0.75 mm Dysplastic nevus syndrome Intermediate 0.76 to 4 mm Xeroderma pigmentosum Thick > 4 mm (80% chance of mets)

103 Part 2.qxd 10/19/05 2:52 AM Page 104

104 Skin and Soft Tissue—Melanoma

Diagnostic Studies mic limb perfusion with melphalan and TNF has received a lot of attention recently CXR Treatment of Stage IV disease CBC, LFT, LDH Isolated mets (liver, lung, brain) should be resected FNA any palpable nodes assuming no other evidence of disease If palpable nodes: CT scan to evaluate nodal and next nodal basin CT liver and brain Common Curveballs Margins of resection (will likely need to re-excise after biopsy) Melanoma won’t be on extremity but on trunk and pre- 5 mm for in-situ lesions op lymphoscintigraphy will light up several nodal 1 cm for < 1 mm depth basins 2 cm for > 1 mm depth Lymph nodes in groin will be clinically palpable head/neck—twice the diameter of the lesion Sentinel node biopsy won’t work subungal finger—split nail to biopsy, amputate distal Will be other melanomas if don’t do complete skin phalanx (elective node dissection if > 0.75 mm) survey subungal toe—ray amputation Expect pulmonary/brain metastases during first several ear— full thickness wedge resection twice the diam- years of follow-up of your pt eter of the lesion Depth will be 0.74 or 0.77 mm anal—local excision, APR only if pt is incontinent Pathology may turn out to be squamous cell or basal or has severe pain from invasion of the sphincters cell anterior to ear—re-excision + modified radical neck There will be in-transit disease dissection and superficial parotidectomy The pt will have two melanomas Microscopic disease in Cloquet’s node—will you do Lymph Node Sampling deep inguinal dissection? Management of subungal/anal melanoma SLN bx should be offered to all pts with extremity and Decline in pulse ox reading during operative procedure truncal primaries greater than 1 mm in depth (except (typical artificial side effect of blue dye) subungal) Pt. may have allergic reaction to blue dye Get pre-op lymphoscintigraphy Use combo of handheld gamma counter and Lymphazurin blue dye Strikeouts Send sentinel node for frozen section Complete node dissection if sentinel node positive Not being able to justify your reasoning on doing or not Only do deep node dissection in groin if have gross doing a deep inguinal node dissection when appro- disease in apical nodes (sapheno-femoral/ priate Cloquet’s node) or CT shows suspicious iliac Performing a shave biopsy or FNA of a suspected adenopathy melanoma →Not if only microscopic disease in superficial Not performing physical exam to lymph node basins nodes or CT suggests nodes that are positive to Not knowing difference between Clark’s levels and level of aortic bifurcation as unlikely to have any Breslow depth therapeutic benefit and downside of severe leg Not knowing re-excision margins for different depth edema! melanomas Prophylactic node dissection if: Not orienting the specimen for the pathologist b/w 1–4 mm and lesion overlies the primary nodal Not getting CT to evaluate next echelon of nodes with basin (parotid, inguinal, axillary) palpable nodes clinically lesion > 1 mm in head or neck No SLN bx in head and neck, subungal and anal Adjuvant therapy melanomas! Stage II melanomas deeper than 4 mm or Stage III Trying to perform SLN bx when lesion overlies a lymph disease get offered vaccine or high-dose interferon node basin Trying to offer chemotherapy to pts with isolated metastases Treatment of in transit/recurrent disease Discussing vaccine therapy (experimental) Re-excision, local radiation, and isolated hyperther- Part 2.qxd 10/19/05 2:52 AM Page 105

Skin and Soft Tissue—Sarcoma

Concept Biopsy Lesion Pathologic type is not as important as size, grade, location, If less than 3 cm, may excise, but don’t shell out due to and resection margins. Lymph node involvement is rare tumor pseudo encapsulation→ aim for 2 cm margin and therefore, lymph node dissections are only done if If larger, incisional biopsy parallel to the muscle group grossly involved. Chemotherapy is controversial and mar- (won’t compromise future resection) ginally beneficial. Core needle biopsy is acceptable but tattoo site of bx for later excision Need to ask pathologist: histologic grade Way Question May be Asked? Treatment “39 y/o man presents to the office with a growing mass on his right anterior thigh. On exam, it is hard and fixed to Surgical excision for grossly clear margin the underlying tissues. What do you do?” May be in the In extremity: extremity of a female after axillary dissection (Stewart- 2 cm margin, remove entire muscle group only if nec- Treeves syndrome) or in patient with history of radiation. essary Mark the excision site for adjuvant XRT that may reduce incidence of local recurrence How to Answer? Can leave microscopic disease if this preserves vital neurovascular structures as post-op XRT will Brief History clean up residual disease Trauma Extremity arteries are expendable and can be Radiation replaced with vein or conduit Café au lait spots (von Recklinghausen pts) Femoral nerve can be sacrificed, but not sciatic (gen- History of prior lymphadenectomy erally, can sacrifice sensory nerves, but try to pre- serve motor nerves) Physical Exam Removing large central extremity veins leaves pt with severe edema Examination of tumor Amputative procedures only for joint involvement Lymph nodes (hip, knee, elbow, shoulder, pelvis) Neurovascular deficit in the affected extremity For small-cell sarcomas (Ewing’s), can consider neoadjuvant chemo/XRT to cytoreduce tumors to Studies allow for limb salvage or salvage of vital neurovascular structures (sciatic nerve) CXR In retroperitoneum: MRI/CT for extremity sarcomas (MRI more helpful in Wide local resection for grossly clear margins only retroperitoneum to allow evaluation of the IVC) Resect en bloc only organs where sarcoma is clearly invaded

105 Part 2.qxd 10/19/05 2:52 AM Page 106

106 Skin and Soft Tissue—Sarcoma

Dissect sarcoma free if adherent to an intra-abdom- Development of lung metastases inal structure IVC will be invaded in retroperitoneal sarcoma No indication for use of adjuvant RT in retroperi- Upper extremity sarcoma toneal sarcomas (too much visceral toxicity) Only do percutaneous biopsy if there is extensive peri-aortic adenopathy and the dx is most likely Strikeouts lymphoma Can excise IVC if involved and replace with Gortex Attempt an FNA of mass if pt hasn’t already developed sufficient collateral Incisional biopsy transverse to underlying muscle group around it. Trying to treat only with chemotherapy (only small cell Pulmonary Mets sarcomas!) Acceptable to remove if primary disease site is con- Removing adjacent organs in retroperitoneal sarcoma if trolled and number of pulmonary mets < 8 no actual invasion Removing entire muscle group when clear margin can be achieved with less aggressive surgery Common Curveballs Not attempting pulmonary metastectomy when sar- coma recurs in lungs Retroperitoneal sarcoma will abut or invade multiple Resecting sciatic nerve intraabdominal organs Not preparing pt pre-op for possible paralyzed leg or Extremity sarcoma will invade neurovascular bundle amputation in attempt to perform adequate resec- Recurrence locally (re-excise in extremity if possible or tion amputate) Part 2.qxd 10/19/05 2:52 AM Page 107

Skin and Soft Tissue—Skin Cancer (Other than Melanoma)

Concept → want 1 cm margin for lesions > 2 cm in size node dissection if Usually we are talking here about squamous cell cancer or palpable nodes or if Margolin’s basal cell cancer. Can consider other types of benign skin ulcer lesions in your differential, but question will be how to For Basal cell → want 2 mm margins manage the malignant type. Basal cell is most common and may present as nodular, superficial, or ulcerating For lesion on head/neck → would want to resect and lesion. close defect with: (1) Free full thickness skin graft from behind ear or base of the neck Way Question May be Asked ? (2) Rotation flap

“66 y/o male presents to office with bleeding scalp lesion. Mohs Surgery Indications Fungating large mass on physical exam. When patient’s hat is removed. What do you want to do?” Recurrent basal/squamous cell CAs Tumors of face, invasive into nasal, periorbital, periau- ricular structures How to Answer? Brief H+P Adjuvant Treatment (XRT) Risk factors: Close margins of resection (< 1 mm) (Excessive) Sun exposure Neuro/vascular invasion Radiation BCC/SCC in medial canthus of eye/nose Inherited skin disorders Lymph Node Dissection Physical Exam Only for clinically involved nodes (modified radical Characteristics of lesion (size, shape, color) neck + suparotidectomy if tumor invading parotid) Full skin survey (include axillae, groin, scalp) Examine lymph node basins related to lesion Common Curveballs Treatment Path will come back melanoma (change scenario) Briefly consider DDx and then proceed to Lymph nodes will be palpable Surgical excision Excised lesion will recur Review pathology Lesion will be on face and needs full thickness skin graft Tumor will be large and ulcerating For Squamous cell → want 5 mm margins for lesions < There will be palpable nodes 2 cm in size Tumor will be preauricular invading parotid gland

107 Part 2.qxd 10/19/05 2:52 AM Page 108

108 Skin and Soft Tissue—Skin Cancer (Other than Melanoma)

Treating a lesion that develops in chronic wound (Margolin’s ulcer)

Strikeouts

Discussing electrodessication and curettage Discussing Mohs surgery when not indicated Discussing simply treating with radiation/chemo- therapy Talking about SLN bxs or elective lymph node dissec- tion Part 2.qxd 10/19/05 2:52 AM Page 109

Stomach and Duodenum—Duodenal Ulcer

Concept History should also focus on symptoms being sure to r/o other possibilities: Majority of questions will be related to obstruction, bleed- Pancreatitis ing, or perforation. Most ulcers are related to H. pylori or MI NSAID use. Nonoperative therapy may be appropriate for Pneumonia → all less likely if see free air, make sure initial discovery of ulcer and for initial bleeding ulcer. Be AXR is upright! sure to rule out ZE syndrome, ulcerogenic medications, Esophagitis hyperparathyroidism, and antral G cell hyperplasia when Gastritis appropriate. Gallbladder dx Aortic dissection

Way Question May be Asked? Physical Exam “A 43 y/o man presents to ED with acute onset of severe Check vital signs epigastric pain with a rigid abdomen on physical exam. Look for peritoneal signs (guarding, rebound) Upright AXR reveals free air.” Unlikely to get a presenta- Remember findings are more subtle in elderly and in pt tion this classic. Be sure to go through your DDx for epi- on steroids gastric pain ruling out MI and pancreatitis, or your DDx for UGI bleeding if appropriate. Labs Full laboratory panel including amylase/lipase How to Answer? Gastrin/Ca++ if suspicion of gastrinoma, hyperparathy- roidism or chronicity History NSAID, smoking, ethanol use Radiologic Studies History of ulcer symptoms (chronic hx affects your choice of operation!) For perforated ulcer, need: H. pylori treatment Upright AXR Family history (MEN I) CT scan could demonstrate free air and r/o H blocker therapy 2 diverticulitis Foreign body ingestion Diarrhea (gastrinoma) For bleeding ulcer, need EGD → will r/o other pathol- ogy, help predict course, treat bleeding, and check for H. pylori

Treatment of bleeding ulcer by EGD: Electrocautery Heater probe Injection therapy

109 Part 2.qxd 10/19/05 2:52 AM Page 110

110 Stomach and Duodenum—Duodenal Ulcer

Endoscopic appearance Notes about Surgery: Clean-based ulcer (rarely rebleed) Adherent clot ( likely to rebleed) Should always try for Bilroth I (avoids afferent/efferent Non-bleeding vessel (likely to rebleed) problems with Bilroth II and problems with second anas- tomotic line). Be sure to extend at least 0.5 cm beyond dis- For obstruction, need UGI tal edge of pylorus and check proximal antrectomy line with frozen section to show parietal cells. Treatment If doing pyloroplasty, may not be able to do typical Heineke-Mikulicz pyloroplasty with a scarred duodenum, For Perforated Ulcer: so do a Finney or a Jaboulay (anastomosis involving distal No role for conservative treatment! stomach to second portion of duodenum). If all three are Need to initially resuscitate pt (IVF, antibiotics, H2 impossible, gastrojejunostomy is effective emptying proce- blockers) dure. Take to OR, upper midline incision Truncal vagotomy involves stripping the esophagus bare Three choices: of areolar tissue in the distal 5–7 cm of esophagus. High risk pt (elderly, > 24 h, unstable, advanced If pt has had prior surgery, and pre-op work-up reveals peritonitis) no specific cause for recurrence, take next most aggressive Omental patch and abdominal lavage (> 5 liters option: → saline) If prior vagotomy with drainage antrectomy → Good risk pt (young, < 24 h, stable, early peritonitis) If prior antrectomy with vagotomy subtotal gas- Omental patch, parietal cell vagotomy, lavage trectomy Good risk pt with hx of PUD: Antrectomy (will include ulcer)/vagotomy, lavage Common Curveballs

For Bleeding Ulcer: EGD will see adherent clot or visible vessel Treatment initially is conservative with EGD, transfu- Perforation will be over 24 h old Perforation will be in pt with long hx refractory ulcer dx sions, H2 blockers Should have your limit of transfusions before going to Perforation will be in elderly pt OR (> 6 in 24 h or hemodynamic instability) Pt will have had prior abdominal surgery Should know what endoscopic appearance is relative Won’t be able to close duodenal stump after antrectomy indication for OR Pt will keep requiring blood transfusions, but spread Three choices here too: out over several days High risk pt: Nonoperative treatment will work and pt will later pres- Vagotomy/pyloroplasty/oversew of ulcer (U ent with gastric outlet obstruction stitch) Asked to describe how to perform vagotomy/pyloro- Good risk pt with small ulcer plasty/antrectomy/ and/or “U stitch” for bleeding Oversew ulcer and parietal cell vagotomy duodenal ulcer Good risk pt with large ulcer (> 2 cm) or hx PUD Gastrojejunostomy will be complicated by marginal Antrectomy/vagotomy ulcer, afferent loop syndrome, bile reflux, gastritis, dumping syndrome. Duodenal stump will leak post-op For Obstruction: Pt may rebleed post-op after U stitch performed (con- Initial conservative Tx with trial of NGT decompres- sider angiographic embolization of gastroduodenal sion artery) Pt will have ZE syndrome H2 blockers Check UGI to confirm Pt will have had prior ulcer surgery If this fails (which it will), then proceed to OR Two choices here: High risk pt: Gastrojejunostomy +/− vagotomy Low risk pt: Antrectomy and vagotomy (Bilroth I reconstruc- tion) Part 2.qxd 10/19/05 2:52 AM Page 111

Strikeouts 111

Strikeouts Stats vary with literature quoted, but rough rates cited below: Not ruling out other etiologies of epigastric pain Trying to treat perforated ulcer conservatively Recurrence Mortality Morbidity Not trying to conservatively treat a bleeding ulcer at Vagotomy/ 10% 1% 15% first presentation pyloroplasty Not being prepared to perform a different operation Vagotomy/ 1% 2% 20% in someone with chronic sxs antrectomy Not performing EGD for bleeding ulcer Parietal cell 10% 0% 5% Trying to treat gastric outlet obstruction with endo- (HSV) vagotomy scopic balloon dilatation Performing any operation laparoscopically Not knowing how to manage the difficult duodenal stump Not knowing how to manage duodenal stump leak Not oversewing bleeding site when performing vago- tomy/pyloroplasty Not having an idea in your head about recur- rence/mortality rates after different operations Forgetting H. pylori Trying to perform highly selective vagotomy in unstable pt Part 2.qxd 10/19/05 2:52 AM Page 112

Stomach and Duodenum—Gastric Cancer

Concept Diagnostic Studies Will likely present as a large ulcer and biopsy proven UGI malignancy. Patient may not be candidate for anything but EGD palliation. Be prepared to describe your work-up and CT scan (to r/o metastatic disease) operation. Remember that gastric lymphoma is a different Can consider laparoscopy at onset of operation (r/o beast from gastric cancer. liver mets/carcinomatosis) Measure basal acid output (achlorhydria assoc. with malignancy) Way Question May be Asked? Location of tumor: (1) Tumors in antrum/distal third of stomach → radi- “A 63 y/o man presents to ED with UGIB. After stabiliza- cal subtotal involving 3 cm of first part tion, an EGD is performed that reveals a large ulcer on the of duodenum, hepatogastric omentum, greater greater curvature, biopsies return with well-differentiated omentum, and a D1 resection (immediately adja- adenocarcinoma. What do you do?” May also present as a cent perigastric lymph nodes) non-healing ulcer with pain, perforation, obstruction, or (2) Tumors in corpus/middle third of stomach→ subto- in work-up for melena or heme + stool. tal or total depending on size of tumor (3) Tumors in proximal third → total gastrectomy, How to Answer? reconstruction with Roux-en-Y (4) Palliation → total gastrectomy (not gastroenteros- History tomy!) Risk factors Comments on Surgery Weight loss Abdominal distension Resection with 5 cm margins (if within 5 cm of GE junction, needs total gastrectomy Only resect spleen if gross tumor involvement Physical Exam No evidence for resection of hepatic metastases Evidence of weight loss/malnutrition Check margins of resection by frozen section Palpable abdominal mass En bloc resection of any directly invaded organ (spleen, Prior surgical scars tail of pancreas, kidney), except CBD or head of Lymphadenopathy (supraclavicular, periumbilical) pancreas Rectal exam (Blummer’s shelf) No evidence for Japanese style D2 resection Should perform D1 resection which includes: suprapy- loric, infrapyloric, and nodes along the greater and Labs lesser curvature Full laboratory panel Can consider adjuvant and neo-adjuvant treatments

112 Part 2.qxd 10/19/05 2:52 AM Page 113

Strikeouts 113

Don’t forget vagotomy (anastomosis is ulcer producing Tumor will have penetrated into surrounding structures procedure) (spleen, kidney, distal pancreas) Don’t forget different types of reconstruction (BII if Being asked the difference between R1, R2 and R3 cancer) nodes Pathology will be lymphoma May actually be esophageal cancer and need traditional Common Curveballs Ivor-Lewis Resection Pt will present later with evidence of metastatic dis- Anastomotic cancer 20 years after prior gastric surgery ease/obstruction Pt will have postoperative anastomotic bleed (especially Celiac node will be positive→ “what does that mean” if didn’t do vagotomy) Pt will have peritoneal mets → how to palliate pt Pt will have leak post-op Treatment for duodenal stump leak (if early, duodenos- Pt will be malnourished tomy, drains, NPO, TPN) Complication of gastric surgery post-op: (if late/abscess, CT guided drain, NPO, TPN) Dumping syndrome—conservative measure first, then Roux-en-Y Postvagotomy diarrhea—conservative measure first, Strikeouts then reversed jejunal segment Alkaline reflux gastritis—confirm by hepatobiliary Resecting hepatic metastases scan, conservative measure first, then RY gastro- Performing less than total gastrectomy for tumor < 5 jejunostomy cm from GE junction Anastomotic bleed—EGD, suture ligation if EGD Not staging pt appropriately fails Discussing laparoscopic resection of gastric cancer Afferent loop syndrome—side to side jejunojejunos- Not checking margins of resection by frozen section tomy Offering any therapy besides surgery for “cure” Gastroperesis—conservative measure first, comple- Discussing photodynamic therapy tion antrectomy or gastrectomy, depending on Discussing endoscopic mucosal resections prior surgery, may be necessary Ulcer will be high on greater curve near GE junction Part 2.qxd 10/19/05 2:52 AM Page 114

Stomach and Duodenum—Gastric Ulcer

Concept FHx ZE syndrome Use of anti-ulcer medications Four basic types of gastric ulcers categorized by location Relevant medical history (heart disease) and etiology. Always, ALWAYS, have a high index of sus- Prior surgeries (especially prior surgery for PUD) picion for malignancy and do everything possible to rule it out. Four types of gastric ulcers: Physical Exam I Lesser curve, unrelated to acid Vital signs (tachycardia/hypotension to suspect shock) II Gastric ulcer with associate duodenal ulcer, Abdominal exam (rigidity/peritoneal signs to suggest related to acid exposure perforation) III Prepyloric ulcer (within 3 cm of pylorus), Rectal exam (heme +, Blummer’s shelf) related to acid exposure IV Adjacent to gastroesophageal junction (juxtac- ardial), unrelated to acid Diagnostic Studies Routine labs including T+C and coags especially if bleeding Way Question May be Asked? Do lytes show evidence of gastric outlet obstruction (low K, low Cl, high bicarb)? “45 y/o male with history of UGIB who has a gastric ulcer Abdominal x-rays (r/o free air) identified on EGD. He has been on omeprazole for 8 +/− Barium UGI (no Barium if suspect perforation) weeks and repeat EGD shows ulcer still present. What do analysis (achlorhydria suggestive of Ca) you want to do?” Question may go in the direction of how EGD + bx! (at least 10 biopsies) to initially treat this patient, how long to trial acid sup- Any attempt at biopsy should include four quadrant pressive therapy, and when to operate, or it may jump right margins, central biopsy, and brushings! into a discussion of how to manage a bleeding or perfo- rated gastric ulcer. Size and pH are particularly important as most ulcers > 3 cm and most ulcers in the achlorhydric Surgical Treatment patient will eventually need surgery. (1) Resuscitate the unstable pt (2) Repeat EGD/biopsy at 6–8 weeks for the chronic ulcer, treat medically, and repeat EGD at 6–8 weeks, How to Answer? if improving, repeat EGD at 6–8 weeks: no improvement at 1st 6–8 week follow-up → OR History failure to disappear at 2nd 6–8 week EGD → OR Risk factors for PUD (3) Indications for surgery: Intractability H. pylori treatment Bleeding Steroid/NSAID use Perforation History of epigastric pain Obstruction Iron deficiency anemia Vomiting/bloating (from gastric outlet obstruction)

114 Part 2.qxd 10/19/05 2:52 AM Page 115

Strikeouts 115

(4) For Type I (lesser curve) ulcer (most common): Asked your method to test for H. pylori antrectomy to include ulcer (goblet cells on duodenal Pt will fail medical management side indicate adequate resection) Will turn out to be gastric cancer (check frozen section reconstruction with BI (make sure frozen section is before reconstruct) negative for malignancy before reconstruct with Asked your treatment algorithm for H. pylori BI) Will be asked how to manage type IV ulcer intra-op recurrence rate 2% Won’t be able to encompass ulcer in antrectomy (5) For Type II and III ulcers: Ulcer will perforate antrectomy and truncal vagotomy Pt will bleed post-op (6) For Type IV ulcers: Gastric acid measurements will show achlorhydria resection with Roux-en-Y esophagogastrojejunos- Discussion of postgastrectomy complications: tomy Bleeding (Csendes’ procedure) Dumping (7) For bleeding ulcer: Afferent/efferent obstruction EGD + biopsy Postvagotomy diarrhea +/− Angiogram with vasopressin/embolization Carcinoma Have threshold in your mind of when to operate on pt (more than 6U pRBC in 48 h—remember baseline comorbidities in your limit) Strikeouts In OR: (a) pt stable→ antrectomy to include ulcer when Describing any laparoscopic approach possible suture ligate ulcer/biopsy + antrec- Not knowing how to treat postgastrectomy syndromes tomy vagotomy for Type II,III ulcer Not knowing how to describe your chosen operation (b) unstable pt→ wedge resection or suture/biopsy Misdiagnosing a gastric cancer as a benign ulcer to ulcer + vagotomy/pyloroplasty Not testing for or treating H. pylori (8) For perforated ulcer: Not knowing importance of achlorhydria and its link (a) stable pt→ antrectomy to include ulcer or to malignancy antrectomy + omental patch and biopsy Not rescoping/re-biopsying pt with chronic non-healing ulcer ulcer (b) unstable pt→ biopsy and omental patch Not knowing indications for surgery (wedge resection of ulcer always an option if Spending too long with angiographic or nonoperative easy to do) methods to control bleeding Not checking for malignancy before performing recon- struction (BII is preferred for malignant gastric Common Curveballs ulcer)

Biopsies will come back malignant, indeterminant, benign Type of ulcer (I-IV) will change during scenario Part 2.qxd 10/19/05 2:52 AM Page 116

Stomach and Duodenum—Mallory–Weiss Tear

Concept Labs UGIB in a patient after forceful vomiting. The result of a Full laboratory panel including coags linear tear in the mucosa of the gastric cardia. T+C

Way Question May be Asked? Management Two large bore IV’s “A 23 y/o man presents to ED with hematemesis after Large caliber NGT binge drinking.” Pain should not be a prominent feature, if Irrigate via NGT to estimate ongoing blood loss so, consider Boerhave’s syndrome. May see in patients with Correct coags vomiting from other causes such as pancreatitis or Resuscitate the pt

chemotherapy. IV H2 blockers Blood transfusion if unstable EGD→identify and control bleeders r/o other How to Answer? pathology heater probe, sclerotherapy, electrocautery Resuscitate pt while doing focused H+P Angiography→ to diagnose bleeder Embolization of branches of left gastric History Selective infusion of vasopressin NO SENGSTAKEN–BLAKEMORE TUBES here NSAID/ethanol use History of PUD Surgery Indications H. pylori treatment Portal HTN Over 6 U PRBC transfused Hiatal Hernia (tear usually in gastric cardia rather than Failure of EGD to stop bleeding at GEJ) Failure of angiographic embolization (used in pts with History of Violent retching severe comorbidities) Remember your DDx of UGIB: Surgical Treatment PUD, esophagitis, varices, Mallory-Weiss Tear, Upper midline incision Physical Exam Explore UGI (may see subserosal hematoma at GEJ along lesser curve of stomach) Check vital signs Gastrostomy Look for peritoneal signs (guarding, rebound) Oversew of mucosal tear with absorbable, locking suture

116 Part 2.qxd 10/19/05 2:52 AM Page 117

Strikeouts 117

Can pack proximal and distal stomach with lap pads to May need to intubate pt with significant hematemesis locate bleeding source before EGD (otherwise pt will aspirate)

Common Curveballs Strikeouts

EGD will not see mucosal laceration Jumping to angiography rather than EGD first There will be evidence of perforation Using Sengstaken-Blakemore tube Stomach will be full of blood Not resuscitating the pt EGD will pick up other pathology (change scenario) Mistaking for Boerhave’s syndrome Endoscopic control/Angiographic control will fail Performing any type of anti-ulcer surgery (V+P, A+V, Pt will have portal HTN subtotal gastrectomy) Sclerotherapy will result in esophageal perforation Not looking for other pathology on EGD Pt will have had prior abdominal surgery Trying to do any of the above with a laparoscope Tears will be in distal esophagus (may need left thora- cotomy and esophagotomy and then suture ligation) Part 2.qxd 10/19/05 2:52 AM Page 118

Stomach and Duodenum—Upper GI Bleeding

Concept Algorithm

Important to pay attention to the broad DDx here as well ABCs + as close attention to the ABCs as the patient with massive Resuscitation (IVF, full labs including PT/PTT, T C, hematemesis may exanguinate while you are still perform- NGT) ing a history with questions related to alcohol use, vomit- Gastric irrigation through NGT + − ing, and liver disease. / endotracheal intubation depending on severity of bleed Endoscopy (localization and possibly therapeutic) Way Question May be Asked? +/− angiography

“51 y/o male presents to ED with vomiting blood twice at Endoscopic Methods to Control Bleeding home. BP is 80/50. What do you want to do?” May have Heater probe patient that presents with more chronic blood loss with Electrocautery black, tarry stools. Presentation will guide how quickly Epinephrine injection you move into treatment options. Band ligation/sclerotherapy (esophageal varices) (appearance important here as overlying clot/visible vessel have higher chance of rebreeding than How to Answer? clean ulcer base) Brief H+P While Resuscitating the Patient History of PUD Angiography Associated pain Can treat certain bleeds with intra-arterial gelfoam, Age metal coil springs, vasopressin ASA, NSAID, steroid/alcohol use Useful for gastric/duodenal ulcers Recent retching/vomiting (Mallory-Weiss Tear) If bleeding controlled, don’t forget: Liver disease Antacids, H blockers, treatment of H. pylori Trauma 2 History of UGI surgery (marginal ulcer) History of AAA repair (aortoenteric fistula) Surgical Treatment Reserved for pts with continued or recurrent bleeding Physical Exam (6 U pRBCs), complicated ulcer disease, massive UGIB, non-healing ulcers Stigmata of liver disease Evidence of prior surgery (always note any surgical For gastric adenoCA: resect with 5 cm margin, if within scars) 5 cm of GEJ= total gastrectomy Melena (never leave out rectal exam) For stress gastritis: total gastrectomy, or gastric devas- cularization if unstable (quicker)

118 Part 2.qxd 10/19/05 2:52 AM Page 119

Strikeouts 119

For gastric ulcer: Common Curveballs →stable pt antrectomy to include ulcer or suture + ligation/biopsy/ antrectomy Angiogram won’t localize lesion, and/or embolization → unstable, wedge resection or suture won’t work ligation/biopsy/vagotomy/pyloroplasty Endoscopy won’t localize lesion Pt will have had prior ulcer surgery For duodenal ulcer: Pt will have had prior AAA repair → high risk/unstable, vagotomy/pyloroplasty/- All coags will be abnormal oversew ulcer (U stitch) NGT won’t get bilious return → stable pt, no hx PUD, small ulcer, oversew and Bleeding will be from duodenum despite non-bloody, parietal cell vagotomy bilious NGT aspirate → → giant ulcer/stable/hx PUD antrectomy Bleeding will recur after endoscopic treatment + vagotomy Large ulcer will be malignant May need to make gastrotomy/duodenotomy to For bleeding from anastomotic line from recent localize bleeding surgery: May be from nasopharynx or hemoptysis from lungs → EGD, if/when fails, re-explore and ligate bleeder GI doc won’t be available to perform EGD Any nonoperative therapy will fail For Mallory–Weiss: “U-stitch” won’t work→ligate gastroduodenal → gastrotomy, suture ligation of mucosal tears (if tears in esophagus, left thoracotomy/esophago- tomy, suture ligate bleeders Strikeouts

For Aorto-enteric fistula: Not placing NGT → control bleeding, then extra anatomic bypass Taking too long in H+P Not resuscitating pt prior to surgery For varices Not taking pt to surgery when appropriate → TIPS or emergency portacaval shunt Not treating for H. pylori Not biopsying an ulcer seen at EGD Placing Sengstaken/Blakemore tube for Mallory-Weiss tear Performing distal splenorenal shunt emergently for bleeding varices Part 2.qxd 10/19/05 2:52 AM Page 120

Thoracic—Empyema

Concept To confirm complete evacuation of effusion To check for trapped lung, loculations, unexplained Infection localized in the thoracic cavity outside of the atelectasis (endobronchial lesion) lung. May be the result of any neighboring infection (pneumonia, esophageal perforation, bronchopleural fis- Send fluid for: tula, recent surgery, subphrenic abscess, generalized sepsis, Cultures (aerobic, anaerobic, acid fast, fungal) undrained pleural effusion). Cytology (r/o malignancy) LDH, cell count, pH, LDH, glucose

Way Question May be Asked? CT scan of chest Adenopathy “55 y/o male in the ICU with persistent left loculated effu- Loculations sion and thoracentesis performed reveals purulent mate- Mass lesion rial. What do you want to do?” Bronchoscopy If suspect endobronchial lesion How to Answer? Empyema stages: History Exudative < 7 days Fibropurulent 7–14 days Tobacco use Organized > 14 days Chest pain Fever Recent pneumonia or infection Surgical Treatment h/o Cancer Exudative stage—Thoracentesis (usually prior to CT HIV placement) Chest tube drainage + Abx Physical Exam Fibropurulent stage—VATS exploration/pleurodesis (pleural biopsy/cytology if sus- Auscultation of chest pect malignancy) or limited Examine for any adenopathy thoracotomy Organized stage—VATS decortication How to Answer? Open decortication Rib resection and Eloesser flap (skin Need CXR prethoracentesis/chest tube sutured to parietal pleura)→ used in Look for lesion high risk pt! Look for air-fluid level (bronchopleural fistula) Need CXR post thoracentesis/chest tube

120 Part 2.qxd 10/19/05 2:52 AM Page 121

Strikeouts 121

Common Curveballs Strikeouts

Cytology will be positive for malignancy (switch sce- Forgetting PFTs if proceeding towards thoracotomy narios) Describing VATS as an option if you don’t do know Empyema will fail treatment with chest tube how to do this procedure Pt will develop bronchopleural fistula post thoraco- Mentioning Eloesser flap if don’t know how to perform tomy it/its indications Stains for acid-fast bacilli will be positive Not checking cytology on drained effusion Lung won’t re-expand after chest tube drainage of effu- Not performing bronch for persistently unexpanded sion/empyema lung fields Empyema will be the result of some extra-pulmonary process (perforated esophagus, subphrenic abscess, . .)—scenario switch! Part 2.qxd 10/19/05 2:52 AM Page 122

Thoracic—Lung Cancer

Concept How to Answer? Surgical treatment is the only potential cure. Key is to Need prior CXR to compare (if > 3 cm and present on determine if patient is resectable or not. May present as a prior CXR, unchanged, can follow) solitary pulmonary nodule where only 1 in 20 turns out to Need CT scan (chest including liver/adrenals) be actually malignant. Evaluate mass size and location Evaluate for metastases Evaluate lymph nodes Way Question May be Asked? Need three morning sputum cytologies and cultures Need flexible bronchoscopy “64 y/o female found to have a new lesion, ~ 2 cm in diam- Need biopsy of lesion eter in the LUL found on a pre-op chest x-ray prior to a Brush biopsy hysterectomy for fibroids. What do you want to do?” Trans-bronchial Percutaneous by CT Thoracotomy How to Answer? Need pre-op pulmonary tests: ABG History PFTs V/Q scan (to predict post-op FEV1) Tobacco use Asbestos exposure Chemical exposure Travel hx Staging of Lung CA: History of prior cancer (mets?) T1 = < 3 cm T2 = > 3 cm Symptoms T3 = < 2 cm from carina or tumor invading chest wall, diaphragm, or New voice changes/ neuro symptoms mediastinal pleura Weight loss T4 = tumor invades any mediastinal structure Chest/bone pain (esophagus, heart, great vessels, trachea) or Shortness of breath satellite tumor nodules within ipsilateral Hemoptysis lobe, or malignant pleural/pericardial effusion Physical Exam N1 = hilar LN involved N2 = ipsilateral mediastinal/subcarinal nodes Auscultation of chest involved Examine for any adenopathy N3 = contralateral mediastinal nodes, ipsilateral Palpate liver scalene/supraclavicular nodes involved

122 Part 2.qxd 10/19/05 2:52 AM Page 123

Strikeouts 123

Contraindications to Surgical Resection Will present as hemoptysis Will present as pleural effusion T3 or T4 lesions Will present as lung abscess N3 lesions Tumor will be hormonally active: Predicted post-op FEV1 < 0.8 ACTH PTH-like Surgical Treatment ADH Pt will have post-pneumonectomy: Mediastinoscopy for left sided nodules Bronchopleural fistula Chamberlain procedure for right sided nodules and Atrial fibrillation enlarged left para-tracheal nodes Hemoptysis Lobectomy Mediastinal shift in recovery room Pneumonectomy (if hilar lesion, lesion encompasses all lobes on a given side (crosses fissures) Strikeouts

Common Curveballs Forgetting PFTs prior to thoracotomy Describing VATS or segmentectomy or wedge resection No lesion will be benign (even if on prior CXR) as an option if you don’t do (know how to do) this Lesion will turn out to be metastatic disease (scenario procedure switch—will you perform pulmonary metastatec- Not performing bronch tomy? For what tumors?) Not checking prior CXR Tumor will have characteristic of unresectability: Operating on small cell carcinoma Horner’s syndrome Not performing mediastinoscopy pre-op when indi- Positive cytology from pleural effusion cated Positive cervical lymph nodes Offering palliative resections Tracheoesophageal fistula Not knowing staging system (important to understand- Recurrent laryngeal nerve or phrenic nerve ing Contraindications to surgical resection) paralysis Part 2.qxd 10/19/05 2:52 AM Page 124

Trauma and Critical Care—Abdominal Compartment Syndrome (ACS)

Concept Intrarenal ATN from any hypotension (sepsis, intra-op) Increased pressure in a confined space will lead to Toxic Medication (aminoglycosides) decreased perfusion of all organs in the abdomen Systemic diseases (SLE,TTP) Postrenal Bilateral ureteral occlusion/injury (rare) Way Question May be Asked? Foley problems (kink, clogged) Answer should include a discussion of: Commonly a disguised question like—“You are called to see Pt’s volume status (place CVP or SGC after a 68 y/o male in the recovery room 5 h after a LAR, per- thorough H&P although examiner is likely to formed by your partner who just left on vacation, begins to tell you that pt is intubated) have a decreasing urine output. What do you want to do?” It Pt’s baseline renal function very well could be a patient you are called to see because of Intra-op events—transfusion, hypotension, high ventilatory pressures where you have to rule out ARDS, placement of CVP with tension pneumotho- pneumothorax, mucus plug, too little sedation,...) rax, or meds administered Review meds—any excreted only by kidney (digoxin) How to Answer? Is Foley patent? All this done quickly so examiner can focus on Have to understand that ACS will affect all intraab- ACS with discussion of how you diagnose

dominal organs. Elevated intraabdominal pressure will (bladder pressures or gastric O2 measure- effect the cardiopulmonary system and increase ventila- ments) and how you treat (open abdomen and tor peak pressures and decrease cardiac output. It will close with mesh or Bogata bag) also precipitate renal failure because of direct pressure effects on the kidney, as well as decreased kidney perfu- sion Common Curveballs Have to be systematic and work through algorithm for Pt had several liters of NSS intra-op to lead you away renal failure including: from thinking further boluses (insensible loses alone Prerenal up to 6 cc/kg per hour OR time) Shock-Hemorrhagic Pt has normal SGC parameters Septic Pt had history of renal insufficiency, MI, or only has Third-space losses related to burn or long one kidney operation Nothing you do will work (just testing your thinking— Pump failure make sure you follow labs for electrolyte abnormali- Acute MI/CHF ties, especially K+, change any meds that need renal Compartment syndrome dosing, and consider early hemodialysis) Vascular Emboli after suprarenal aorta clamp

124 Part 2.qxd 10/19/05 2:52 AM Page 125

Strikeouts 125

Strikeouts

Not considering compartment syndrome (check Not placing Foley catheter or checking its patency bladder pressures!) Giving diuretic before ensuring adequate volume status Not ruling out other causes of renal failure (it will be Not frequently reassessing pt if all else fails (labs/PE) whatever you leave out!) Not being complete: Not being invasive to determine volume status (CVP or not performing physical exam (JVD, rales, skin SGC) tugor, abdominal distension, ventilator pressures) or Not checking CXR to r/o pneumothorax from intra-op checking labs (lytes, BUN, Cr, U/A, urine lytes) line or CHF Part 2.qxd 10/19/05 2:52 AM Page 126

Trauma and Critical Care—Colon and Rectal Trauma

Concept Physical Exam Likely to be seen in the context of multiple other injuries Rectal exam for example, GSW to abdomen with small bowel and Rigid sigmoidoscope colon injury or in the setting of a pelvic fracture with an obvious rectal injury. Again, go through the ABCs in all Diagnostic Studies these questions to avoid missing an injury. DPL CT scan Way Question May be Asked? Surgical Treatment “36 y/o male is being evaluated for a pelvic fracture in the ED and, on , there is gross blood. What Resuscitate the pt do you want to do?” This may be in the setting of a pelvic Rule out other life-threatening injuries fracture in the OR after an exploratory laparotomy was performed, in the setting of a GSW to the pelvis or thigh, Colon injuries: or from direct rectal trauma. Remember to deal with life- (1) Primary repair if: threatening injuries first. Pt stable Small laceration (< 1 cm) Contamination minimal How to Answer? Not on anti-mesenteric border Debride to healthy tissue In trauma setting, always the ABCs: Close in one or two layers Airway and C-spine control (intubate with C-spine (2) Colostomy for left sided injuries in: control if necessary) Pt in shock Breathing and Ventilation (does pt need chest tube— Multiple other injuries place before CXR) Peritonitis Circulation and IV access (3) Resection and anastomosis for right sided injuries Disability (Neuro status) in: Don’t skip secondary survey either, or you will miss Destructive wounds but pt stable with minimal some key finding! injuries

History Rectal injuries: Diverting stoma Pelvic fracture Presacral drainage: Penetrating abdominal injury (remember abdomen 3 cm curvilinear incision b/w coccyx and rectum stretches from nipples to groin) and

126 Part 2.qxd 10/19/05 2:52 AM Page 127

Strikeouts 127

Posterior dissection carried up to level of injury Strikeouts Distal rectal washout 2 liters of GU irrigant following an anal stretch Not knowing how to treat rectal injury with diversion, drains, and washout Not knowing to primarily repair small colon injuries Common Curveballs Not looking for other injuries Not looking for rectal injury in pelvic fracture Pt will have other associated injuries (intra- and Not looking for bladder injury in pelvic fracture extraabdominal) Missed bowel injury Pt will be unstable intra-op Pt will develop abdominal compartment syndrome post-op Pt will leak for primarily repaired colon wound Pt will have open pelvic fracture (gets diverting sigmoid colostomy regardless of whether rectal injury or not) Pelvic fracture will have coincident bladder injury Part 2.qxd 10/19/05 2:52 AM Page 128

Trauma and Critical Care—Extremity Compartment Syndrome

Concept (2) Sensation of dorsum of foot (superficial peroneal nerve = lateral compartment) Elevated pressure in a closed compartment that leads to (3) Sensation of plantar surface (tibial nerve = ischemia damage to muscle and nerve that may lead to deep posterior compartment) limb loss and myoglobinuria with resulting renal failure. (4) Pain with passive dorsiflexion and plantar Most commonly in the extremities as the result of crush flexion of great toe injuries, vascular injuries, reperfusion after prolonged ischemia, compression by cast, or burns. Key is high index Diagnostic Tests of suspicion. Measuring compartment pressures (1) Use specific device such as Stic catheter Way Question May Be Asked? (2) Attach 16 g needle to A-line setup with three way stop cock and sterile saline, zero monitor “You have just finished repairing a GSW to the femoral and inject 1cc into compartment artery and vein and you notice that it took you about 6 h Should measure all compartments at risk to perform. The nurse wants to know if there is anything Repeat exam at intervals if suspicions remains high else you would like to do.” Rarely would the question be so especially during resuscitation leading, but remember in any case of vascular trauma, orthopedic trauma, or vascular repair. Also, consider in Surgical Treatment patients with deep burns to the extremity. Decompression for: Strong clinical suspicion How to Answer? Compartment pressure > 40 mmHg Compartment pressure within 30 mmHg of dias- History tolic BP Mechanism of injury Bivalve any cast Pain out of proportion to injury OR for fasciotomy: Pain distal from site of injury In leg: Tingling/numbness in extremity Two incisions First incision from knee to ankle and centered Physical Exam between anterior and lateral compartments Divide fascia 1 cm above and below intermuscular Tense, swollen extremity septum to free anterior and lateral compart- Pain with passive range of motion ments respectively Sensory deficit Careful to avoid superficial peroneal nerve in Absence of pulses is late finding lateral compartment In leg, examine: Second incision also from knee to ankle and is (1) Sensation in first web space (deep peroneal 2 cm posterior to posteromedial border of nerve. = anterior compartment) tibia

128 Part 2.qxd 10/19/05 2:52 AM Page 129

Strikeouts 129

Avoid saphenous vein Compartment pressure will change on repeated record- Divide fascia overlying gastrocnemius and soleus ings muscles (medial compartment) Asked to describe how to perform fasciotomy Detach soleus from posterior tibia to reach fascia Pt will have necrotic muscle after fasciotomy and asked of deep compartment and incise if you debride (no, wait and return to OR, necrotic Apply loose dressings tissue may improve) Keep extremity at heart level (don’t raise!) Pt will develop myoglobinuria and renal failure Return to OR q36 h to debride necrotic tissue/dressing changes Keep pt well hydrated to avoid renal failure Strikeouts STSG if can’t close after 7 days Not correctly diagnosing compartment syndrome Only performing escharotomies when fasciotomy indi- Common Curveballs cated Not being able to describe fasciotomy Pt will have altered sensorium/be intubated and you Waiting until extremity is pulseless before performing won’t be able to obtain H+P fasciotomy Compartment pressure will be 30 mmHg Trying to describe one incision quadruple fasciotomy for the leg Part 2.qxd 10/19/05 2:52 AM Page 130

Trauma and Critical Care—Duodenal Trauma

Concept determine amount of duodenal tissue loss determine location of injury (part 1 vs. 4) Low incidence of injury and typically in conjunction with type/severity of other injuries (liver, spleen, colon, injury to other organ systems. Diagnosis is likely not made ureter,...) preoperatively. Often times may be in a young teenager (a) simple laceration→ two layer closure, omental after blunt abdominal trauma. patch (b) intramural hematoma→ can observe, usually no leak but once in OR, safer to explore and rule Way Question May be Asked? out laceration or leak (c) more complicated lacerations→ debridement “You are exploring a patient for a splenic laceration and and closure as long as no luminal compro- find a periduodenal hematoma on your exploratory mise/undue tension laparotomy. What do you want to do?” May be given the (d) large laceration to second portion of duodenum diagnosis of a hematoma on pre-op CT scan, might get the or whenever unable to do primary repair→ history of the patient receiving a blow to the epigastric Roux-en-Y or loop duodenojejunostomy region, or might get a patient referred to your hospital for (e) multiple complex lacerations→ duodenal definitive management after diagnosis made. exclusion pyloric closure (staples or sewn) diverting gastrojejunostomy primary repair of duodenal lacerations How to Answer? tube duodenostomy, external drainage, +/− T-tube As with nearly everything covered on the Orals, this is a (f) always leave drain! management question. (g) always decompress post-op: NGT decompression post-op or tube duo Surgical Treatment denostomy remote from injury! (h) if associated contusion to head of pancreas, To identify/expose intra-op: must look for CBD or pancreatic duct injury Wide Kocher maneuver to visualize posterior duode- (i) avoid temptation to perform the “trauma num (extend mobilization to ligament of Trietz) Whipple”→ only in the stable pt with combined Careful inspection of pancreas pancreaticoduodenal injury where all other measures fail or severe ampullary injuries (1) If find pre-op: (j) leave J-tube for post-op feeding can observe if isolated injury no evidence of leak on gastrografin swallow followed by negative Barium swallow Common Curveballs observation limited to 2 weeks, at which time you should explore pt Pt will have multiple other injuries (2) If find intra-op (or take to OR): Pt will have pancreatic injury fully expose duodenum

130 Part 2.qxd 10/19/05 2:52 AM Page 131

Strikeouts 131

Pt will become acidotic/coagulopathic intra-op and you Strikeouts will need to perform “damage control” surgery Pt will fail nonoperative management Not looking for other injuries (including pancreas) Post-op leak Not adequately visualizing duodenum (need to be able Pt will develop intra-op DIC (change scenario) to describe intra-op techniques) Pt will have post-op fever Not having a variety of ways to treat duodenal injuries Asking your post-op management Not using NGT post-op or leaving an external drain Marginal ulcer post-op pyloric exclusion (change sce- Not considering “damage control” when appropriate nario with pt having UGIB) Part 2.qxd 10/19/05 2:52 AM Page 132

Trauma and Critical Care—GU Trauma

Concept How to Answer? Will usually be couched in another question, for example, Need complete labs, CXR, U/A (hematuria) the multiple injured trauma patient who has a retroperi- DPL in unstable pts (this is a common Oral Exam toneal hematoma seen after exploration of a penetrating theme!) abdominal injury. Could also be seen in the setting of CT scan in stable pts with IV contrast (evaluate both blunt trauma with a pelvic fracture. Don’t forget the pri- kidneys) orities in trauma patients . . . ABCs. Retrograde cystourethrogram to define urethral injury

Way Question May be Asked? Surgical Treatment Be sure to r/o other intra/extraabdominal injuries: “19 y/o male seen in the emergency room after a GSW to (1) Bladder injuries his right flank with a SBP of 90 and gross hematuria after (a) extraperitoneal female—Foley catheter or placement of a foley catheter. What do you want to do?” suprapubic cystostomy Could also have presentation after a fall from a height or (b) extraperitoneal male—suprapubic cystostomy a car accident with a pelvic fracture secondary to blunt (c) intraperitoneal—primary repair and suprapu- trauma. Make decision on stability of the patient early and bic cystostomy frequently reassess throughout the scenario. (2) Ureteral injuries—key is the level of injury (a) lower third—ureterneocystostomy +/− psoas hitch How to Answer? (b) middle third—end to side ureteroureterostomy to other ureter (most urologists hate this option) ABCs (c) proximal third—nephrostomy tube in ipsilat- Primary Survey eral kidney Secondary Survey (d) if tissue loss minimal, can try primary repair over a stent History ALWAYS drain site of repair! (3) Renal parenchymal injury AMPLE history (Mechanism of injury,...) (a) non-visualization on CT/IVP→ angiography Pre-existing renal disease and/or exploration promptly (1 h warm ischemia time too much!) → Physical Exam (b) renal vein injury repair in stable pts, otherwise ligate Full exam in secondary survey especially (c) renal artery→repair in stable pts otherwise Blood at urethral meatus nephrectomy Stool guiac for possible rectal injury (d) extravasation→repair or partial resection in “High riding” prostate in male stable pt otherwise nephrectomy (e) pedicle avulsion→nephrectomy

132 Part 2.qxd 10/19/05 2:52 AM Page 133

Strikeouts 133

(4) Retroperitoneal hemotomas Post-op extravasation of contrast from bladder/kidney (a) all hematomas in penetrating trauma should be injury explored unless subhepatic Asked when you will perform a nephrectomy (b) can observe hematoma in blunt trauma as long There will be injury to other retroperitoneal organs as not expanding (duodenum, pancreas, colon) (c) should explore central, portal, and pericolonic Will be asked to describe performing psoas hitch or hematomas nephrostomy tube placement (5) Urethral injuries whether partial or total, gets cys- tostomy and delayed urethroplasty (6) To expose retroperitoneal structures Strikeouts Mattox maneuver on left Cattell maneuver on right Any sort of laparoscopic treatment/evaluation Getting stuck on therapeutic embolization for renal laceration Common Curveballs Not checking meatus/rectum in pelvic fracture pt Not exploring retroperitoneal hematoma when appro- Pt will be unstable priate Will have intra-op retroperitoneal hematoma Not performing nephrectomy when appropriate Will have one kidney Not ruling out other, more life-threatening injuries Will have bladder injury, first extraperitoneal, then Not looking for injuries to other retroperitoneal organs intraperitoneal Performing CT scan in unstable pt Will have ureteral injury in a variety of locations Post-op hypertension from activation of renin/ angiotensin/aldosterone axis Part 2.qxd 10/19/05 2:52 AM Page 134

Trauma and Critical Care—Liver Trauma

Concept CXR Pelvis x-ray Frequently injured abdominal organ. Remember to resus- citate patient and rule out other injuries. Can consider Nonoperative Management nonoperative management in the stable patient, with no other indications for abdominal exploration. For blunt trauma with minimal other injuries, no indi- cations for abdominal exploration, and no hemody- namic instability Way Question May be Asked? Have low threshold to take to OR

“You are called to the trauma bay to help out your partner Operative Management who has a 26 y/o motor cyclist injured after impact against a guard rail who is tachycardic, hypotensive, and has con- Prep chin to knees tusions about the right side of his chest and abdomen. Midline incision What do you want to do?” May also be given the intra-op 4 quadrant packing setting of multiple injuries—liver, spleen, small bowel, and Rapid abdominal survey and control any intestinal ureter and asked how you will proceed. spillage Mobilize liver (divide falciform, triangular/coronary ligaments) How to Answer? +/− pringle maneuver with vascular clamp if major hemorrhage (will stop hepatic artery and portal vein Brief H+P while resuscitating the pt: branch bleeding) ABCs (1) Simple laceration—direct pressure, topical hemo- PMHx static agents, cautery, argon beam coagulator Meds (2) Deep laceration—Pringle maneuver, ligation Allergies individual vessels, pack laceration with vascular- ized tongue of omentum mobilized from trans- Physical Exam verse colon (3) Hepatic vein injury—Pringle maneuver, Rummel Head to toe physical exam tourniquet around infrahepatic (suprarenal) IVC, median sternotomy, open pericardium, Algorithm Rummel tourniquet around intrapericardial IVC, +/− atriocaval shunt ABCs (4) Extensive injuries (bilobar, hepatic venous injury, Resuscitation (IVF, full labs including PT/PTT, T+C, retrohepatic cava)—remember lessons from NGT, Foley) “damage control” surgery C-spine

134 Part 2.qxd 10/19/05 2:52 AM Page 135

Strikeouts 135

(a) mobilize liver No simple methods of controlling bleeding will work (b) gauze packing––anteriorly and posteriorly to Questions about how to perform Pringle maneuver tamponade bleeding Pt will develop post-op hemobilia/hepatic artery (c) temporary abdominal wall closure pseudoaneurysm (→ angiographic embolization) (d) return to OR––24 hours when no longer Pt undergoing nonoperative management will get septic coagulopathic and blood products available from small bowel injury (e) place drains Strikeouts Common Curveballs Not performing DPL but CT scan in unstable pt Pt will become coagulopathic intra-op Not taking pt to OR when clearly indicated Pt will have transfusion reaction Not knowing several techniques to control bleeding Pt will have associated intra/extra abdominal injuries Not doing “damage control” surgery when indicated Pt will have retrohepatic caval injury Not ruling out other injuries prior to going to OR Pt will have post-op abscess or biloma (→ percuta- Taking unstabl pt to angiography suite for embolization neously drain) Part 2.qxd 10/19/05 2:52 AM Page 136

Trauma and Critical Care—Pelvic Fracture

Concept History Should be an “AMPLE” one High frequency of associated injuries given the force Allergies needed to fracture the pelvis. Often associated with falls Meds and motor vehicle accidents. Usually classified by the vec- Past medical history tors of force that produced the injury: Last meal Events surrounding trauma Anterior-posterior compression Lateral compression Vertical shear Physical Exam Combined vector injury Head to toe Finger/scope in every hole/orifice Way Question May be Asked? Pelvic and rectal exam

“33 y/o male is brought into the emergency room after Labs/Diagnostic Studies falling off of the second story of a building. He is tachy- cardic and has a systolic blood pressure of 90. What do Full panel including T+C you want to do?” May get the scenario with patient status Lateral C-spine post MVA, fall, or crushed in an industrial accident. Be CXR systematic in the work-up and on guard for the associated Pelvis x-ray ureteral/rectal injuries and the ongoing blood loss requir- DPL if unstable ing angiography. CT scan abdomen/pelvis if stable (include head if neuro sx’s)

How to Answer? Surgical Treatment: In trauma setting, always the ABCs first: (1) Resuscitate the pt and treat any associated life- Airway and C-spine control (intubate with C-spine threatening injuries: control if necessary) (a) 2 large bore peripheral IVs Breathing and Ventilation (does pt need chest tube— 2 liters crystalloid (20 cc /kg), can repeat once place before CXR) if no response, followed by blood if hemody- Circulation and IV access namically unstable Disability (Neuro status) (2) DPL if suspect intra-abominal injury in unstable pt (incision above the umbilicus to avoid entering Don’t skip secondary survey either, or you will miss pelvic hematoma!) some key finding (high riding prostrate, blood at urethral (a) take to OR only if grossly bloody, otherwise, meatus, blood on rectal exam)! unlikely to be enough hemorrhage to be source of pt’s hypotension

136 Part 2.qxd 10/19/05 2:52 AM Page 137

Strikeouts 137

(b) if grossly bloody, position in lithotomy to be Spleen able to perform rigid sig to evaluate rectum Liver (3) Blood at urethral meatus Chest (a) urethrogram first→ suprapubic cystotomy if Small bowel positive urethrogram Pancreas (b) cystogram if urethrogram negative Pelvic fracture will be unstable (c) if cystogram positive, is injury intra or Pelvic fracture will be “open” extraperitoneal? DPL will be grossly positive (d) intraperitoneal gets primary repair in layers DPL will only be positive by RBC count (don’t do (e) consider all bladder injuries get suprapubic laparotomy first) cystotomy Pelvic hematoma will be expanding/ruptured (4) If rectal injury, Pt will have neurologic injury and be hypotensive with (a) diverting sigmoid loop colostomy grossly positive DPL (testing your priorities→ explore (b) presacral drains abdomen first as this is most life-threatening) (c) rectal washout Pt will develop DVT/PE during hospitalization (change (5) if retroperitoneal hematoma, scenario!) (a) don’t explore unless ruptured or expanding (b) if explore, ligate internal iliacs, pack, and go to angiogram for embolization if necessary Strikeouts (6) Stabilize all unstable fractures early with external fixator in ER (after DPL if hemodynamically Not knowing how to proceed or proceeding expedi- unstable) tiously in unstable pt (7) Angiogram to embolize bleeders especially if pt is Not performing DPL in unstable pt bleeding externally Performing DPL with incision below umbilicus (8) All open pelvic fractures get diverting sigmoid Not identifying associated injuries colostomy Not knowing what to do with pelvic hematoma Exploring stable retroperitoneal hematoma Common Curveballs

Associated injuries and how to manage: Ureteral Rectal Part 2.qxd 10/19/05 2:52 AM Page 138

Trauma and Critical Care—Penetrating Neck Trauma

Concept Hemorrhage—external pressure Control C-spine if any concern about injury Mortality as high as 10% with many important structures Secondary survey in stable pts to include: in close proximity. Systematic evaluation necessary and should include evaluation of potential injuries to airway, History esophagus, and vascular system. Classically, broken down into three zones: Mechanism of injury (I) clavicles to cricoid cartilage Size of weapon (proximal carotid, subclavian, vertebrals, esopha- Amount of bleeding gus, trachea, brachial plexus, spinal cord, tho- Change in neurologic status racic duct, and upper lung) Stridor (II) cricoid to angle of mandible (carotid, vertebral, jugular, larynx, esophagus, Physical Exam trachea, vagus, recurrent laryngeal, spinal cord) Hematoma (III) angle of mandible to base of skull Bruit (pharynx, distal carotid, vertebrals, parotid, cra- Crepitance nial nerves) Hemoptysis Hoarseness Bubbling from wound Way Question May Be Asked? Loss of pulses in upper extremity Horner’s syndrome “You are called to the ED to evaluate a 26 y/o male who was involved in a bar fight and sustained a stab wound to Diagnostic Tests (Stable Patients Only) his left neck. On exam, he has a 1 cm laceration at the level of his thyroid cartilage just anterior to his left SCM. What Laryngoscopy/Bronchoscopy—to assess for airway do you want to do?” Presentation may vary and don’t injuries expect much time wasted on differentiating between Zone Lateral C-spine—SQ emphysema, tracheal deviation I-III. May be given an injury to esophagus, carotid, tra- CXR—widened or pneumo-mediastinum, pneumotho- chea,...or some combination and asked how you will rax, hemothorax, tracheal deviation manage. Be sure to secure the airway early! Gastrografin swallow—assess esophageal injuries Angiography—All Zone I and Zone III injuries

How to Answer? Surgical Treatment Your history and physical exam come second here to (1) Management of the airway— basic ATLS, key points: (a) intubate any pt with difficulty in respiration, Airway Management—endotracheal intubation if depressed level of consciousness, expanding any doubt hematoma

138 Part 2.qxd 10/19/05 2:52 AM Page 139

Strikeouts 139

(b) careful of cricothyroidotomy as it may release (v) carotid injury— an underlying hematoma Repair even if comatose as this may be (c) if tracheal injury, can place ETT directly secondary to drugs or shock through wound into trachea Ligate if unstable or other life-threatening (d) watch for tension pneumothorax that might injuries compromise respiration and need urgent Ligate sup. thyroid and repair carotid end to decompression end for small defects, interpose SVG for (2) Hemorrhage— defects longer than 2 cm (a) usually controlled by direct pressure Can reach high carotid artery by: (b) avoid blindly applying hemostats anterior subluxation of mandible (c) if pulsatile or rapidly expanding, intubate, and division of omohyoid/digastric go to OR using Fogarty balloon to control proximal (d) do not probe the wound! bleeding (3) Zone I (CXR very important!) (a) stable—work-up as outlined above (b) unstable—OR for urgent exploration Common Curveballs (i) sternotomy to obtain proximal control for everything except injury to left of left Combined injuries to multiple structures in neck midclavicular line (then left anterolateral tho- Tension pneumothorax at presentation or after intuba- racotomy) tion (ii) right subclavian Trans-cervical injury (collar incision unless above thy- median sternotomy roid cartilage) (iii) inominant artery Multiple GSW with abdominal, chest, and neck median sternotomy injuries—what are your priorities? (iv) left subclavian Thoracic duct injury becomes apparent several days trap door incision post-op (4) Zone II Questions regarding for which injuries median ster- (a) explore anything that penetrates the platysma notomy vs. anterolateral thoracotomy vs. trap-door (b) prep earlobe to umbilicus and upper thigh for incisions possible SVG harvest Carotid injury in comatose patient (c) anterior SCM incision Tissue loss and circumferential carotid injury (i) esophageal injuries If < 24 h, repair injury in 2 layers if early If > 24 h, T-tube drainage to create controlled Strikeouts fistula or Esophagostomy (ii) tracheal injuries Not securing the airway Repair primarily with 3’0 vicryl Removing impaled foreign body in ED/Trauma Bay Tracheostomy if severe injury Not evaluating for tracheobroncheal or esophageal (iii) thoracic duct injuries Injury may occur in Zone I or II Not performing angiography for Zone I or III injuries Ligation acceptable Discussing use of U/S instead of angiography (iv) jugular vein Performing endoscopy before of gastrografin swallow Repair if possible for esophageal injuries Can ligate unilaterally only Probing neck wounds in Trauma Bay Watch for air embolism Part 2.qxd 10/19/05 2:52 AM Page 140

Trauma and Critical Care—Pulmonary Embolism

Concept Symptoms: Chest pain Life-threatening postoperative complication. Will likely be Dyspnea from femoral or iliac DVT. Usually associated with Hemoptysis trauma, stasis, and hypercoagulability (Virchow’s triad). Anxiety Don’t spend time here worrying about inherited defects like FV Leiden or Protein C/S deficiencies—question is Physical Exam regarding resuscitation, diagnosis, and definitive treat- ment. Check vital signs, pulse ox (tachycardia, tachypnea) Lung sounds Neck veins Way Question May be Asked? Diagnostic Tests “Called by nurse to evaluate a 45 y/o male POD#5 sig- moid colectomy now anxious, with HR 110s and pulse ox CXR (usually normal but may show decreased

85% on supplemental O2. What do you do?” Question may pulmonary vascularity, will r/o CHF or be asked several different ways with unexplained anxiety, pneumothorax) change in mental status, new hypotension in ICU patient, EKG (may show signs of right heart strain with T wave or sudden onset, pleuritic chest pain. Need to be method- and ST segment changes, need to r/o MI) ical, but expeditious, or your patient will die and you’ll be Blood gases (hypoxia and hypocarbia, very unlikely to back to take the exam next year. be PE if PO2 > 90) +/− CVP (will be elevated) V/Q scan How to Answer? Spiral CT scan (good for large PE’s) Pulmonary Angiogram (gold standard) Think of history and physical exam while resuscitating pt: Supplemental O , IVF, EKG, CXR, ABG, transfer 2 Surgical Treatment to ICU (1) Resuscitate the pt O , fluids, transfer to unit History 2 (2) Anticoagulation Risk factors: heparin bolus to keep PTT 2–3 times normal value Prior DVT 80U/kg bolus followed by 18U/kg continuous infu- Malignancy sion Pregnancy (Can use LMWH-lovenox at dose of 1mg/kg Q12) Obesity (3) Thrombolytics Prolonged immobility/Length of sugery for pt with ongoing shock despite maximal support- Recent trauma ive care (pressors, fluids, intubation, Swan-ganz Recent surgery (especially lower extremity fractures) catheter)

140 Part 2.qxd 10/19/05 2:52 AM Page 141

Strikeouts 141

can’t use in pt with recent hemorrhagic stroke, HIT after administering heparin intracranial neoplasm or recent trauma, recent Recurrent PEs on therapeutic heparin intracranial procedure, active/recent internal Being asked how to dose heparin/coumadin bleeding Forcing you to do surgical embolectomy or use throm- (4) Surgical embolectomy bolytics for pt with contraindication to thrombolytics or Pt will be pregnant (can’t use coumadin in first failure of thrombolytics with impending cardio- trimester of pregnancy) vascular collapse can support pt with femofemoral AV partial bypass until operation Strikeouts (5) IVC filter for pt with recurrent PE despite therapeutic antico- Giving thrombolytics to fresh post-op pt, or pt with agulation recent intracranial bleed/trauma/procedure for prophylaxis in pt with DVT and contraindica- Not knowing indications for IVC filter tion to anticoagulation Sending pt for V/Q scan/Spiral CT scan/angiogram for pt s/p pulmonary artery embolectomy before starting Heparin Missing diagnosis (treating as MI or sepsis) Discussing suction catheter embolectomies Common Curveballs Spending too much time on hypercoagulable work-up or on H+P in unstable pt. Fresh post-op pt (colectomy, AAA repair) Not putting patients in ICU Pt will have recent bleeding duodenal ulcer or divertic- ular bleed Part 2.qxd 10/19/05 2:52 AM Page 142

Trauma and Critical Care—Splenic Trauma

Concept Last meal Events surrounding trauma Most commonly injured organ in abdominal trauma. Will be in the setting of penetrating or blunt trauma. Gunshot Physical Exam wounds to the abdomen need exploration. Stab wounds can be managed by local wound exploration + DPL. Blunt Head to toe trauma means CT scan in stable pts, DPL in unstable pts. Finger/scope in every hole/orifice

Way Question May be Asked? Labs/Diagnostic Studies Full panel including T+C “14 y/o male is brought into the emergency room after Lateral C-spine falling off of his bike onto his left side. His chest x-ray shows CXR rib fractures of ribs 10–12. He is tachycardic to the 120s, but Pelvis x-ray not hypotensive. What do you want to do?” May get the sce- DPL if unstable nario with pt comatose and have to work through the whole CT scan abdomen/pelvis if stable (include head if neuro ABC’s and DPL may show gross blood and be faced with sx’s) splenic injury on ex lap. May get the diagnosis given to you and asked your indications to proceed to the OR. Resuscitate the Patient 2 large bore peripheral IVs How to Answer? 2 liters crystalloid (20 cc /kg), can repeat once if no response, followed by blood if hemodynamically In trauma setting, always the ABCs: unstable Airway and C-spine control (intubate with C-spine control if necessary) Surgical Treatment Breathing and Ventilation (does pt need chest tube—place before CXR) To OR when: Circulation and IV access (1) Adult pt (the incidence of OPSS is very low) so Disability (Neuro status) if there are multiple associated injuries (neuro Don’t skip secondary survey either, or you will miss injuries won’t tolerate hypotension and will pre- some key finding! clude serial abdominal assessments) or if the pt is unstable or in DIC→ splenectomy History should be AMPLE: (2) Pediatric pt, don’t transfuse more than 2 units of blood to try to stabilize the pt, if more necessary Allergies or pt becomes unstable (some won’t even trans- Meds fuse as risk of transfusion > risk of OPSS)→ Past medical history operate

142 Part 2.qxd 10/19/05 2:52 AM Page 143

Strikeouts 143

(3) Penetrating trauma will usually bring you to the atic tail, and don’t need drain unless suspect OR (can try local wound exploration if stab pancreatic injury wound, but nothing wrong with ex lap (7) Do not perform autotransplanation

Nonoperative Management Common Curveballs (1) Blunt trauma→nonoperative management as long as no free blood in abdomen on CT scan Pt will have other associated injuries (intra and extra (2) Grade 1, 2, 3 injuries abdominal) (3) Pt has minimal other injuries Pt will fail nonoperative treatment (4) Will perform serial exams, labs, CT scans and have Splenorraphy will fail low threshold for operative management Pt will get OPSS (overwhelming postsplenectomy sepsis) Will be asked how to manage pediatric pt after splenec- (1) Pack all four quadrants of the abdomen, suction tomy blood with cellsaver, try to determine source of Abscess in splenic bed post-op bleeding (clamp aorta at hiatus if necessary) Pt will have bowel injury missed in the nonoperative (2) Mobilize spleen from ligamentous attachments management until pt becomes septic and inspect Pt will be cirrhotic, be on anticoagulants, have h/o (3) Splenorraphy can be considered for stable pts severe CAD with Grade 1,2, and 3 injuries There will be pancreatic or gastric injury during (4) Splenorraphy→ splenectomy topical agents—fibrin glue (for peripheral injuries) mattress pledgeted sutures (for deeper lacerations) dexon/vicryl mesh wrap (multiple injuries with Strikeouts out time to repair) argon beam coagulator (superficial lacera Not knowing how to manage pt after splenectomy tions/capsular tears) Not knowing techniques for splenorraphy partial resection if polar injury Not doing adequate trauma work-up (5) Don’t spend time on splenorraphy if pt unsta- Not having clear criteria for nonoperative management ble or multiple other injuries Discussing autotransplantation of splenic fragments (6) If doing splenectomy, be careful to ligate short Not knowing what OPSS is or what immunizations to gastrics (not gastric wall), not to injure pancre- give pt after splenectomy Part 2.qxd 10/19/05 2:52 AM Page 144

Trauma and Critical Care—Thoracic Trauma

Concept Distended neck veins and hypotension (tamponade, tension pneumothorax) Major cause of trauma mortality. Life threatening prob- Head to toe physical exam lems should be treated as they are identified. Multiple, pos- Chest auscultation (to determine pneumothorax) sibly lethal, injuries that must be identified and treated Do not remove any foreign bodies (knives are removed promptly: in OR!) Tension Pneumothorax SQ emphysema Massive hemothorax “Sucking chest wound” Tracheobronchial tree injuries Flail segment Cardiac tamponade Traumatic aortic injury Algorithm Examiners will also be interested in determining if you know when and when not to get additional tests versus ABCs (airway first!) move directly to pericardial window, thoracotomy, or Resuscitation (IVF, full labs including PT/PTT, T+C, median sternotomy. Don’t forget to rule out other non- NGT, Foley) thoraic injuries. Lateral C-spine CXR (r/o widened mediastinum, hemothorax, apical cap, depressed left mainstem bronchus, deviated Way Question May be Asked? NGT) Pelvis x-ray “Called to ED to see a 24 y/o male who suffered a GSW to the chest. His HR is 130 and his SBP is 90/palp. What do Diagnostic Tests you want to do?” Will usually be a question testing your management priorities and if you know how to take care CT scan (not in unstable pt) of the unstable patient. Angiogram (any pt with suspicion of traumatic rupture of aorta but not in unstable pt—traumatic rupture doesn’t explain hypotension in trauma pt) How to Answer? Surgical Treatment Brief H+P while resuscitating the patient: ABCs then: (1) When to place chest tube? PMHx Absent breath sounds, even prior to CXR Meds Penetrating injury to one side of chest Allergies Initial output > 1000 cc, or > 200cc/h for four consecutive h→OR! Physical Exam (2) When to do ED thoracotomy? Penetrating trauma with actual or impending car- VS (if unstable, you will be doing procedures during diac arrest or “Signs of life” in the field and lost your evaluation) en route to ED

144 Part 2.qxd 10/19/05 2:52 AM Page 145

Strikeouts 145

Done through sixth intercostal space, left antero- If any ischemic changes, treat as MI pt in ICU lateral incision Echo will diagnose contusion, dyskinesia, tamponade, Rib spreader, hold lung superiorly with sponge valvular injury stick or assistant’s hand (9) Flail chest Open pericardium anterior to phrenic nerve Aggressive pulmonary toilet Clamp descending aorta jut above diaphragm Careful fluid management (feel for NGT) Pain control (+/− epidural, intercostal blocks) Not for blunt trauma or when lost “signs of life” Will be underlying lung contusion and > 10 minutes resuscitation en route Follow ABGs/CXR (3) When to take to OR for thoracotomy? Selective intubation based on associated CT output > 1000 cc initially injuries/respiratory status CT output > 200 cc for 4 consecutive hours Large air leak with hypoxemia (on side of injury) Esophageal injury (right thoracotomy unless distal Common Curveballs third esophagus) Left subclavian and descending aortic injury When to do thoracotomy (4) When to do median sternotomy? When to place chest tube Left supraclavicular stab wound When to take to OR Suspicion of great vessel injury (pulmonary hilum) When to do angiogram to r/o aortic injury Suspicion of injury to right inominant artery (can’t Pt will be unstable reach this through anterolateral thoracotomy) Pt will have tracheo/esophageal injury Tracheal injury Pt will need lung resection Allows access to ascending aorta, inominant, prox- Pt will have visceral herniation through diaphragmatic imal right subclavian, right carotid hernia (5) When to do pericardial window? Pt will need median sternotomy Stable pt with transmediastinal GSW Pt will have coincident closed head injury (which do Dilated neck veins/high CVP and pt in shock you approach first) (6) Diaphragmatic injuries Pt will have obvious intraabdominal injury (which do High index of suspicion (CT, MRI, U/S, fluo- you approach first) roscopy, laparoscopy) To describe where aortic injury typically occurs and Look for displaced NGT into left chest how to repair Repair in two layers with non-absorbable sutures There will be contralateral injury and place G-tube if on left side There will be post-op thoracic duct leak Repair transabdominally if diagnose early, through There will be post-op empyema thoracotomy if diagnose late secondary to intra-thoracic adhesions May need a mesh if very large defect (Gortex) Strikeouts (7) Mediastinal injuries Make sure to do bronchoscopy, esophagoscopy, Not knowing when to do ED thoracotomy (never in gastrografin swallow to rule out tracheo- blunt trauma) bronchial/esophageal injuries in penetrating Not knowing when to place CT tube (before Cxe) trauma Not knowing how to diagnose aortic injury (8) Cardiac contusion Performing CT scan in unstable pt Rarely the cause of shock in trauma pts Not knowing how to repair diaphragmatic injury Check EKG, enzyme levels, observation on telemetry Not going through ABCs Part 2.qxd 10/19/05 2:52 AM Page 146

Vascular—Abdominal Aortic Aneurysm

Concept Get proximal control of aorta quickly: If free intraperitoneal rupture, can consider An aneurysm is defined as a greater than 50% increase in balloon occulusion of the aorta, otherwise: the normal vessel diameter. Incidence in normal popula- Divide hepatogastric ligament tion ~ 2%, increase risk in patients with Peripheral Pull stomach to left Valvular Disease, coronary disease, and popliteal or Retract left hepatic lobe superiorly femoral artery aneurysms. Also, an association with fam- Aortic occluded placed against aorta and com- ily history of AAA. press against spine Distal control by clamping both iliacs Then enter retroperitoneal hematoma (often Way Question May be Asked? hematoma does dissection for you) Identify aneurysm neck and place proximal clamp “56 y/o female seen in ER with complaints of back pain, in infrarenal position and is hypotensive and has a pulsatile epigastric mass on Then give anesthesiologist time to catch up (lines, exam.” Rarely will get the patient with the classic triad of fluids, blood and blood products) flank/back/abdominal pain, hypotension, and pulsatile Don’t need systemic heparinization if ruptured abdominal mass. Mass is palpable above the umbilicus. Then open aneurysm, evacuate thrombus, suture Should also be prepared to answer for the patient referred ligate any lumbars, place graft (tube graft 24 mm to you from the PMDs office after a routine physical exam if iliacs non-aneurysmal) discovers an asymptomatic pulsatile abdominal mass. Close peritoneum over top of graft Return to ICU Be prepared to deal with hypothermia, acidosis, How to Answer? coagulopathy, renal failure, loss of pulses in an extremity, abdominal compartment syndrome, In the unstable pt, you don’t have too much time to do and post-op MI your routine H+P: Don’t perform aorto-bifem if mild ectasia/aneurys- Focused H+P while resuscitating the patient and mal dilation of iliacs (it’s just a longer operation getting them ready for the OR in already unstable pt!) Stat bedside Ultrasound Flat or lateral abdominal (lumbar) x-ray In Asymptomatic Patient CT scan with IV contrast is a really bad choice here + Fluids to keep SBP 90 is OK! (higher is associated Complete H P with increased risk of rupture) Size of AAA by U/S T+C 6–8 U PRBC and send to OR with FFP, CT scan is best preoperative study for AAA evaluation platelets, cryo, O-neg blood Risk of rupture increases with size Prep pt including neck, chest, abdomen, and thighs Any AAA more than 5 cm in size is candidate for repair Do not anesthetize pt until completely prepped and or AAA showing rapid growth (> 0–0.5 cm over 6 draped!!! months) during follow-up ≤ Xiphoid to pubis incision with knife If 5 cm, ultrasound q 6 months

146 Part 2.qxd 10/19/05 2:52 AM Page 147

Risk of Rupture 147

Cardiac work-up in all pts: persantine thallium stress Being asked what to do with pt with dementia, metasta- test tic cancer, or elderly If positive finding on cardiac work-up→ cardiac Pt will have had prior abdominal surgery cath/surgery (colectomy→reimplant IMA!) A-line, SGC, Foley intra-op and post-op Being asked your feelings about “stent-grafts” (results good so far and done in symptomatic pts, but always Operative Approach be the conservative surgeon on the Oral Exam) Be prepared to deal with common post-op problems (pt If elective, can discuss retroperitoneal approach will likely have at least one of the following): through left flank incision: hypothermia Less post-op ileus and pain acidosis Useful in pt with prior abd surgery coagulopathy Easier access to suprarenal aorta renal failure—(abdominal compartment syndrome, Only describe if have seen this before ATN, atheromatous debris, ureteral injury, Harder to access right iliac through this incision hypovolemia—favorite question of the examiners Open transperitoneal approach: is post-op AAA low UO!!!) Better pelvic exposure loss of pulses in an extremity Used in cases of rupture abdominal compartment syndrome Careful of injuring duodenum/iliac veins/intra-op ischemic left colon (reimplant IMA if stump coagulopathy pressure < 40 or Hartmann procedure post-op) post-op MI spinal cord ischemia Common Curveballs impotence

Aneurysm will be suprarenal Aneurysm will be 4 cm Strikeouts Asked when you will reimplant IMA (don’t need to if strong backbleeding or no backbleeding) Not knowing how to gain control of rupturing aorta Pt will also have obstructing or near obstructing sig- Not prepping pt widely enough (include neck and moid colon lesion (key to answer here is size of AAA thighs . . . “chin to knees”) vs. how close to obstructing is colon lesion) Not ruling out MI or other abdominal processes Pt will have had prior transverse colectomy for cancer Not palpating pulses (need to reimplant the IMA if stump pressure < 70 Addressing AAA first in pt with + persantine thallium and check arteriogram pre-op) or cardiac cath Aneurysm will present in atypical fashion: embolization Not knowing how to manage the common postopera- to the legs, inflammatory, aortacaval fistula, acute tive problems (see above) AAA thrombosis with pelvic and lower extremity Trying to perform any endovascular stent graft ischemia Not taking asymptomatic pt with rapid AAA growth to Pt will present later with: graft infection, aortoenteric the OR fistula (ax-bifem 1st followed by ligation aorta), pseudoaneurysm at anastomosis Being asked to describe your pre-op cardiac assessment Risk of Rupture (stress thallium) There will be an intra-op anomaly: horseshoe kidney, 5 cm AAA→ annual risk ~5% retroaortic renal vein, left-sided IVC 7 cm AAA→ annual risk ~20% Pt will be in shock and they will ask you if you want to get a CT or go straight to the OR (go to the OR and resuscitate en route! Part 2.qxd 10/19/05 2:52 AM Page 148

Vascular—Acute Lower Extremity Ischemia

Concept Ankle-Brachial Index (ABI) Check for abdominal/femoral/popliteal bruits/- An acute event characterized by the “6 Ps”: pain, pares- pulsatile masses thesias, pulselessness, pallor, paralysis, and pulselessness. Often hard to determine if it is secondary to thrombosis Pre-op Labs Including: on top of chronic PVD vs. acute emboli. History is very important here to make the right decision. Coagulation studies Hypercoagulability (if time and history don’t elucidate a cause) Way Question May be Asked? CPK levels (you’ll need these to make sure patient doesn’t develop rhabdomyolysis and renal failure) “A 63 y/o man was admitted to the hospital status post an inferior wall MI and just before he goes home, 5 days later, Nonoperative Therapy he develops the sudden onset of acute left leg pain. On All pts get started on Heparin exam the leg is cool to the touch and pulses are absent on Only for the pt with acute on chronic that you think the left except for the left femoral.” You’re lucky if the sce- is a thrombosis nario points you in the direction of an embolus. Need to Take pt to angiography suite for catheter-directed act promptly if you don’t want to end up doing fas- thrombolytics ciotomies (which you’ll likely have to describe anyway). Checking the pulses is critical and should tell you the level of the likely occlusion which helps you plan your operative Operative Approach management. History and PE very important here

If both femoral pulses are absent: How to Answer? Explore bilateral groins for aortic saddle embolism History should focus on risk factors: Prep abdomen in case need to obtain control of History of embolic episodes aorta Cardiac arrhythmia (a. fib) Prep infraclavicular area in case need to do extra- Recent MI with mural thrombus anatomic bypass (ax-bifem) AAA PVD If unilateral absent femoral pulse: Recent bypass surgery (Aortobifem, fem-pop) Dealing with iliac embolism or thrombosis of a stenotic iliac Physical exam should look for signs of vascular disease Explore unilateral groin Document pulses! (and compare to previously Again, need to be prepared to get to aorta and recorded pulses) axilla Neuro exam If unilateral absent popliteal pulse and femoral Hairless skin, changes in nails present: 148 Part 2.qxd 10/19/05 2:52 AM Page 149

Strikeouts 149

Dealing with an embolism at the femoral bifur- Keep track of time in the OR as fasciotomies may cation or below (but above popliteal) be necessary Explore unilateral groin Transverse arteriotomy in common femoral Common Curveballs If femorals and popliteals present but pedal pulses absent: Pt will have just had some sort of bypass surgery Embolus at popliteal trifurcation or below Post-op compartment syndrome will develop Explore popliteal below the knee Post-op rhabdomyolysis Need to perform thromboembolectomy for all Post-op acidosis, hyperkalemia from reperfusion injury three tibial arteries Pt will need amputation for severe ischemia If after recent aorta bifem, make a longitudinal Pt will develop renal failure incision on the distal femoral limb just proximal Being asked to describe your fasciotomies to the anastomosis Aortic saddle embolism will need extra-anatomic If pre-existing SFA disease, an emergency fem-pop bypass bypass could be the correct choice Pt will still not have pulses after your balloon embolec- Could try revising the distal anastomosis tomy If after recent fem-pop: Need to expose both anastomoses Strikeouts Hard to embolectomize a vein graft Usually need to revise the distal anastomosis Trying to avoid the OR with catheter-directed throm- May need to take down vein graft and replace bolytics or suction thromboembolectomy with Gortex Not taking fresh post-op pt back to the OR and expos- ing the graft and being prepared to revise or replace Remember always to get proximal and distal the graft control before opening a vessel Forgetting to anticoagulate the pt pre-op, intra-op, Fogarty balloon embolectomy and/or post-op Can do angiogram initially in the OR and use Start discussing angioscopy to identify the clot thrombolytics Forgetting to check the pulses pre-op Always do completion angiogram in OR Forgetting to check intra-op completion angiogram/ Don’t forget to consider bicarb and mannitol to checking pulses post-op protect the kidneys Part 2.qxd 10/19/05 2:52 AM Page 150

Vascular—Carotid Stenosis

Concept Remember non-invasive testing (these pts have vascular disease everywhere): Usually described as extracranial cerebrovascular disease. Arrhythmias on EKG Must have appropriate indications for surgery here! Be Carotid duplex scanning—plaque characteristics, sure to differentiate from the rare “posterior” ischemia flow velocities, % stenosis that comes from the basovertebral system. CT or MRI of brain in symptomatic pts MRA if available (if not, angiogram to include aortic arch and proximal common carotid artery) Way Question May be Asked? Indications for Surgery “A 53 y/o female seen in the ED for a TIA which resolved in the next several hours is later referred to your office for Symptomatic pts with ≥ 70% stenosis evaluation for a carotid endarterectomy. She has a bruit on Symptomatic pts with < 70% and ulcerated plaque or the left side.” failure of medical therapy Asymptomatic pts with ≥ 80% stenosis How to Answer? Crescendo TIAs

History should focus on risk factors: Nonoperative Therapy HTN Asymptomatic pt is treated with aspirin or Plavix DM Q 6 month follow-up with repeat non-invasive testing Elevated cholesterol Vascular disease elsewhere History of TIA (ocular or hemispheric symptoms) Operative Approach History of stroke Position pt supine with head elevated and turn to oppo- History should also focus on symptoms being sure to site side r/o other possibilities: Gentle prepping of neck (don’t want to injury fragile Recent MI plaque) A. fib Oblique incision along anterior border of SCM Ataxia, gait disturbances, bilateral lower extremity Dissect common carotid along medial border to avoid weakness→basovertebral ischemia vagus nerve injury Intracranial pathology Expose common carotid, internal carotid, and external carotid with minimal manipulation Physical Exam Divide facial vein as usually enters IJ at level of bifurcation Look for signs of vascular disease Apply clamps/tourniquets in disease free areas Neuro exam Anticoagulate pt with Heparin 100U/kg Bruits Select shunt (safest on the oral exam to shunt all pts BP gradient between the two arms and perform surgery under general anesthesia)

150 Part 2.qxd 10/19/05 2:52 AM Page 151

Strikeouts 151

Arteriotomy begins on proximal CCA and extends onto Pt will have expanding hematoma in neck post-op ICA (above and below gross intimal disease) Disease/plaque will continue up into base of skull Make sure to carefully back-bleed shunt free of Won’t be an appropriate vein to harvest to use as a air/debris patch Dissect plaque free of arterial wall with blunt dissector Post-op MI Make sure there is no loose flap or tack down shelf Post-op hypertension/hypotension (NTG/nitroprusside (6-O prolene double arm, vertically placed so as not or volume replacement/dopamine respectively) to constrict lumen of ICA, knots on outside of Post-op nerve injury vessel) Management of crescendo TIAs (→OR!) Close vessel with vein patch or Hemashield patch Management of pt with triple vessel heart disease and Flush all vessels before closure and releasing ICA 90% R carotid (CEA+CABG) clamp Management of carotid if at time of surgery, completely Open external first again to flush air/debris away from thrombosed ICA distribution Reversal of heparin with protamine and use of drains is surgeon dependent Strikeouts

Not being clear on your pre-op indications Common Curveballs Not differentiating from basovertebral ischemia Operating on fresh ipsilateral stroke Carotid artery will be completely occluded at time of Not getting cardiac work-up, angiogram or MRA pre- surgery op Pt will have post-op stroke or stroke after you order an Don’t start discussing things like anterior subluxation angiogram of the mandible to extend exposure unless you do Pt will have 78% stenosis and be asymptomatic this procedure! Pt will have 68% stenosis and be symptomatic Not being able to describe your operation or ways to Pt will have ulcerated plaque minimize debris in ICA Consulted to see a pt with carotid occlusion Performing blind endarterectomy (need to visualize dis- Consulted to see a pt with recent stroke (check CT, if no tal endpoint to avoid distal flaps!) stroke, heparin and CEA in 1 week, if stroke, CEA in Discussing carotid angioplasty/stents (only pt with 2 months) clear fibromuscular dysplasia unresponsive to med- Restenosis after CEA ical Tx gets dilatation) Post-op: MI, neurologic deficit in recovery room, headache, bradycardia in recovery room Pt will have acute stroke in your follow-up of an asymp- tomatic lesion (don’t rush to operate→get CT of head, treat with TPA if in first 3 h, otherwise ASA, physical therapy, and CEA in 6 weeks) Part 2.qxd 10/19/05 2:52 AM Page 152

Vascular—Chronic Lower Extremity Ischemia

Concept Remember to perform pre-op cardiac work-up

Peripheral vascular disease in the extremities that may Nonoperative Therapy present in several ways. Often times the examiners here will be testing your knowledge of the indications for surgery Risk reduction (smoking cessation, weight reduction, and your nonoperative management of PVD. control of glucose levels and SBP) Graded exercise program Anti-platelet therapy (Trental) Way Question May be Asked?

“45 y/o male presents to the office for evaluation of pain Non-Invasive Vascular Laboratory in his right thigh and calf after walking 3 blocks. He is a smoker, overweight, and is currently on no medications.” Doppler ultrasound Segmental PVRs Exercise treadmill testing How to Answer?

Be specific in your indications for surgery: Invasive Tests Rest pain Non-healing ulcer Aortogram with distal run-off or Gangrene MR Angiogram Intermittent claudication that: Only take claudicant to surgery after long term follow-up Fails to improve with nonoperative therapies has shown failure of nonoperative therapies and the pt has Severe enough to interfere with person’s been committed to risk reduction (smoker doesn’t get lifestyle bypass!

History should focus on risk factors: Smoking Operative Approach DM HTN Always need adequate inflow and adequate Overweight outflow Isolated aortoiliac lesion is amenable to angioplasty/ Physical exam should look for signs of vascular disease stent Document pulses! Try to use vein if possible (in-situ or reversed SVG)— Neuro exam assuming pt has not had cardiac surgery— Hairless skin, changes in nails SVG harvesting (autogenous grafts have best Ankle-Brachial Index patency rates)— Check for abdominal/femoral bruits pre-op duplex will evaluate the suitability of vein and can map location

152 Part 2.qxd 10/19/05 2:52 AM Page 153

Strikeouts 153

If using prosthetic graft, use PTFE Common Curveballs Choice of distal target depends on pre-op arteriography Vertical incision to expose common femoral artery at Post-op hemorrhage groin Post-op graft thrombosis or infection Medial approach to expose popliteal above the knee Any stent placed will occlude Angioplasty performed will restenose Exposure of peroneal and posterior tibial below trifur- Angioplasty performed will lead to vessel dissection cation are best exposed by detaching the soleus mus- Post-op compartment syndrome will develop cle from the tibia Pt will develop heparin induced thrombocytopenia Anterior tibial exposed by a longitudinal incision two fingers-breadths lateral to the anterior tibial border Strikeouts exposing the anterior tibial between the anterior tib- ialis and extensor digitorum longus Not being clear in your indications for surgery For in-situ grafts, valves are incised through side Not trying nonoperative therapy for the intermittent branches using Mills valvulotome claudicant Not knowing your non-invasive vascular studies Patency rates for infra-inguinal bypass for disabling Not taking pt back to surgery for immediate post-op claudication: graft thrombosis ~70% at 5 years Trying to describe a technique you don’t do (in-situ Completion angiogram in OR SVG) Don’t forget to give heparin intra-op Not being prepared for postoperative complications Not being able to read angiogram if handed to you Spending lots of time on endovascular techniques. Part 2.qxd 10/19/05 2:52 AM Page 154

Vascular—Venous Stasis Ulcer

Concept Brodie-trendelenburg test (elevate leg until drained of venous blood, place tourniquet below knee, have pt Spectrum of valvular incompetence and chronic venous stand, see if more blood flows into varicosities outflow obstruction with resultant venous hypertension, when tourniquet released after they fill from arte- edema, cellulitis, ulcers. Valvular incompetence accounts rial pressure (30 seconds)→tests superficial reflux for 90% cases, DVT for other 10%. Data Way Question May be Asked? Labs including PT/PTT, Factor V Leiden, Protein C and S, ATIII level

“A 49 y/o female G4P4 comes to your office complaining of Check ABI (if < 0.6, question your diagnosis) pain and swelling in the lower legs, varicose veins, and a Duplex scanning to determine sites of obstruction and non-healing ulcer on the medial aspect of her right ankle. incompetence (deep, superficial, perforators), What do you want to do?” May even be given the increased evaluate for DVT skin pigmentation and the varicosities. Surgical Treatment How to Answer? (1) Conservative treatment with: compression therapy History leg elevation/antibiotics Vascular disease weekly application of Unna boots h/o DVT topical agents (PDGF, EGF) Clotting disorders if ulcer heals (on the exam, it won’t), pt is fitted for Prior treatments graduated compression stockings (30–40 Professions with “long standing” periods mmHg) Pregnancy (2) For superficial vein incompetence alone→ high lig- ation and stripping of greater saphenous vein Physical Exam surgery (3) For superficial and perforator incompetence→ Complete vascular exam subfascial endoscopic perforator veins Varicosities (size, location, firmness) (4) For deep vein obstruction→ air plethysmography to Calf tenderness/swelling (DVT) delineate anatomy and choose appropriate surgical Edema procedure Brawny induration/discoloration (a) deep venous obstruction from femoral/iliac Dermatitis thrombosis→ cross femoral and/or sapheno- Ulceration (most occur medial) popliteal bypass (b) deep venous reflux→ valvuloplasty

154 Part 2.qxd 10/19/05 2:52 AM Page 155

Strikeouts 155

(5) May also need STSG for healing of ulcer Pt will have recent DVT Pt will have some coagulopathy Common Curveballs Strikeouts Conservative therapies will fail Trying to get you do to variceal ablation in pt with deep Confusing with arterial ischemic ulcer vein obstruction/incompetent perforators Not performing adequate H+P Asking you to describe physical exam tests— Not trying conservative therapies Trendelenburg test Not performing duplex scan Pt will have associated vascular disease (changing sce- Performing variceal ablation in pt with deep vein nario) obstruction (this is an important collateral in these Pt will be pregnant pts and can cripple venous outflow) Part 2.qxd 10/19/05 2:52 AM Page 156

Conclusion and Common Curveballs

As a final conclusion to the information I’ve presented in The pt will have neurologic findings after a GSW to the this book, I have some parting words of advice: abdomen The chest tube placed for a hemothorax will clot off (1) dress conservatively, and make you think the output has truly decreased (2) don’t appear hesitant, The AAA will have post-op: MI, colonic ischemia, (3) don’t quote the textbooks or the literature, renal failure (4) never mention a test or procedure you can’t fully The pt started on Heparin will develop HIT describe and The low anterior resection will leak (5) remember that the stress of the exam will do strange Medical therapy will always fail things to you. The MI pt will throw a clot to the SMA 6 weeks post- I recommend a single dose of Immodium, a good movie, op and possibly a “shot” of alcohol the night before. The pt will develop a pseudoaneurysm I’ve also included a list of the most common curveballs to The pt will develop a enterocutaneous fistula after anticipate. We all see these in real life and know how to extensive LOA manage them, but it’s easy to end up with a swing and a The FNA will not be definitive in any solid mass miss when one comes at you on the exam. The examiners The sarcoma will involve an artery/nerve are trying to determine whether or not you are a safe sur- The colon cancer will have a colovesical fistula geon. Keeping that in mind, anticipate the following ahead Dissection in the pelvis for diverticulitis/cancer will of time and you won’t get shelled in anyone scenario and injure the ureter you may avoid a return trip to the Oral Exam: The colon cancer will involve the ureters The rectal cancer pt will present with large bowel The pt with a bowel obstruction will have had a recent obstruction MI The young pt with HTN will have an MEN syndrome The incarcerated/strangulated hernia will reduce on The ERCP will fail to remove an impacted stone or induction of anesthesia diagnose malignancy The trauma pt will have multi-system injury especially The uneventful lap chole will develop post-op jaundice abdominal and neurologic trauma The gallstone ileus will have more than one stone in the Consultants will never be available Pts needing laparotomy will often have had prior The pregnant pt will need surgical intervention abdominal surgery The pre-op lymphoscintigraphy for a SLN in melanoma There will be a synchronous mass in the colon cancer will light up two lymph node basins patient Sarcoma will locally recur The obstructing left colon cancer will have a cecal A.fib complicating any scenario perforation Gastric MALT-oma won’t respond to H. pylori treat- The pt will have a cervical leak after a Zenker’s ment diverticulectomy Lung resection pt will have continued air-leak The pt will perforate after upper/lower endoscopy Abscess will complicate every operation Percutaneous drains won’t work The ex lap/splenectomy/lap chole will have an unantici- The pt will be hypotensive/hypoxic in the recovery room pated ovarian mass The pt will have multiple GSWs Mobilizing the left colon will injure the spleen 156 Part 2.qxd 10/19/05 2:52 AM Page 157

Strikeouts 157

Mobilizing the right colon will injure the ureter/IVC Stomach cancer will be high on lesser curve adjacent to The colostomy will become ischemic post-op GEJ The post-op thyroidectomy will have parathyroid or Post-op Whipple will have pancreatic leak laryngeal nerve injury Duodenal stump will blow out on BII Vascular prosthetic grafts will thrombose/get infected Pancreatic necrosis will get infected The DVT will fail medical therapy or develop phlegmasia Pseudocyst will erode into adjacent structures (stom- Post-op trauma pt will get abdominal compartment ach, colon, splenic vessels) syndrome Results of percutaneous biopsies will always be incon- Excisional biopsy for DCIS will have positive margins clusive Index.qxd 10/19/05 2:50 AM Page 159

Index

A Cancer (Continued) pancreatic, 86–87 Abdominal aortic aneurysm, 146–147 rectal, 31–32 Abdominal compartment syndrome, 124–125 skin, 107–108 Abscess Carcinoid, 37–38 liver, 74–75 Carotid stenosis, 150–151 perirectal, 29–30 Cholangitis, 78–79 Achalasia, 58–59 Colitis, ulcerative, 35–36 Aldosteronism, primary, 47–48 Colon and small bowel Angina, intestinal, 23–24 acute bowel ischemia, 12–13 colon cancer, 14–16 enterocutaneous fistula, 17–18 B hemorrhoids, 19–20 incarcerated hernia, 21–22 Bleeding intestinal angina, 23–24 lower gastrointestinal, 27–28 large bowel obstruction, 25–26 upper gastrointestinal, 118–119 lower GI bleeding, 27–28 Bowel ischemia, acute, 12–13 perirectal abscess, 29–30 Bowel obstruction, neonatal, 90–91 rectal cancer, 31–32 Breast RLQ pain, 33–34 ductal carcinoma in situ, 5–6 ulcerative colitis, 35–36 inflammatory breast cancer, 7–8 Colon cancer, 14–16 invasive ductal carcinoma, 9–10 Colon trauma, 126–127 nipple discharge, 3–4 Compartment syndrome, 128–129 Paget’s disease, 11 Critical care, trauma and, see Trauma and critical care Breast cancer, inflammatory, 7–8 Cushing’s syndrome, 39–40

C D

Cancer Ductal carcinoma breast, 7–8 invasive, 9–10 colon, 14–16 in situ (DCIS), 5–6 esophageal, 60–61 Duodenal trauma, 130–131 gastric, 112–113 Duodenal ulcer, 109–111 lung, 122–123 Duodenum, stomach and, see Stomach and duodenum

159 Index.qxd 10/19/05 2:50 AM Page 160

160 Index

E Hyperthyroidism, 41–42 Hypotension in recovery room, 96–97 Emphysema, 120–121 Endocrine carcinoid, 37–38 I Cushing’s syndrome, 39–40 hyperparathyroidism, 52–53 Ileus, gallstone, 72–73 hyperthyroidism, 41–42 Incarcerated hernia, 21–22 insulinoma, 43–44 Insulinoma, 43–44 neck mass, 49–51 Invasive ductal carcinoma, 9–10 pheochromocytoma, 45–46 primary aldosteronism, 47–48 thyroid nodule, 54–55 J Enterocutaneous fistula, 17–18 Esophageal cancer, 60–61 Jaundice, postcholecystectomy, 80–81 Esophageal perforation, 62–63 Esophageal varices, 64–65 Esophagus L achalasia, 58–59 esophageal cancer, 60–61 Large bowel obstruction, 25–26 esophageal perforation, 62–63 Liver abscess, 74–75 esophageal varices, 64–65 Liver mass, 76–77 GERD, 66–67 Liver trauma, 134–135 hiatal hernia, 68–69 Lower extremity ischemia Zenker’s diverticulum, 56–57 acute, 148–149 chronic, 152–153 Lower GI bleeding, 27–28 F Lung cancer, 122–123

Fever, postoperative, 98–99 Fistula M enterocutaneous, 17–18 tracheoesophageal, 94–95 Mallory–Weiss tear, 116–117 Melanoma, 103–104 Myocardial infarction, 100 G

Gallstone ileus, 72–73 N Gastric cancer, 112–113 Gastric ulcer, 114–115 Neck mass, 49–51 Gastrointestinal bleeding Neck trauma, penetrating, 138–139 lower, 27–28 Neonatal bowel obstruction, 90–91 upper, 118–119 Nipple discharge, 3–4 Genitourinary, scrotal mass, 71 Genitourinary trauma, 132–133 GERD, 66–67 P

Paget’s disease, 11 H Pancreas acute pancreatitis, 82–83 Hemorrhoids, 19–20 chronic pancreatitis, 84–85 Hernia, incarcerated, 21–22 pancreatic cancer, 86–87 Hiatal hernia, 68–69 pancreatic pseudocyst, 88–89 Hyperparathyroidism, 52–53 Pancreatic cancer, 86–87 Index.qxd 10/19/05 2:50 AM Page 161

Index 161

Pancreatic pseudocyst, 88–89 T Pancreatitis acute, 82–83 Thoracic trauma, 144–145 chronic, 84–85 Thyroid nodule, 54–55 Pediatric surgery Tracheoesophageal fistula, 94–95 neonatal bowel obstruction, 90–91 Trauma pyloric stenosis, 92–93 colon, 126–127 tracheoesophageal fistula, 94–95 duodenal, 130–131 Pelvic fracture, 136–137 genitourinary, 132–133 Penetrating neck trauma, 138–139 liver, 134–135 Perioperative care neck, penetrating, 138–139 hypotension in recovery room, 96–97 rectal, 126–127 postoperative fever, 98–99 splenic, 142–143 recent myocardial infarction, 100 thoracic, 144–145 renal failure, 101–102 Trauma and critical care Perirectal abscess, 29–30 abdominal compartment syndrome, Pheochromocytoma, 45–46 124–125 Postcholecystectomy cholangitis, 78–79 colon trauma, 126–127 Postcholecystectomy jaundice, 80–81 compartment syndrome, 128–129 Postoperative fever, 98–99 duodenal trauma, 130–131 Primary aldosteronism, 47–48 genitourinary trauma, 132–133 Pseudocyst, pancreatic, 88–89 liver trauma, 134–135 Pulmonary embolism, 140–141 pelvic fracture, 136–137 Pyloric stenosis, 92–93 penetrating neck trauma, 138–139 pulmonary embolism, 140–141 rectal trauma, 126–127 R splenic trauma, 142–143 thoracic trauma, 144–145 Rectal cancer, 31–32 Rectal trauma, 126–127 Renal failure, 101–102 U Renal mass, 70 Ulcer duodenal, 109–111 S gastric, 114–115 venous stasis, 154–156 Sarcoma, 105–106 Ulcerative colitis, 35–36 Scrotal mass, 71 Upper gastrointestinal bleeding, 118–119 Skin and soft tissue melanoma, 103–104 sarcoma, 105–106 V skin cancer, 107–108 Skin cancer, 107–108 Vascular Small bowel, colon and, see Colon and small bowel abdominal aortic aneurysm, 146–147 Soft tissue, skin and, see Skin and soft tissue acute lower extremity ischemia, 148–149 Splenic trauma, 142–143 carotid stenosis, 150–151 Stenosis chronic lower extremity ischemia, carotid, 150–151 152–153 pyloric, 92–93 venous stasis ulcer, 154–156 Stomach and duodenum Venous stasis ulcer, 154–156 duodenal ulcer, 109–111 gastric cancer, 112–113 gastric ulcer, 114–115 Z Mallory–Weiss tear, 116–117 upper gastrointestinal bleeding, 118–119 Zenker’s diverticulum, 56–57